guía de trabajos prácticos micro 2009 deefi

93
Contenidos 0 Producción Precios y Costos CMg CMeT CMeV 10 25 30 50 1 5 6 8 g a b c d e 7 40 f M M I I C C R R O O E E C C O O N N O O M M Í Í A A G G u u í í a a d d e e e e j j e e r r c c i i c c i i o o s s A A ñ ñ o o 2 2 0 0 0 0 9 9

Upload: jesse-marquez

Post on 11-Aug-2015

1.012 views

Category:

Documents


37 download

TRANSCRIPT

Contenidos

0

Producción

Precios y Costos CMg CMeT

CMeV

10

25

30

50

1 5 6 8

g

a

b

c

d

e

7

40

f

MMIICCRROOEECCOONNOOMMÍÍAA

GGuuííaa ddee eejjeerrcciicciiooss

AAññoo 22000099

Guía de ejercicios Microeconomía- UADE

1

El uso de esta guía de ejercicios se propone como ejercitación básica para todos los

cursos de Microeconomía dictados en UADE. Se incluyen tanto ejercicios

ilustrativos de diversos libros como los elaborados por docentes de Microeconomía

de UADE.

La guía está dividida en capítulos, uno por cada unidad del programa de la materia.

Cada capítulo se encuentra subdividido en Preguntas de repaso, Ejercicios y

Estudio de casos, donde se pueden aplicar las herramientas de análisis adquiridas

en la unidad respectiva y en las anteriores.

A simple modo de ejemplo se desarrollan las respuestas de algunos ejercicios (al

final del capítulo), que tienen el símbolo a la derecha del enunciado.

Asimismo los ejercicios con el símbolo a la derecha cuentan con el resultado

al final del capítulo.

Docentes que colaboraron en la elaboración de esta guía (en orden alfabético):

• Aranguren, Mariano Daniel • Calero, Analía Verónica • Manrique, Verónica Leticia • Rottenschweiler, Sergio Gabriel • Suárez, Roque Nestor • Zaccai, Ana María

R

D

Contenidos

2

Índice

Unidad 1: Conceptos introductorios.

Unidad 2: La oferta y la demanda.

Unidad 3: La producción y los costos.

Unidad 4: El consumidor.

Unidad 5: Los mercados. Casos extremos: competencia perfecta y monopolio.

Unidad 6: Los mercados. Casos intermedios: el oligopolio y la competencia monopolística.

Unidad 7: Introducción a los mercados de factores.

Unidad 8: Incertidumbre, riesgo e información privada. Externalidades.

Bibliografía

Guía de ejercicios Microeconomía- UADE

3

Unidad 1: Conceptos introductorios.

Preguntas de repaso 1) Busque en su libro de Microeconomía y asegúrese que entendió los siguientes conceptos:

Necesidad Escasez Mercado Circuito económico Consumo Producción Eficiencia Equidad Productos Factores

Para comprobarlo pruebe su capacidad de redacción relacionando los conceptos listados en un breve texto. 2) Indique si los temas listados pertenecen a la microeconomía o a la macroeconomía o ambas:

a) La decisión de una familia sobre la cantidad de renta que debe ahorrar. b) La influencia de la legislación en la contaminación de los ríos. c) La influencia de un aumento del ahorro en el crecimiento económico. d) La decisión de una empresa sobre el número de trabajadores que debe contratar. e) La relación entre la tasa de inflación y las variaciones en la cantidad de dinero. f) El aumento del desempleo. g) Un laboratorio farmacéutico inventa y empieza a comercializar un nuevo medicamento. h) El banco RIO presta dinero a una gran empresa pero pide mayor cantidad de requisitos a una

Pyme. 3) Identifique las afirmaciones normativas y las positivas:

a) La crisis del golfo Pérsico de 1.990 subió los precios del petróleo, lo que provocó una disminución del consumo de gasolina.

b) La economía norteamericana ha crecido más que la soviética. c) Las compañías de petróleo están obteniendo excesivos beneficios. Por lo tanto, debe

obligárseles a pagar un impuesto sobre los beneficios extraordinarios. d) Si países desarrollados destinaran el 0,7% de su PIB como ayuda a los países menos

desarrollados todos estaríamos mejor. e) Debe prohibirse el consumo de tabaco en lugares cerrados porque genera externalidades

negativas. f) Las empresas deben pagar salarios superiores a aquellos trabajadores que tengan mayores g) cargas familiares.

4) “En un mundo de escasez, elegir una cosa significa, renunciar a alguna otra”. Comente al menos tres ejemplos personales dónde se halla visto en tal situación. 5) Marque la alternativa correcta:

I) La economía puede definirse como la ciencia que explica: a) las decisiones que toman las familias b) las elecciones que hacemos cuando enfrentamos la escasez c) toda conducta humana d) las decisiones que toman los políticos

II) Cuando usted compra una lata de refresco de cola:

a) usted gana pero el vendedor pierde si usted compra después de la medianoche b) usted gana y el vendedor gana c) el vendedor gana pero usted pierde porque tiene que pagar el refresco d) el vendedor gana pero usted sólo gana si es un día caluroso

Unidad 1

4

III) La tarea de la ciencia económica es:

a) tomar decisiones morales sobre asuntos como las drogas b) proteger a nuestro planeta del abuso de los recursos naturales c) decirnos qué nos conviene d) ayudarnos a entender cómo funciona el mundo económico

Ejercicios 1) Complete el cuadro y señale los puntos correspondientes en la FPP. Diferentes Posibilidades de Producción Posibilidades Comida Armas

A 15 B 14 C 2 D 9 E 4 F 5

a) ¿Por qué para producir más de un bien hay que renunciar a la producción de otro? b) En el largo plazo ¿cómo se supera esta situación? c) ¿Cuál es el costo de oportunidad (marginal) de trasladarse del punto C al D? d) ¿Y cuál es el costo total de oportunidad de esas 3 unidades de Comida? e) Señale en el gráfico los puntos G y H, donde el primero indique ineficiencia y el segundo inasequibilidad. Explique con sus palabras. Estudio de Casos 1) La economía intenta ofrecer la explicación de diferentes fenómenos que se pueden observar en la vida cotidiana de los llamados “agentes económicos”. Por este motivo, encuentre un ejemplo periodístico para cada uno de los siguientes temas tratados en la teoría económica:

a) Análisis positivo. b) Análisis normativo. c) Funcionamiento de los mercados.

Explique en cada caso.

AUTOEVALUACIÓN. Puntaje obtenido:

1) Diga cuál de las dos afirmaciones siguientes implica un análisis económico positivo y cuál normativo: (2 puntos) a) El racionamiento del combustible (la asignación a cada individuo de la cantidad máxima de combustible que puede comprar anualmente) es una mala política social porque interfiere en el funcionamiento del sistema de mercado. b) El racionamiento de combustible es una política con la que el número de personas cuyo bienestar empeora es mayor que el número de personas cuyo bienestar mejora. 2) Indique si las siguientes afirmaciones se refieren a la micro o macroeconomía. (2 puntos)

Frontera de Posibilidades de Producción

0

2

4

6

8

10

12

14

16

0 2 4 6Comida

Arm

as

Guía de ejercicios Microeconomía- UADE

5

a) El aumento de la inversión y el ahorro es un factor relevante del crecimiento económico b) El precio de la leche ha subido en enero c) La reglamentación sobre las emanaciones de gases tóxicos de los automóviles influye en la calidad medioambiental d) La tasa de desempleo ha bajado en el último trimestre. 3) Coloque Verdadero o Falso. Para éste último caso subraye la parte falsa para justificar: (2 puntos) a) Un modelo es una representación simplificada de la realidad que se utiliza para entender mejor situaciones de la vida real. b) La economía es el estudio de la manera en que las sociedades utilizan todos los recursos para producir bienes y servicios de la manera más equitativa. c) El costo de oportunidad de algo es aquello a lo que se debe renunciar para conseguirlo. d) El precio real de un bien es simplemente su precio absoluto. 4) Si después de dormir 8 hs. al día, le quedan 16 hs. que puede repartir entre ocio (X) y estudio: (4 puntos) a) Represente gráficamente todas las combinaciones posibles. b) ¿Cómo es la pendiente y porqué? c) Marque en el gráfico el punto que corresponde a 6 hs de ocio. ¿Cuánto tiempo le queda para estudiar? ¿Y si sólo quisiera tener 4 hs de ocio, dónde se ubicaría? (movimientos a lo largo de la curva) d) Si Ud. sólo necesitara dormir 6 hs, ¿cómo se modificaría su curva? Represente.

Unidad 1

6

Respuestas Sugeridas Autoevaluación 1) a) Normativo. Hay un juicio de valor sobre el racionamiento (es una “mala” política) b) Positivo. Estudiamos en la teoría microeconómica que la intervención del mercado mediante un racionamiento genera ineficiencias. 2) a) Macro. El crecimiento económico es un tema de la Macroeconomía. b) Micro. Se refiere a lo que ocurre con el precio de un bien en particular. c) Micro. La reglamentación afectará al mercado de automóviles, ya que será necesario incorporarles algún sistema que limite o reduzca las emanaciones, aumentando el costo de los autos. d) Macro. El desempleo es un tema de la Macroeconomía. 3) Nota: se subraya la parte incorrecta y con la flecha se indica la respuesta correcta. a) V b) F todos los recursos → los recursos escasos ; F manera más equitativa → manera más eficiente c) V d) F real → nominal. 4) b) La pendiente es constante, porque es una línea recta. c) 10 primero y 12 luego. d) se expande, como si se tratara de una “mejora tecnológica”. Más tiempo para todo. El desplazamiento es paralelo, hacia “afuera”.

Volver al Índice.

Guía de ejercicios Microeconomía- UADE

7

Unidad 2: La oferta y la demanda. Preguntas de repaso 1) Explique que ocurrirá con el precio y la cantidad de equilibrio en cada uno de los siguientes mercados, y en las situaciones que se detallan a continuación:

a) en el mercado de cereales, como consecuencia de una sequía b) en el mercado de panes dulces durante diciembre c) en el mercado de helados durante el invierno d) en el mercado de té, ante un aumento del precio del café e) en el mercado de pipas, ante un aumento del precio del tabaco.

2) Explique como afectará al precio y la cantidad transada de lustradoras eléctricas:

a) un aumento de las tarifas de electricidad b) el otorgamiento de un subsidio a los productores de lustradoras por parte del gobierno c) un aumento en los costos de producción d) la apertura a la importación de lustradoras

3) ¿Qué factores podrían elevar la demanda de hamburguesas? ¿Y la oferta? ¿Cómo afectaría la existencia de pizzas congeladas baratas el equilibrio de mercado de hamburguesas? ¿Y los salarios de los jóvenes que trabajan en Mc Donald’s? (Samuelson, 2003) 4) Ud. ya sabe muy bien, porque lo estudió en su libro de microeconomía, que no es lo mismo un movimiento a lo largo de una curva, que un desplazamiento de la misma. Teniendo en cuenta esta información, determine si las siguientes situaciones afectan la oferta o la demanda del mercado de automóviles y de qué manera (ingreso, gustos, expectativas, tecnología, precio de factores, política del gobierno). Grafique. Tiene dos ejemplos como ayuda:

a) Este año te ascendieron en el trabajo y tienes un aumento del sueldo del un 40%. (Respuesta: es un aumento del ingreso que desplaza la demanda de automóviles hacia la derecha)

b) El precio de los automóviles pasa de $ 45000 a $80000 y las cantidades ofrecidas aumentan en un 30% (Respuesta: es un movimiento a lo largo de la curva de oferta, si aumenta el precio, aumentan las cantidades ofrecidas)

c) El precio de los automóviles pasa de $80000 a $45000 y las cantidades demandadas aumentan en un 25%.

d) Baja el precio de la nafta. e) Se descubre un nuevo método de producción que reduce los costos de fabricación de

automóviles. f) Para fomentar el desarrollo de la industria local, el gobierno decide otorgar subsidios a la

producción de automóviles. d) Aumenta el precio de venta de los camiones y las fábricas automotrices modifican su

cadena de montaje pues éstos resultan más rentables. g) Te gustaría tener un automóvil Alpha Romeo en vez de un Renault. h) Mejora terriblemente la infraestructura de la ciudad: se extiende la red de subte incluso

hacia zona norte, zona sur y zona oeste. Los subtes y colectivos tienen una frecuencia de 5’, cuentan con aire acondicionado y el viaje, en cualquiera de ellos sólo cuesta $1 independientemente de la distancia.

i) Aumenta la población que tiene poder de compra. 5) Marque la alternativa correcta.

I) Si aumenta el precio de un bien: a) aumentará la demanda de dicho bien b) disminuirá la demanda de dicho bien c) aumentará la demanda de los bienes complementarios d) aumentará la demanda de los bienes sustitutos

Unidad 3

8

II) En la venta de entradas para el partido final del Mundial de Fútbol se formaron largas colas en los puestos de ventas y los revendedores vendieron entradas a precios exorbitantes. Este comportamiento refleja:

a) que los precios a los cuales se vendían las entradas en los puntos de ventas oficiales estaban por debajo del nivel de equilibrio b) que la curva de demanda de entradas para la final del mundial tiene pendiente positiva c) un incremento en la oferta de finales de mundiales d) que los precios cobrados en los puntos de venta son muy elevados ya que le sacan hasta el último céntimo a los asistentes al partido. e) que existen precios mínimos en el mercado de entradas a finales de fútbol

III) La curva de oferta se desplazará ante cambios en:

a) la tecnología b) la renta c) los gustos de la gente d) el número de compradores e) el precio de mercado

IV) El precio del tomate sube y la gente compra menos cebollas. Usted infiere que las cebollas y los tomates son _________________.

a) sustitutos b) bienes inferiores c) bienes normales d) complementarios

V) Si el precio del alquiler de videos está por debajo del precio de equilibrio, la cantidad ofrecida es _______ que la cantidad demandada. Si el precio del alquiler de videos está por arriba del precio de equilibrio, la cantidad ofrecida es _______ que la cantidad demandada.

a) mayor; menor b) menor; menor c) menor; mayor d) mayor; mayor

VI) El pollo y el pescado son sustitutos. Si el precio del pollo aumenta, la demanda de pescado_____________________________.

a) no cambiará pero habrá un movimiento a lo largo de la curva de demanda de pescado. b) aumentará y la curva de demanda de pescado se desplazará a la derecha. c) aumentará o disminuirá pero la curva de demanda de pollo no cambiará. d) disminuirá y la curva de demanda de pescado se desplazará a la izquierda.

5) Busque en su libro de texto los dos argumentos que se esgrimen para la intervención del Estado en la economía. Dentro de las razones de eficiencia, enumere las tres más importantes. 6) Marque la alternativa correcta.

I) Si la demanda de un bien es inelástica, la elasticidad precio de la demanda (en valor absoluto)____________________.

a) Es menor que cero b) está entre 0 y 1 c) es igual a 1 d) es mayor 1

II) Debido a que la pasta de dientes y los cepillos de dientes son complementos, la elasticidad____________ de la demanda es_________.

a) ingreso; positiva b) ingreso; negativa c) cruzada; negativa d) cruzada; positiva

Guía de ejercicios Microeconomía- UADE

9

III) Los viajes en taxi y los cortes de cabello son bienes normales. La elasticidad ________ de la demanda es ____________.

a) cruzada; positiva b) ingreso; positiva c) ingreso; negativa d) cruzada; negativa

IV) El hecho de que a medida que la renta aumenta el porcentaje de ésta que los consumidores gastan en alimentos cae, indica que los alimentos:

a) son un bien normal con demanda elástica b) son un bien inferior c) tienen una elasticidad renta de la demanda menor a uno d) que la gente más rica no come

V) Si la elasticidad precio de la oferta de un bien es 2, entonces se puede concluir que: a) la oferta es inelástica b) un aumento en el precio ocasiona una caída más que proporcional en la cantidad ofrecida c) un aumento en el precio ocasiona un aumento menos que proporcional en la cantidad

ofrecida d) una caída en el precio lleva a una caída más que proporcional en la cantidad ofrecida. VI) Si el precio de la insulina sube 5%, la cantidad demandada de insulina no cambia. En ese rango de precios, la demanda de insulina: a) es infinitamente elástica b) presenta elasticidad precio igual a uno c) es totalmente inelástica d) presenta elasticidad renta mayor que uno

VII) Si el dueño de una fábrica de helados cobra $12 el kilo, su ingreso es de $540 por día. Si el precio baja a $10 el kilo, su ingreso por ventas disminuye a $500. Entonces, la demanda de helados en el tramo considerado: a) es elástica b) es inelástica c) presenta elasticidad unitaria d) no es elástica ni inelástica porque esta situación viola la ley de demanda

Unidad 3

10

D

Ejercicios 1) Utilizando los datos siguientes, trace las curvas de oferta, demanda y averigüe el precio y la cantidad de equilibrio (Samuelson, 2003)

OFERTA Y DEMANDA DE PIZZAS Precio ($ por Pizza) Cantidad demandada

(pizzas por semestre) Cantidad ofrecida

(pizzas por semestre) 10 0 40 8 10 30 6 20 20 4 30 10 2 40 0 0 50 0

¿Qué ocurriría si la demanda de pizzas se triplicara a cada uno de los precios? ¿Y si se fijara un precio inicial de $4 por pizza? 2) Las curvas de oferta y demanda de relojes de pared son lineales. Se sabe que para el precio de 5 los oferentes desearían vender 1250 relojes y los demandantes querrían comprar 8000, y a un precio de 10 se ofrecerían 1500 relojes y se demandarían 6000. a) Encuentre las funciones de oferta y demanda. b) Calcule el precio y la cantidad de equilibrio de relojes de pared c) Calcule la elasticidad de la demanda y la elasticidad de la oferta en el punto de equilibrio. 3) Suponga un mercado en el cual se conoce la función de oferta y la función de demanda, donde: qd = 50 – 10 p qo = 20 + 5 p a) Grafique ambas funciones y encuentre el precio y la cantidad de equilibrio (analítica y

gráficamente) b) Calcule la magnitud de los excesos de oferta o de demanda correspondientes a los siguientes

precios: p1 = 1; p2 = 2; p3 = 2,5 Suponga que la función de demanda cambia y ahora es qd’= 70 – 10 p c) Enumere las posibles causas de este cambio en la demanda d) Que situación se presentaría en el mercado con la nueva función de demanda mientras prevalezca el precio de equilibrio anterior al cambio? e) Obtenga el nuevo precio y la nueva cantidad de equilibrio. 4) Suponga que la demanda de localidades para el recital de Cold Play en Argentina responde a la siguiente expresión Xd = 90.000 – 400 p. El teatro donde se llevará a cabo tiene lugar para 50.000 espectadores. a) ¿A que precio se llenará el teatro? b) ¿Cuál es el precio al que debe venderse la entrada para maximizar los ingresos totales?. c) ¿Si la entrada fuera gratis, cuanta gente quisiera ir al concierto? ¿Es esto posible? Explique e ilustre con un gráfico. d) Si se coloca un precio máximo de $75 ¿cuánta gente estaría dispuesta a asistir al concierto? ¿Qué sucede con la oferta? e) ¿Qué precio haría que ninguna persona asista al concierto? 5) Suponga que las curvas de oferta y demanda de un cierto mercado vienen dadas por las siguientes funciones:

Xd(p) = 400 – 0,25p

Xs(p) = p – 100

Guía de ejercicios Microeconomía- UADE

11

a) Calcule el precio y la cantidad de equilibrio. Obtenga las elasticidades de oferta y demanda al precio de equilibrio.

b) ¿Qué pasará con el precio que pagan los consumidores y el que reciben los productores, si el gobierno impone un impuesto al consumo de $100 por unidad? ¿En quién recae la mayor parte de la carga del impuesto y porqué?

c) Suponga ahora que la oferta ha cambiado y viene dada por la curva xs(p) = 0,1p +50. Obtenga el precio de equilibrio antes y después del impuesto. Calcule las elasticidades de oferta y demanda en el punto de equilibrio antes del impuesto. ¿En quién recae ahora la mayor parte de la carga del impuesto?

6) La demanda de mercado, por parte de los alumnos de primer curso de las licenciaturas en Economía y en Administración y Dirección de Empresas, del libro de Microeconomía de Krugman, viene dada por la expresión D(p) = 1000 – 10p. A su vez, este libro puede ser adquirido en 100 librerías distintas, aunque cada una de ellas tiene la misma función de oferta individual Si(p) = -5 + 0,2p para cada i = 1,..., 100. a) Obtenga la función de oferta de mercado y el equilibrio competitivo. b) Muchos alumnos optan por fotocopiar el libro en lugar de adquirirlo, violando, con ello, los

derechos de propiedad intelectual. Ante este problema, los libreros solicitan al gobierno que conceda una subvención con el fin de limitar el alcance de la práctica de la piratería. Si se desea conseguir que las ventas asciendan a 700 ejemplares, ¿a cuánto tendría que ascender la subvención que habría que otorgar por cada ejemplar? Representar gráficamente esta situación.

c) Dadas las importantes restricciones presupuestarias que tiene el Ministerio de Educación y Cultura, la Sra. Ministra plantea, como alternativa a la subvención, fijar un precio máximo de venta de $30. Obtenga el resultado correspondiente a esta regulación. ¿Se conseguirá, con esta medida, reducir el recurso al uso fraudulento de la fotocopia del libro? Representar gráficamente.

7) Suponga que la demanda y la oferta de X responden a las siguientes expresiones: X P I P Pd x c s= − + − +100 2 6 X Ps x= +20 Donde: I = ingreso; Pc = precio del bien complementario y Ps = precio del bien sustituto; Xd = demanda del bien “X”; Xs = oferta del bien “x”.

a) Determine la cantidad y el precio de equilibrio para los valores I=20, Pc=10 y Ps=5. b) Determine lo mismo para los valores I=60 en vez de I=20, con todos los demás valores

igual que antes.

c) Determine lo mismo para Pc = 20 en vez de 10, cuando todos los valores de las demás variables son los indicados en (a).

d) Determine lo mismo para Ps = 3 en vez de 5, si todos los demás valores son los indicados en (a). Haga los gráficos correspondientes.

Marque claramente los efectos que tienen los cambios en el ingreso, en el precio de los sustitutos y en el precio de los complementarios sobre la demanda de X y sobre el precio y la cantidad transada de X. 8) Encuentre la elasticidad precio cruzada de la demanda de amapolas con respecto al precio de los helechos, y la elasticidad ingreso de la demanda de amapolas, a partir de la información contenida en el siguiente cuadro. Explique fehacientemente los resultados obtenidos.

ANTES DESPUES

Precio Cantidad demandada Precio Cantidad

demandada Amapolas 10 120 10 100 Helechos 15 300 10 350

ANTES DESPUES

Ingreso Cantidad demandada Ingreso Cantidad

demandada Amapolas 1200 100 8000 50

Unidad 3

12

9) La siguiente tabla contiene información sobre la demanda de naranjas y manzanas en tres días distintos.

Dia Cantidad de naranjas (Kg.)

Precio de las naranjas ($/Kg.)

Precio de las manzanas

($/Kg.)

Renta de los consumidores

($) 1 10 5 1 1000 2 15 5 1 2000 3 8 5 0,5 1000

a) ¿Qué elasticidades puede calcular con los datos anteriores? Calcúlelas. b) ¿Qué generalizaciones puede obtener acerca de las naranjas?

10) Se conoce la siguiente información sobre la cantidad demandada (Qx y Qy), y los precios (Px y Py) de los bienes X e Y que elige cierto consumidor con un ingreso monetario (I) : Qx = 400; Px = 4; Py = 8; I = 2000 Si además se conoce que la elasticidad precio de X es igual a -1, que la elasticidad ingreso para X es 1,5 y que la elasticidad cruzada entre X e Y es -2, entonces explique que pasa si:

a) El precio del bien X baja a $3. b) El ingreso sube a $2.500. c) El precio de Y sube a $10. d) ¿Qué tipo de bien es X con respecto al ingreso y al bien Y?

Guía de ejercicios Microeconomía- UADE

13

Estudio de Casos 1) Lea el siguiente artículo del diario La Capital y luego responda las preguntas sugeridas al final del mismo:

Diario La Capital 28/10/08

La pelea por la soja abre otro frente

Como habían anticipado distintos analistas del mercado, la caída de los precios de los granos desde los récords de mitad de año, sumado al clima enrarecido a nivel local, provocan una importante retracción vendedora por parte de los productores agropecuarios ,que se tradujo en un parate en el eslabón del procesamiento y la exportación.

Pablo Reguera, secretario general del gremio de los aceiteros, confirmó que hace 30 días están paradas las plantas de Vicentín y Buyatti por falta de mercadería. En el plano laboral, esta parálisis provocó el adelanto de las vacaciones de los trabajadores. Por ahora esas medidas corren por carriles diferentes a los de otros sectores económicos que redujeron plantel acusando menor demanda. Están especulando para que le quiten algún punto de retención o que suban el dólar", denunció el dirigente.

La industria aceitera viene trabajando por debajo de su capacidad instalada desde hace varios meses, producto del conflicto agropecuario y luego de la retracción de la oferta, que no quiere convalidar la baja de precios del mercado internacional.

Un reciente informe de la consultora "Granos del Paraná" indicó que en septiembre el 21% del total de la cosecha de granos (trigo, maíz, soja y girasol) permanecía en poder de los productores. Tampoco hay muchos negocios con la futura cosecha. En soja, la próxima entrada de la cosecha americana es un factor bajista adicional, lo cual induciría a los productores a aguantar hasta que pase esa maroma. Algunas industrias ofrecen a los productores desdoblar el precio: 70% a valor de pizarra y 30% a fijar en dólares.

Los acopios registran también un menor ritmo de actividad. Estacionalmente este es un período de baja pero admitieron que el ritmo al que está saliendo la mercadería es menor en 50% al del año pasado. En el punto final de la cadena, industrias y exportadores, la modorra comercial provoca un difícil balance, entre trabajar a contramargen para hacerse de la soja y cumplir con los compromisos al exterior.

a) ¿Qué forma tiene la curva de oferta de soja de acuerdo a este artículo? b) Enumere las posibles causas de la caída del precio de los granos mencionada c) ¿Cómo se verá afectado el mercado de aceites a partir de la decisión de los productores agropecuarios de retener parte de la cosecha? d) ¿Qué medidas podría tomar el Estado para solucionar el problema planteado en el artículo?

2) La Batalla del precio del Gas Natural (Mansfield, 1998)

a) El argumento básico de los oponentes al precio máximo era que estaba debajo del precio de equilibrio, como se ve abajo. (OP es el precio máximo) ¿Cuál es el efecto probable?

Una de las más sangrientas batallas económicas en el Congreso al final de la década de 1970 fue sobre el precio tope del gas natural (en el comercio interestadual) fijado por el gobierno. Tal tope surgió como resultado de la decisión de 1954 de Suprema Corte de que la Comisión Federal Reguladora de la Energía (entonces Comisión Eléctrica Federal) regulara el precio del gas natural. En 1977 y 1978 hubo una puja entre aquellos que favorecían la continuación del precio máximo y aquellos que se oponían. Esta puja se libró con vigor, si no con ferocidad, que ya estaban en juego miles de millones de dólares. A fines de 1978 se aprobó la legislación que imponía la desregulación del precio del gas nuevo desde el 1º de enero de 1985.

Unidad 3

14

b) Si el precio máximo estaba debajo del precio de equilibrio ¿qué ventaja podía obtenerse permitiendo el aumento del precio del gas natural?

c) Muchos de los que estaban a favor de continuar con el precio máximo, argumentaban que la curva de oferta del gas natural tiene una elasticidad precio muy baja. ¿Por qué se consideraba esto relevante?

d) Muchos observadores también cuestionaban que el mercado del gas natural fuera competitivo. ¿Por qué se consideraba esto relevante?

3) Argentina y Uruguay exportan carne y trigo, que son bienes de demanda inelástica respecto a la renta. Discuta porqué perdió importancia relativa el consumo de estos bienes en los países desarrollados con respecto al gasto en bienes de mayor valor agregado, como computadoras, telefonía celular, viajes en avión al exterior. (Samuelson, 2003) 4) A principios de 1980, en un intento por aumentar la recaudación de impuestos, el alcalde de Washington, Marion Barry, decidió subir el impuesto municipal sobre la gasolina. El 6 de agosto de 1980 el ayuntamiento lo subió de 10 centavos el galón (medida de capacidad usada en EEUU que es equivalente a 3,785 litros) a 18. Esta subida elevó el precio de venta final en 8 centavos.

El Departamento de Finanzas e Ingresos de la ciudad conocía la ley de la demanda. Pero pensaba que la reducción de la cantidad demandada (ventas de gasolina) sería muy pequeña en relación con la subida del impuesto sobre la gasolina. Los economistas habían estimado sistemáticamente que la elasticidad-precio de la demanda de gasolina era muy baja.

Desgraciadamente, las proyecciones de la ciudad resultaron claramente erróneas. En agosto de 1980 las ventas de gasolina de la ciudad de Washington descendieron de 16 millones de galones al mes a 11 millones solamente. Cerraron diez estaciones de servicio y fueron despedidos más de 300 trabajadores. a) Explique utilizando un gráfico de oferta y demanda, porqué cayó el nivel de empleo en las estaciones de servicio de Washington. b) ¿Cuál fue el error que cometió el ayuntamiento de la ciudad, al evaluar el posible efecto de la suba del impuesto? (Sugerencia: tenga en cuenta que la ciudad de Washington está a menos de 2 kilometros del estado de Maryland).

Precio del gas natural

Cantidad de gas natural

Oferta

Demanda

0

P

Qs Qd

Guía de ejercicios Microeconomía- UADE

15

AUTOEVALUACIÓN. Puntaje obtenido: 1) Considere el siguiente caso: Ud. acaba de conseguir un nuevo trabajo en una compañía de indumentaria. Revisando unos informes descubrió que cuando la empresa decidió aumentar el precio de su línea de remeras básicas de $35 a $40, la demanda disminuyó de 55 mil a 45 mil unidades. Le piden asesoramiento a Ud. acerca de la política de precios para obtener más ingresos. Ud. qué les aconseja? (2 puntos) 2) Considere el mercado de del whisky cuyas funciones de demanda y oferta son: (4 puntos)

D(p) = 100 – p S(p) = 2p – 8.

a) Calcule el equilibrio de mercado y represente gráficamente. Tenga en cuenta que para el presente caso, p estará expresado en $ y q en miles de litros/mes.

b) Obtenga las elasticidades de oferta y demanda al precio de equilibrio. ¿Sobre quién le parece que recaería en mayor medida la carga de un impuesto?

c) Calcule el nuevo equilibrio que tendrá lugar si la autoridad pública grava el consumo del bien con un impuesto de $ 12 por unidad.

d) Compare con la situación inicial ¿Qué parte del impuesto se traslada a los consumidores y porqué?

3) Coloque Verdadero o Falso. Para éste último caso subraye la parte falsa para justificar: (4 puntos)

a) Si la demanda de asientos para una obra de teatro cuya sala cuenta con 250 butacas, viene expresada por Qd= 1000-15P, el precio que se debe cobrar para llenar todas las butacas es de $25. b) Para el caso anterior, si la entrada fuera gratis, 750 personas quedarían de pie. c) Si sube el precio de la entrada a la obra de teatro, la demanda de entradas al cine también sube porque son bienes complementarios entre sí. d) Si un arquitecto reorganiza la distribución de butacas y aprovechando los espacios muertos logra introducir 50 nuevos asientos, el precio de las entradas baja porque aumenta la demanda. e) Si el Estado fija un precio máximo a la entrada para esta determinada obra de teatro los días domingo, de manera que todos puedan acceder, lo hace por razones de eficiencia. f) La ley que prohíbe fumar dentro de los espacios públicos, como el teatro, es una intervención del Estado para corregir una falla de mercado denominada externalidad negativa. g) Si aumenta el ingreso de todos los consumidores, la cantidad demandada de entradas de teatro aumentará, pues es un bien normal. h) Si baja el precio de la entrada por debajo del nivel de equilibrio habrá exceso de demanda o insuficiencia de oferta.

Unidad 3

16

Respuestas Sugeridas

5) a) Para encontrar el precio de equilibrio, igualamos las ecuaciones de oferta y demanda:

Xd=Xs 400 – 0,25p = p – 100 500 = 1,25p p = 400

Una vez obtenido el precio de equilibrio, reemplazamos en cualquiera de las ecuaciones, ya que con ambas debemos obtener la misma cantidad. Asi la cantidad de equilibrio es Q = 300.

Seguidamente calculamos las elasticidades,

Edp= ∆qd/∆p . p/qd = -0,25 . 400/300 = -0,33 inelástica

Esp= ∆qs/∆p . p/qs = 1 . 400/300 = 1,33 elástica

(el cociente de incrementos es igual la pendiente de la ecuación)

b) Si el gobierno determina un impuesto de $100 por unidad, debemos replantear el equilibrio, separando el precio de venta que paga el consumidor (Pc) y el precio que termina recibiendo el vendedor del bien (Pv = Pc – 100).

Xd=Xs 400 – 0,25pc = pv – 100 400 – 0,25 pc = (pc – 100) – 100

600 = 1,25pc pc = 480 y pv = 380

La cantidad de equilibrio se obtiene reemplazando en la demanda el precio que paga el que compra el bien o reemplazando en la oferta el precio que recibe el vendedor del bien. Con el impuesto la cantidad de equilibrio es Q = 280.

De los $100 del impuesto, el consumidor paga $80 y el vendedor $20 (que es la diferencia entre el precio antes del impuesto y después del impuesto). La incidencia es mayor sobre los consumidores, ya que su demanda es inelástica, mientras que la oferta es elástica.

c) Rehacemos los cálculos con la nueva oferta:

Xd=Xs 400 – 0,25p = 0,1p + 50 350 = 0,35p p = 1000 y Q = 150

Edp= ∆qd/∆p . p/qd = -0,25 . 1000/150 = -1,66 elástica

Esp= ∆qs/∆p . p/qs = 0,1 . 1000/150 = 0,66 inelástica

Por lo tanto ahora con el impuesto el efecto será inverso: la mayor carga del impuesto recae sobre el vendedor ya que la oferta es más inelástica que la demanda:

Xd=Xs 400 – 0,25pc = 0,1 pv +50 400 – 0,25 pc = 0,1 (pc – 100) + 50

400 – 0,25 pc = 0,1 pc – 10 + 50 360 = 0,35pc pc = 1028,57 y pv = 928,57

Ahora la cantidad de equilibrio es Q = 142,86

Autoevaluación

1) El precio sube un 14,3% mientras que la cantidad demandada baja un 18,2%. Por lo tanto la elasticidad precio de la demanda es de -1,27 (si utilizamos la fórmula de la elasticidad arco el resultado es -1,53). Por lo tanto la demanda es elástica (el valor de la elasticidad es en valor absoluto mayor que uno), y entonces el aumento en el precio provocó una reducción en la cantidad demandada en una proporción mucho mayor, reduciendo el ingreso total para la empresa. Podemos verificar el resultado calculando el ingreso por ventas antes y después del aumento del precio: Ingreso antes $35 x 55 mil remeras = $1.925.000 Ingreso después $40 x 45 mil remeras = $1.800.000 Por lo tanto, como asesores debemos recomendar una reducción de precios para mejorar los ingresos por ventas. 2) a) Cantidad de equilibrio = 64 mil litros/mes; Precio de equilibrio = $36

D

Guía de ejercicios Microeconomía- UADE

17

b) Epd = - 0.56 < Esp = 1.12 → la incidencia del impuesto será mayor para e consumidor, ya que la demanda es menos elástica (de hecho es inelástica) que la oferta. c) La oferta con el impuesto será: S(p) = 2 (p – T) – 8, y por lo tanto el precio de equilibrio será $44 y las cantidades vendidas 56 mil litros/mes. Al vendedor le quedan $32 por litro vendido después de impuestos. d) El consumidor paga $8 más que antes, mientras que el vendedor se queda con $4 menos que en la situación sin impuestos. Por lo tanto 2/3 del impuesto son trasladados al consumidor, y el tercio restante lo paga el vendedor. 3) a) F $25. → 50. b) V. c) F. complementarios→Sustitutos d) F la demanda →la oferta. e) F eficiencia → equidad. f) V. g) V. h) V.

Volver al Índice.

Unidad 3

18

Unidad 3: La producción y los costos. Preguntas de Repaso 1) En economía existe una importante distinción entre desplazamientos de la función de producción y los movimientos a lo largo de ésta. Cite y explique al menos tres ejemplos. 2) Explique las falacias que encierran las siguientes afirmaciones: (Samuelson, 2004)

a) Los costos medios se minimizan cuando los costos marginales se encuentran en su punto mínimo.

b) Dado que los costos fijos nunca varían, el costo medio fijo es una constante, cualquiera sea el nivel de producción.

c) El costo medio es creciente siempre que lo es el costo marginal. d) Una empresa minimiza costos cuando gasta la misma cantidad en cada factor.

3) Coloque el concepto correspondiente para las siguientes definiciones. a) Describe el máximo nivel de producción que puede obtener una empresa con una cantidad de

factores. (Rta: Función de producción.) b) Cantidad total de producción que se obtiene en unidades físicas. c) Producción total dividida por el total de unidades del factor. d) Producto adicional que se obtiene al agregar una unidad más de ese factor, manteniéndose

constantes los demás. e) Cuando añadimos cantidades adicionales de un factor y mantenemos fijas todas las demás,

obtenemos una cantidad adicional de producto cada vez más pequeña. f) Un aumento equilibrado de todos los factores genera un incremento más que proporcional,

menos que proporcional o proporcional de la producción. g) Período en el que la empresa puede alterar todos los factores. h) Suma de los costos que no resultan afectados por las decisiones de producción y de los que

aumentan conforme ésta se incrementa. i) Costo que comprende los gastos de bolsillo o transacciones monetarias, así como los costos de

oportunidad más sutiles. j) Costo que no comprende el costo de oportunidad. k) Costo de una unidad adicional de producción l) Costo por unidad de producción m) Curva de nivel que muestra todas las combinaciones posibles de insumos que pueden

producir una cierta cantidad de producto n) Costos que no pueden recuperarse en el momento en que se toma una decisión o) Situación en la cual los costos medios a largo plazo disminuyen a medida que aumenta la

producción p) El costo de fabricar Q1 y Q2 juntos es inferior al costo de fabricarlos por separado.

4) Marque la alternativa correcta: I) Los rendimientos marginales decrecientes comienzan cuando:

a) el producto total es máximo b) el producto el producto medio es máximo c) el producto medio empieza a disminuir d) el producto marginal empieza a disminuir

5) Evalúe cada una de las siguientes afirmaciones: si es verdadera explique porqué; si es falsa, identifique el error y trate de corregirlo.

a) Un producto marginal decreciente nos dice que el coste marginal debe ser creciente. b) Un aumento en el coste fijo incrementa el nivel de producción de costo mínimo. c) Un incremento en el costo fijo incrementa el costo marginal.

Guía de ejercicios Microeconomía- UADE

19

d) Cuando el costo marginal está por encima del costo total medio, el costo total medio debe ser decreciente.

e) El costo total medio a corto plazo nunca puede ser menor que el costo total medio a largo plazo.

f) El costo variable medio a corto plazo nunca puede ser menor que el costo total medio a largo plazo.

g) En el largo plazo, elegir un nivel más alto de costo fijo desplaza la curva de costo total medio a largo plazo hacia arriba.

Ejercicios 1) Complete la siguiente tabla y responda:

Cantidad de Trabajo (L)

Cantidad de Capital (K)

Producción Total ( QT)

Producto Medio (Q/L)

Producto Marginal (∆Q/ ∆L)

0 10 0 --- --- 1 10 10 2 10 30 3 10 60 4 10 80 5 10 95 6 10 108 7 10 112 8 10 112 9 10 108 10 10 100

a) De acuerdo a los datos de la tabla ¿A qué plazo cree Ud. que se está haciendo referencia? ¿Por qué? b) Construya el gráfico de Producción Total (Ayuda: utilice L en el eje de las X) y explique su forma. c) Construya en otro gráfico las curvas de Producto medio y Producto marginal (Ayuda: constrúyalo debajo del anterior, de manera que sea comparable.) d) A partir de los gráficos construidos, explique la relación que existe entre el producto medio y el marginal, y de éstos con la Producción total.

2) La siguiente tabla presenta la cantidad de camisas que un taller de costura puede producir con una máquina de coser.

a) Complete la tabla e interprete los resultados obtenidos. b) Grafique los datos y señale las etapas de la producción que encuentre en este caso.

L PTL PMeL PMgL 0 0 ---- 1 12 2 22 3 30 4 36 5 40

Unidad 3

20

3) Todos los procesos detallados en la tabla ofrecen un mismo nivel de producción:

Proceso 1 2 3 4 5 6 7 8 Capital 8 7,5 5 3,8 3 2,5 1 3 Trabajo 3 1 2 3 4 5 12 14

a) ¿Se trata de un análisis de corto o largo plazo? b) Grafique la isocuanta correspondiente. c) Determine qué procesos son ineficientes. Justifique. d) Determinar la TMST de K por L al pasar del proceso 2 al 3, del 3 al 4 y del 4 al 5. ¿Se verifica la

TMST decreciente?

4) Sabiendo que la función de Cobb – Douglas es: 10 L0.5 K0.5 encuentre: a) el sendero de expansión de la empresa para cuatro pares de valores de insumos, considerando que el PK = PL = $ 2 b) El PT, medio y marginal si Ud. deja constante el capital en el primer valor elegido y cambia la cantidad de trabajo según lo hace en el punto anterior. c) En qué zona de la producción se encuentra esta función (ayúdese con el punto b) d) Que tipo de rendimientos a escala presenta esta función (ayúdese con el punto a)

5) Considere los siguientes casos de funciones de producción:

1. Q = F (K, L) = 4 K + L

2. Q = F (K, L) = min {3K, 4L}

3. Q = F (K, L) = K1/2 L1/2

a) ¿A qué tipo de función de producción corresponde cada caso? Explique como serán las isocuantas, los rendimientos a escala, y calcule la TMST.

b) ¿Cuál es el nivel de producción que se alcanza en cada caso si K = 9 y L = 4? c) Para el caso 1, ¿qué factor es más productivo y en qué proporción? d) ¿Para qué caso es mayor la sustitución entre factores y para cuál menor? e) Para cada caso calcule la Productividad media del trabajo y del capital.

6) ¿Cómo son los rendimientos a escala de las siguientes funciones de producción?

a) F (L,K) = K1/2L1/2 b) F (L,K) = 0,5 K1/4 L2 c) F (L,K) = 5 K + 6 L d) F (L,K) = 2 K1/8 L1/3

7) La tabla del producto total de Pegatodo S.A., una empresa que fabrica potes de pegamento, es la siguiente:

Trabajo (trabajadores por

semana)

Producto (potes de pegamento por

semana)

Producto Medio

Producto Marginal

1 1 2 3 3 6 4 10 5 15 6 21 7 26 8 30 9 33 10 35

R

Guía de ejercicios Microeconomía- UADE

21

a) Complete la tabla b) Dibuje las curvas de PT, PMe, y PMg c) Suponga que el precio del trabajo es de $400 a la semana, y el costo fijo total es de $10.000

a la semana. Con la tabla anterior y los datos adicionales complete la siguiente tabla y grafique:

Producto (potes por semana)

Costo Total

Costo Fijo

Costo Variable CMeT CMeV CMeF CMg

1 3 6 10 15 21 26 30 33 35

d) Suponga que el costo fijo total aumenta a $11.000 a la semana. ¿Cómo afectará esto a las

curvas dibujadas en el punto anterior? e) Suponga que el costo fijo permanece en $10.000 a la semana pero que el precio del trabajo

aumenta a $450 por semana. Dibuje las nuevas curvas de costo. 8) Dado el siguiente cuadro donde, excepto la columna (1) todo está expresado en pesos:

(1) Cantidad

(2) Costo Fijo

(3) Costo Variable

(4) Costo Total

(5) Costo Marginal

(6) Costo medio por

unidad

(7) Costo fijo medio por

unidad

(8) Costo variable medio

por unidad

q CF CV CT=CF+CV CM CMe= CT/q CMe =CF/q CvMe= Cv/q

0 55 0 55 -------------

1 85

2 110

3 130

4 160

5 210

6 280

7 370

8 480

a) Complete el cuadro. b) Realice un gráfico (o dos, uno sobre el otro) para ilustrar la relación entre el nivel de producción (eje x), el costo total (eje y) y el costo marginal (eje y). c) Realice un gráfico para ilustrar la relación entre el nivel de producción (eje x) y el costo total, fijo y variable (eje y). d) Debajo del gráfico anterior ilustre la relación entre el nivel de producción y el costo medio y marginal (eje y). ¿En qué nivel de producción el CMe es mínimo?

9) Complete el cuadro siguiente y dibuje las curvas de CMe, CVme y CMg correspondientes a esta empresa.

Unidad 3

22

Número de Clientes CT CF CV CMe CVMe CFMe CMg

0 24 1 16 2 30 3 69 4 19 5 22 6 21

10) Dada la siguiente función de producción: f(K,L) = 2LK encuentre la máxima producción posible con la condición de que los costos no superen las 100 unidades monetarias, sabiendo que los precios de los insumos son: w = 5 y r = 4. 11) El gerente de una empresa está preocupado por saber si la empresa está minimizando el coste de producción. La empresa utiliza capital y trabajo en su proceso productivo. Ambos factores son factores de producción variables. Dada la distribución de capital y trabajo en la empresa, el producto marginal del trabajo es de 49 y el producto marginal del capital es de 45. El precio del capital es 10 y el salario es 7. Analiza la situación de la empresa. ¿Está la empresa minimizando el coste? Si no es así, ¿en qué dirección deberían reasignarse el capital y el trabajo? 12) “Servicios Temporales” utiliza 4 procesadores de texto y 2 maquinas de escribir para redactar informes. El producto marginal de una máquina de escribir es de 50 páginas al día, mientras que la producción marginal de un procesador de textos es de 500 páginas al día. El precio del alquiler de una máquina de escribir es de $1 al día, mientras que el precio del alquiler de un procesador de textos es de $50 al día. ¿Está “Servicios temporales” utilizando las máquinas de escribir y los procesadores de textos de forma que minimiza los costos? (Baye, 2006. Adaptado) 13) ACME Coal pagó 5.000 dólares para alquilar un vagón de carga a la Reading Railroad. Según las condiciones contractuales, se devolverán 1.000 dólares si se devuelve el vagón en los dos días siguientes a la firma del contrato de alquiler. a) En el momento de la firma y de pagar los 5.000 dólares ¿a cuánto ascienden los costos fijos de ACME? ¿Y sus costos hundidos? b) Al día siguiente de la firma del contrato, ACME se da cuenta de que no necesita el vagón. Un granjero tiene que juntar soja y ha ofrecido subcontratar el alquiler del vagón a ACME a un precio de 4.500 dólares ¿Debe ACME aceptar la oferta del granjero? 14) David es propietario de una pequeña empresa de fabricación de hormigón. Su costo fijo es el costo de las hormigoneras y de sus camiones-hormigonera. Su costo variable es (i) el costo de la arena, grava y otras materias primas para producir el hormigón; (ii) la gasolina y el mantenimiento de su maquinaria y camiones; y (iii) sus trabajadores. David intenta decidir cuántos camiones-hormigonera comprar. Ha estimado los costos a partir de sus estimaciones sobre el número de pedidos que su empresa recibirá por semana:

CV Cantidad de camiones CF

20 pedidos 40 pedidos 60 pedidos 2 6000 2000 5000 12000 3 7000 1800 3800 10800 4 8000 1200 3600 8400

a) Para cada nivel de costo fijo, calcule el costo total de David de cubrir 20, 40 y 60 pedidos a

la semana. b) Si David está cubriendo 20 pedidos por semana, ¿cuántos camiones debería comprar y cuál

será su coste total medio? Responda la misma pregunta cuando cubre 40 pedidos a la semana y cuando cubre 60 pedidos a la semana.

c) Dibuje la curva de costo total medio de largo plazo.

D

R

Guía de ejercicios Microeconomía- UADE

23

Estudio de Casos 1) Lea el siguiente artículo de la sección Economía de LA NACION, y luego responda las preguntas sugeridas al final del mismo: Martes 6 de Marzo de 2007 En medio del rebote técnico de los mercados Nuevos datos suman incertidumbre sobre la economía norteamericana Los costos laborales tuvieron en 2006 el mayor alza desde 2000 y se desaceleró el crecimiento de la productividad; la venta de casas y las compras fabriles arrojaron números débiles en enero Inmersos en un rebote técnico, los mercados hicieron caso omiso hoy a una serie de datos negativos sobre la economía norteamericana. Un dato oficial y dos informes mostraron un repunte en la inflación salarial y señales de debilidad en el sector inmobiliario y en los pedidos fabriles, que elevan los temores de que la desaceleración en la principal economía del mundo pueda profundizarse. Pero el repunte que mostraban las principales plazas bursátiles reducía la estampida que suelen generar estos datos hacia los llamados activos seguros o de refugio, como los metales o los bonos del Tesoro de Estados Unidos. El gobierno norteamericano revisó hoy fuertemente a la baja el crecimiento de la productividad de sus empresas en el cuarto trimestre, que ahora registró un avance del 1,6% contra el 3% reportado originalmente. Si bien la desaceleración en la productividad, una medida de lo que produce un trabajador en una hora, igualó las previsiones de los analistas consultados por la agencia de noticias Reuters, tuvo como contracara un aumento de los costos laborales, lo que el mercado denomina la inflación salarial. Ese indicador es tomado en cuenta por la Reserva Federal (Fed) a la hora de decidir variaciones en su tasa de interés. Y, lógicamente, un aumento reduce las posibilidades de una baja en el costo del dinero. En todo 2006, la productividad avanzó un 1,6%, su menor incremento desde un aumento igual en 1997, según informó el Departamento de Trabajo. Este menor ritmo de la productividad hizo que los costos laborales, una medida clave sobre la inflación, subieran un 6,6% en la medición interanual en el cuarto trimestre del año anterior. Los analistas esperaban que los costos laborales fueran revisados en sentido contrario, es decir, con una aceleración en el crecimiento anual hasta el 3,2% en el cuarto trimestre desde la cifra anterior de 1,7%. Para todo 2006, los costos laborales aumentaron un 3,2%, el mayor incremento desde 2000, cuando subieron un 4,2%. Más datos sombríos. Un índice que mide el número de casas pendientes de venta cayó un 4,1 por ciento en enero, mientras que los pedidos fabriles descendieron un 5,6 por ciento en el mismo mes. "El informe sobre las casas pendientes de venta es débil y pienso que eso ayudará a sostener los precios de los bonos, pero no esperaría una escalada porque estamos siguiendo al mercado de acciones, con los datos económicos en un segundo plano", dijo Tony Crescenzi, estratega de bonos de Miller, Tabak & Co en Nueva York. Pese al alza que registraban las acciones, los inversores seguían preocupados por la posibilidad de que las bolsas vuelvan a caer y de que los activos más riesgosos puedan quedar nuevamente bajo una corriente vendedora, dijo Kim Rupert, director gerente de análisis global de Action Economics LLC en San Francisco. "Las cifras [sobre los costos laborales] son malas para los bonos porque implican ciertas presiones salariales en el camino, pero no están reaccionando de forma tan negativa a la noticia debido a factores externos", agregó Rupert.

Link corto: http://www.lanacion.com.ar/889059

a) Comente a qué tipo de productividad está haciendo referencia el artículo.

Unidad 3

24

b) ¿Cómo ha sido el comportamiento de tal productividad en los últimos tiempos? c) En la teoría ¿cuál es la relación que existe entre la productividad marginal de los factores y su costo? d) ¿Qué está sucediendo en la economía norteamericana en relación a la productividad y sus salarios?

2) La Producción Lechera (Mansfield, 1998) Si nos basamos en los datos obtenidos por el Departamento de Agricultura de Estados Unidos, pueden producirse 8.500 libras de leche durante un período especificado de tiempo con una vaca alimentada mediante las siguientes combinaciones de cantidades de pasto y grano (en libras):

Cantidad de pasto

Cantidad de grano

5000 6154 5500 5454 6000 4892 6500 4423 7000 4029 7500 3694

a) Grafique estos datos mediante una isocuanta. b) Calcule la tasa marginal de sustitución técnica de todos los puntos de esta isocuanta c) ¿Es convexa esta isocuanta? d) Si el precio de una libra de pasto es igual al precio de una libra de grano, ¿debe alimentarse una vaca con 5000 libras de paja y 6154 libras de grano?

3) Lea el siguiente artículo publicado en el diario Clarín el día 5/01/2008 y luego responda las preguntas sugeridas al final del mismo:

La leche busca el reparo Jorge Corredera tiene, en la localidad bonaerense de Ameghino, un pequeño tambo con unas 110 vacas en ordeñe. En estos días está produciendo unos 1.800 litros diarios de leche. Es decir: menos de 17 litros/día/VO. "La seca en esta zona pegó fuerte y tenemos mucha falta de pasto", dice Corredera. "Estaba dando algo de silo (de maíz), pero lo suspendí porque necesito tener reservas para el próximo otoño", añade. El problema es que, de los dos lotes de maíz 2007/08 que serán destinados a silo, uno está bastante flojo por la falta de agua. "Este invierno va a haber un faltante importante de rollos. No se ha hecho casi nada; los contratistas de rollos trabajaron muy poco en esta campaña", apunta Corredera. Lo habitual para esta época es una productividad de 20 a 21 litros/día/VO. Pero ahora las vacas hacen lo que pueden con lo que quedó de una pastura base alfalfa y algo de balanceado. Todo esto podría compensarse si estuviese recibiendo buenos precios. Pero no es el caso: una Pyme quesera de la zona apenas le paga 0,75 $/litro. Al tratarse de un tambo pequeño, los costos fijos pesan mucho. El tambero que trabaja con Corredera recibe el 15% de liquidación mensual, mientras que en los tambos medianos de la zona la remuneración se ubica en torno al 10%. "Estoy trabajando con lo justo; no puedo pensar en hacer ninguna inversión", explica Corredera. El establecimiento de este pequeño empresario cuenta con alrededor de 180 hectáreas, la mayor parte de las cuales se distribuyen en un ambiente de calidad regular. "No puedo hacer otra cosa que tambo; la escala y el campo no dan para hacer ni ganadería ni agricultura", agrega. El caso de Corredera no es una excepción, sino más bien la regla general en la zona. En Ameghino existen actualmente 35 tambos (dos décadas atrás eran más de 100) y dos tercios de los mismos cuentan con una baja escala productiva (unidades con unas 120 vacas en ordeñe). "Si esto sigue así, tengo la posibilidad de alquilarle el tambo a un empresario lechero grande de la zona; quiero seguir en la actividad, pero con estos números no puedo seguir mucho más", indica Corredera.

Guía de ejercicios Microeconomía- UADE

25

Con los precios de los granos y los subproductos subiendo mes tras mes, un escenario de precios de la leche planchados hará que muchos pequeños tamberos deban dejar de ser empresarios para pasar a ser rentistas. En este esquema, los ganadores son los propietarios de grandes tambos, que podrán producir un mayor volumen de leche y obtener así mejores precios por su producto.

a) Enumere los costos fijos y variables que tiene que enfrentar el dueño del tambo? b) ¿Cómo cree que son los rendimientos de escala de un tambo de acuerdo a lo mencionado en el artículo? c) ¿Qué opina de la mención de que buenos precios compensan bajas en la productividad? d) ¿La actividad del productor del artículo le parece inviable? ¿Qué condiciones deberían darse para que mejore su situación?

4) Caso: El costo de corto plazo de un BOEIN 747(Mansfield, 1998). El Boeing 747 es un avión que transporta muchos pasajeros en el mundo. Según los datos proporcionados por Boeing a un comité senatorial en 1975, el costo (en centavos) por pasajero-milla de operación de este avión, en un vuelo de 1.200 y 2.500 millas con 250, 300 y 350 pasajeros era el siguiente:

Número de millas 1.200 2.500 Número de

pasajeros (centavos por pasajero-milla)

250 4.3 3.4 300 3.8 3.0 350 3.5 2.7

a) Si el número de pasajeros está entre 250 y 300 ¿cuál es el costo marginal de transportar un pasajero adicional en un vuelo de 1.200 millas? b) Si el número de pasajeros es de 300 y el vuelo es de entre 1.200 y 2.500 millas, ¿cuál es el costo marginal de volar una milla adicional? c) En 1975 la tarifa económica la tarifa económica de un vuelo de 2.500 millas era de $156.60. Si un Boeing 747 transportaba 300 pasajeros en tal vuelo, ¿cubriría esto los costos operativos? d) Usted cree que la tabla de arriba puede aplicarse en 1985? ¿Porqué sí o porqué no?

5) Lea el siguiente caso de aplicación referido a costos y responda a las preguntas planteadas al final de mismo (Mansfield, 1998).

EL PROCESAMIENTO DEL ARROZ EN INDONESIA Recientemente, Indonesia le encargó a un equipo de ingenieros estadounidenses determinar qué clase de instalaciones debía adoptar para el procesamiento del arroz. Se evaluaron cuatro tipos de instalaciones: (1) descascaradotas, (2) fábricas integradas de arroz, (3) carga satélite, (4) carga terminal. En su informe inicial, el equipo recomendó que Indonesia dedicara el grueso de sus fondos a los últimos tres tipos. Poco después que se elevara este informe, C. Meter Timmer, un economista de Harvard, construyó una isocuanta para la procesamiento del arroz (correspondiente a la cantidad relevante del arroz a ser producido), siendo los insumos el trabajo y el capital. Se muestra esa isocuanta en los dos paneles de la figura. El punto sobre la isocuanta que corresponde a cada tipo de instalación está indicado. Por ejemplo, la carga terminal requiere alrededor de $78.000 de insumos de capital y alrededor de un trabajador para producir esta cantidad de producto.

Unidad 3

26

(a) Si las curvas de isocosto fueran líneas rectas paralelas a AB (como en el panel del lado izquierdo de la figura), ¿qué tipo de instalación sería óptima? (b) Si las curvas de isocosto fueran líneas paralelas a CD (como en el panel del lado derecho de la figura), ¿qué tipo de instalación sería óptima? (c) Timmer no estaba seguro cuál sería la razón entre el precio del trabajo y el precio del capital, pero estaba razonablemente seguro de que estaría entre la curva de isocosto subyacente AB y la curva isocosto subyacente CD. Si esto fuera así, ¿podemos estar razonablemente seguros de qué tipo de instalación es óptima? (d) las curvas de isocosto paralelas a AB ¿están basadas en una razón entre el precio del trabajo y el precio del capital más elevada o más baja que la curva de isocosto subyacente CD? (e) El equipo de ingeniería recomendó la utilización de cargas terminales y cargas satélites porque eran necesarias para modernizar (es su expresión) el mercadeo del arroz indonesio. ¿Es persuasivo este argumento?

40

Descas-caradora

A

20

Capital (miles de pesos) 80

60

0 20 30 B 10

10 10 Número de trabajadores

Carga terminal

Carga satélite

Fábrica Integrada de arroz

Isocuanta

40

Descas-caradora

C

20

80

60

0 D 30 10

10 10 Número de trabajadores

Carga terminal

Carga satélite

Fábrica Integrada de arroz

Isocuanta

Guía de ejercicios Microeconomía- UADE

27

AUTOEVALUACIÓN. Puntaje obtenido:

1) Elija la opción correcta (5 puntos)

I) Considere un proceso productivo con un solo factor variable. Suponga que se produce una mejora tecnológica. Entonces,

a) para representar el proceso productivo, utilizaremos otra función de producción que tenga la propiedad de que la producción obtenida es menor para cualquier nivel de factor variable que con la tecnología anterior.

b) la productividad del factor variable disminuye. c) para representar el proceso productivo, utilizaremos otra función de producción que tenga la

propiedad de que la producción obtenida es mayor o igual para cualquier nivel de factor variable que con la tecnología anterior.

d) para alcanzar el mismo nivel de producción que antes debemos utilizar mayor cantidad del factor variable.

II) Una empresa se dedica al montaje de triciclos, para lo cual adquiere chasis –C- y bolsas de tres ruedas –R- a otra empresa. Si cada trabajador emplea una unidad del factor trabajo –L- en el montaje de los mismos, la tecnología de la empresa se podría describir a través de la siguiente función de producción:

a) Q = Máx{C, R, L}, donde Q es el número de triciclos. b) Q = (C + R + L) / 3, donde Q es el número de triciclos. c) Q = Mín{C, R, L}, donde Q es el número de triciclos. d) Ninguna de las anteriores es correcta.

III) Una empresa de la construcción utiliza la siguiente tecnología: a cada trabajador le corresponde una pala. Un trabajador sin una pala no es eficaz y, por supuesto, una pala no puede cavar sola. Las isocuantas para esta empresa son:

a) líneas rectas con pendiente negativa. b) convexas y con pendiente negativa. c) cóncavas y con pendiente negativa. d) en forma de “L”.

IV) El gerente de la empresa “Cerámicas Alicantinas” sabe que el producto marginal del trabajo es 6 y que el producto marginal del capital es 30. La empresa comprará una unidad más de capital. Si la empresa desea que el nivel de producción no varíe, entonces el gerente deberá:

a) aumentar el trabajo contratado en 5 unidades. b) disminuir el trabajo contratado en 5 unidades. c) aumentar el trabajo contratado en 1/5 unidades. d) disminuir el trabajo contratado en 1/5 unidades.

V) Sea Q = F(K, L) una función de producción que describe la tecnología de una empresa. Suponga que, a corto plazo, el valor de K se puede considerar fijo en el valor K0. Entonces:

a) El producto medio y marginal del trabajo son coincidentes. b) El producto medio y marginal se cortan en el máximo del producto marginal. c) El producto medio y marginal se cortan en el máximo del producto medio. d) Ninguna de las anteriores respuestas es correcta.

VI) Los costos hundidos de una empresa son gastos:

a) que no pueden recuperarse y, por lo tanto, deben de ser tomados en cuenta en las decisiones de producción óptimas.

b) que pueden recuperarse y, por lo tanto, deben de ser tomados en cuenta en las decisiones de producción óptimas..

c) que no pueden recuperarse y, por lo tanto, no deben de ser tomados en cuenta en las decisiones de producción óptimas.

d) que pueden recuperarse y, por lo tanto, no deben de ser tomados en cuenta en las decisiones de producción óptimas.

Unidad 3

28

VII) TMB Associates, una consultora local compró un programa informático por valor de $10.000 Tras detectar algunos errores en el programa, el fabricante del software ofreció solucionarlos por un precio de $2.000. TBM encontró otro programa sin errores de otro fabricante por $6.000. ¿Debería comprar TMB el programa del segundo fabricante?

a) Sí, porque $ 6.000 es una cifra menor que $12.000. b) Sí, porque $ 6.000son menos que $ 10.000. c) No, porque $ 6.000 son más que $2.000. d) No, porque $ 6.000 son más que $0.

VIII) Carlos heredó un terreno de su abuelo. Este último pagó el equivalente a $1.000en efectivo por este mismo terreno en el año 1951. Hoy en día, el terreno en esta zona se vende a $200.000. ¿Cuál es el coste de oportunidad para Carlos si se queda con el terreno?

a) Ninguno, ya que el terreno ha sido heredado. b) Ninguno, ya que el abuelo pagó en efectivo. c) $1.000, ya que esto fue lo que le costó el terreno al abuelo de Carlos. d) $200.000, ya que esto es lo que pierde Carlos al quedarse con el terreno.

IX) Los costes fijos:

a) no varían con el nivel de producción y se incurre en ellos aunque la empresa cierre. b) varían con el nivel de producción y no se incurre en ellos si la empresa cierra. c) varían con el nivel de producción y se incurre en ellos si la empresa cierra. d) no varían con el nivel de producción y no se incurre en ellos si la empresa cierra.

X) Javier es estudiante en un instituto de enseñanza secundaria. Está pensando en matricularse en la licenciatura en economía. Julia se acaba de graduar en económicas y busca un empleo como asistente de investigación. ¿Para cuál de los dos las tasas de la universidad constituyen un coste de oportunidad?

a) Sólo para Javier. b) Sólo para Juana. c) Para ambos. d) Para ninguno de los dos.

2) Suponga que Ud. como empresario, se encontraba minimizando costos en el punto B del siguiente gráfico (2 puntos): a) ¿Las pendientes de qué curvas deben ser tangentes para asegurar la minimización de costos? b) Ud. tiene curvas con 3 canastas A, B y C. ¿Cuál de ellas implica un menor nivel de producción? c) Si Ud. decide desplazarse al punto A, ¿es porque tiene más ingreso? Justifique d) Si el presupuesto del empresario es de $1000. ¿Cuál es el precio de los factores? (Recurede que Ud. está en el punto B). 3) Good Kitchen utiliza 4 hornos a leña y 2 hornos eléctricos para producir pasteles. La Productividad marginal del horno a leña es de 25 pasteles al día y la Productividad marginal del horno eléctrico es de 250 pasteles al día. El precio del alquiler de horno a leña es de $0.50 ctvos al día, mientras que el precio del alquiler de un horno eléctrico es de $25 al día. ¿Está Good Kitchen utilizando los hornos de manera tal de minimizar los costos? (2 puntos) 4) Coloque Verdadero o Falso. Para éste último caso subraye la parte falsa para justificar: (1 punto)

a) El costo de asistir a la Universidad no es meramente el gasto de la matricula, alojamiento, libros, viáticos. También comprenden el costo de oportunidad de los ingresos que perdemos mientras estamos estudiando. b) Las economías de escala se producen cuando el costo de fabricar Q1 y Q2 juntos es inferior al coste de fabricarlos por separado.

K

B

80

42

95 200

A

C

L

Guía de ejercicios Microeconomía- UADE

29

Respuestas Sugeridas

10) Este ejercicio se puede resolver de varias maneras, aquí presentamos dos de ellas. a) El problema consiste en maximizar la función de producción f(K,L) = 2LK (1), con la restricción de que el costo sea de 100, C = 100 = w*L + r*K = 4K + 5L (2). Despejando L de (2),

L = 20 – 4/5K (3) y reemplazando en (1) tenemos que:

f(K) = 40K – 8/5K2 (4) Para maximizar la expresión (4), derivamos e igualamos a cero:

f´(K) = 40 – 16/5K = 0 K* = 25/2 E introduciendo el resultado en (3),

L* = 10 La producción alcanzada con esos valores óptimos de L y K es igual a 250. b) Para que el productor esté minimizando costos deben verificarse dos condiciones:

TMST = PMgL/PMgK = w/r (1) C = wL + rK (2)

Calculamos los productos marginales respectivos: PMgL = f’L = 2K y PMgK = f’K = 2L

Por lo tanto, TMST = 2K/2L = K/L

Reemplazando en (1), K/L = 5/4 K = 5/4 L (3)

Si introducimos (3) en (2), 100 = 5L + 4(5/4L) = 5L + 5L = 10L L* = 10

Y reemplazando en (3), K* = 25/2

6) a) Constantes b) Crecientes c) Constantes d) Decrecientes 11) No está minimizando costos. Le conviene usar más trabajo y menos capital. Autoevaluación

1) I)c II)c III)d IV)b V)c VI)c VII)c VIII)d IX)d X) a

2) a) Isocosto e isocuanta b) Canasta C c) No, es porque el L se abarata por la caída en los salarios y con el mismo presupuesto se puede adquirir mayor cantidad. Es un incremento real en los presupuesto, aunque nominalmente no varíe. d) w(salario) = 1000/200 = 5 y r (tasa de interés) = 1000/80 = 12.50

3) Comparamos (PMgHL / PHL) > (PMgHE / PHE) = (25/0.50) > (250/25). La Productividad Marginal por peso gastado en hornos a leña es mayor que la Productividad Marginal por peso gastado en hornos eléctricos. Si bien los hornos eléctricos son diez veces más productivos, también son 50 veces más caros que los hornos a leña. Good Kitchen no está minimizando costos y deberá reemplazar hornos eléctricos por hornos a leña hasta el punto que las Productividades Marginales por peso gastado en cada tipo de horno se igualen.

4) a) V b) F economías de escala. → economías de alcance.

Volver al Índice.

D

R

Unidad 4

30

Unidad 4: El consumidor.

Preguntas de Repaso 1) En cada uno de los siguientes casos, ¿el consumidor tiene una utilidad marginal decreciente, constante o creciente? Explique sus resupuestas. (Krugman, 2006)

a) Cuanto más ejercicio hace Mabel, más disfruta de cada visita adicional al gimnasio b) Aunque la colección de CDs de música clásica de Mei es enorme, cada nuevo CD que ha

comprado le ha proporcionado la misma satisfacción conforme la colección ha ido creciendo. c) Cuando Dexter era un estudiante sin muchos recursos, la satisfacción que obtenía de comer en

un buen restaurante era mayor que ahora, que lo hace más a menudo. 2) Determine en cada uno de los siguientes casos, si los dos bienes son sustitutivos perfectos, complementario perfectos o bienes regulares. Explique su respuesta, prestando especial atención a la relación marginal de sustitución de un bien por otro. (Krugman, 2006)

a) Sanjay se procupa sólo del número de caramelos que recibe, y no le preocupa si tienen sabor a plátano o a piña.

b) Para Hillary, la utilidad marginal de los pasteles de cereza aumenta conforme dispone de más bolas de helado de vainilla para comer con cada porción de pastel. Pero ella está dispuesta a consumir algo de pastel aunque no tenga helado de vainilla.

c) A pesar de las repetidas rebajas en su precio, los consumidores no comprarán software de la Compañía Ominsoft a menos que ésta también venda conjuntamente el sistema operativo que permite utilizar dichos programas en el ordenador.

d) Darnell trabaja tiempo parcial en la biblioteca de la universidad. Su jefe le ha pedido que trabaja más horas esta semana. Darnell está dispuesto a hacer trabajo adicional, pero también se dá cuenta que cuanto más horas ha trabajado, menos le apetece trabajar otra hora más. (Pista: piense que los bienes entre los que elige Darnell son renta y tiempo de ocio).

3) Marque la alternativa correcta:

I) La teoría de la utilidad marginal supone que cuando Sandra consume más de un bien que a ella le gusta:

a) su ingreso aumenta b) su utilidad total aumenta c) su utilidad total se maximiza d) su utilidad marginal aumenta

II) Conforme usted consume más y más de cualquier bien, usted experimenta una utilidad:

a) total decreciente b) marginal decreciente c) marginal creciente d) marginal negativa

III) Andrés prefiere jugar al fútbol que jugar al básquet. Si juega una hora al fútbol su ___________ será ___________ que si juega una hora al básquet.

a) utilidad marginal; igual b) utilidad total; mayor c) utilidad total; igual d) utilidad marginal; menor

IV) El principio de utilidad marginal decreciente significa que si usted pasa más y más horas viendo televisión, su utilidad total de ver televisión _______ y su utilidad marginal por cada hora adicional de televisión _______

a) Aumentará; disminuirá b) aumentará; aumentará c) disminuirá; aumentará d) disminuirá; aumentará

Guía de ejercicios Microeconomía- UADE

31

D

V) Cuando el bien es Giffen:

a) el efecto sustitución es menor que el efecto ingreso b) el efecto sustitución es mayor que el efecto ingreso c) el efecto sustitución es igual al efecto ingreso d) el efecto sustitución es menor que el efecto ingreso y es opuesto a él.

Ejercicios 1) Brunella dispone de $12 semanales para sus gastos; se los gasta en alfajores, que le cuestan $1,5 cada uno, o en gaseosa, que le cuesta $0,5 el litro. Trace su conjunto de oportunidades. ¿Cuál es el costo de oportunidad de consumir un litro de gaseosa, expresado en unidades de alfajores? Ahora, represente cada uno de los conjuntos presupuestarios correspondientes a los siguientes casos:

a) Brunella recibe $6 más a la semana. b) El precio de los alfajores baja a $1,2. c) El precio de la gaseosa sube a $1 el litro. d) El precio de la gaseosa sube a $0,60 el litro y el de los alfajores a $1,8. e) El precio de la gaseosa sube a $0,60 el litro y el de los alfajores $1,8 y la el ingreso a $14,4. f) A los precios iniciales, su dietista le señala que la cantidad máxima de gaseosa que puede

adquirir por semana es de 3 litros. g) A los precios iniciales, si consume más de 3 litros de gaseosa entonces el dueño del kiosco,

como premio por ser buena clienta, le cobra sólo $ 0.4 por litro adicional de gaseosa. 2) Suponga que la ecuación presupuestaria de un consumidor es Px.X + Py.Y = I, donde Px y Py son los precios de los bienes. X e Y son las cantidades consumidas de cada bien respectivamente, e I es el ingreso del consumidor.

a) Suponga que el gobierno decide establecer un impuesto sobre la cantidad del bien X de $4 y una subvención al bien Y de $2 por unidad. ¿Cuál será la nueva ecuación que describa la recta presupuestaria del consumidor?

b) Suponga inicialmente que Px es $120 y que Py es $80. Determine si la siguiente proposición es verdadera o falsa: “Si Px sube $18 y Py sube $12 la nueva recta presupuestaria se desplazará hacia adentro y en forma paralela a la antigua recta presupuestaria”. Explique su respuesta.

3) Suponga que José Garay gasta todo su ingreso en los bienes X e Y. La utilidad marginal de cada bien (mostrada abajo) es independiente de la cantidad consumida del otro bien. El precio de X es de $100 y el precio de Y es de $500. (Mansfield, 1998)

Utilidad marginal del señor Garay Nº de unidades consumidas del

bien Bien X Bien Y

1 20 50 2 18 45 3 16 40 4 13 35 5 10 30 6 6 25 7 4 20 8 2 15

a) Si el señor Garay tiene un ingreso de $1000 por mes, ¿cuántas unidades de cada bien debe

comprar? b) Dibuje la recta presupuestaria del señor Garay a partir de los datos de la pregunta precedente.

¿En qué punto corta al eje en que se mide la cantidad del bien Y?

Unidad 4

32

D

4) En el diagrama de abajo mostramos una de las curvas de indiferencia de Helena Blanco así como su recta presupuestaria. (Mansfield, 1998)

a) Si el precio del bien A es de $50, ¿cuál es el ingreso de la señorita Blanco? b)¿Cuál es la ecuación de su recta presupuestaria? c)¿Cuál es el precio del bien B? d) ¿Cuál es su tasa marginal de sustitución?

5) El costo del transporte en automóviles es de 30 ctvs./Km., en tanto el precio del transporte en autobús público es de 60 ctvs/Km. Actualmente la utilidad marginal para un consumidor del último kilómetro de transporte en automóvil es 80, mientras que su utilidad marginal del último kilómetro del transporte en autobús es 150. ¿Está maximizando su utilidad? Si no es así, ¿cómo debería modificar su asignación entre ambos medios de transporte para obtener la máxima utilidad posible? 6) Elena gasta $10 a la semana en pizzas y hamburguesas. El precio de la pizza es de $2 la porción, mientras que el precio de una hamburguesa es también de $2. La tabla muestra la utilidad de Elena derivada del consumo de ambos bienes. ¿Con qué combinación de pizzas y hamburguesas Elena maximiza su utilidad?

Pizzas Hamburguesas Porciones por

semana Utilidad

Total Unidades por

semana Utilidad

Total 0 0 0 0 1 100 1 70 2 150 2 130 3 175 3 180 4 190 4 220 5 195 5 250

7) Suponga que la función de utilidad de un consumidor es de la forma U = xy a) Complete el siguiente cuadro

Niveles de Utilidad U1 = XY = 25 U2 = XY = 100 U3 = XY = 225 X Y X Y X Y 2 12.5 2 2 3 8.3 3 3 4 6.3 4 4 5 5.0 5 5 6 4.2 6 6 7 3.6 7 7 8 3.1 8 8 9 2.8 9 9 10 2.5 10 10 15 1.7 15 15 25

Guía de ejercicios Microeconomía- UADE

33

D

D

b) Represente gráficamente las curvas de indiferencia correspondientes a los niveles U1 = 25, U2=100

y U3 = 225. c) Suponga que Px = 100 y Py = 100. Dibuje la curva de consumo-ingreso para este consumidor,

verifique que X e Y son normales, y dibuje la curva de Engel para el bien X. d) Con Px=100 y Py=100 siendo el ingreso I=2000, determine gráficamente las cantidades de X e Y que

maximizan la utilidad. e) Suponga ahora que Px = 200 y lo demás igual que en el punto anterior. Determine gráficamente el

nuevo equilibrio del consumidor. f) Con los datos de d) y e) trace aproximadamente la curva de demanda de X. 8) Un consumidor tiene como función de utilidad U = X1X2

2 y se enfrenta a los precios p1=10 y p2=20, siendo su renta (M) igual a 180. Encuentre la elección óptima de este consumidor. 9) Determine el optimo del consumidor que tiene una función de utilidad U (x,y) = x1/3 y1/3, con un ingreso igual a $24, y donde el precio de x es $4 y el precio de y es $2. 10) Suponga que un estudiante gasta toda su renta en almuerzos y libros de microeconomía. Sus preferencias están representadas por la función: U = x3y. El precio de x es $5 y el de y es $50. La renta del estudiante es de $1000.

a) Encuentre la elección optima de ambos bienes b) Recalcule el punto anterior si ahora el precio de x sube a $6.

11) Un consumidor con preferencias U = X + 3Y tiene una renta de $200 para gastar en los bienes X e Y cuyos precios son $10 y $20 respectivamente. Encuentre la elección óptima de este consumidor. 12) Suponga que un individuo es indiferente entre consumir dos cajetillas de tabaco light (L) y una de tabaco común (C), es decir, su función de utilidad será U = L + 2C.

a) Si inicialmente la renta de este individuo es de $10, y los precios del tabaco común (pc) y del tabaco light (pl) son respectivamente $5 y $2 respectivamente, cuál será su elección óptima?

b) Cambiaría su respuesta anterior si ahora los precios fueran pL = $2,5 y pC = $5?

13) Pablo usa siempre tres pares de medias (M) con cada par de zapatillas (Z) cuando sale a correr. Si sólo le dan pares extras de medias no las usa, y si le dan pares adicionales de zapatillas tampoco las usa. Es decir, la función de utilidad de Pablo es U=min {M, 3Z}.

a) Si Pablo tiene $100 para gastar en su hobby de correr, el par de medias cuesta $10 y el par de zapatillas $20, que cantidad de ambos bienes demandará Pablo?

b) Si ahora ambos bienes costaran $20, cual sería su elección?

14) Considere el mercado de pizza ilustrado en la tabla adjunta. Suponga que el gobierno decide gravar con un impuesto indirecto las pizzas, siendo el mismo de $4 por pizza. (Krugman, 2006. Adaptado)

a) ¿Cuál es la cantidad de pizza comprada y vendida después de la aplicación del impuesto? ¿Cuál es el precio pagado por los consumidores? ¿Cuál es el precio que reciben los productores?

b) Calcule el excedente del consumidor y del productor después de la aplicación del impuesto. ¿En cuánto se reduce el excedente del consumidor? ¿Y el del productor?

c) ¿Cuál es el ingreso impositivo calculado por el gobierno? d) Calcule la pérdida irrecuperable de eficiencia del

impuesto. e) Suponga que ahora el gobierno decide, en vez de cobrar un

impuesto, imponer un precio mínimo sobre la pizza, de $8. ¿Cuál es la cantidad comprada y vendida de pizza luego de tal política?

f) Para el punto anterior calcule el excedente del consumidor y del productor. Compare con la situación de impuesto.

Unidad 4

34

15) Damián es un actor ocupado que asigna su tiempo libre entre ver películas y hacer ejercicio en el gimnasio. Cada película le lleva 2 horas de tiempo, y cada visita al gimnasio también 2 horas. En total dispone de 14 horas a la semana para dedicar a estas actividades.

Visitas al gimnasio por

semana

Utilidad Total

Cantidad de películas por

semana

Utilidad Total

1 100 1 60 2 180 2 110 3 240 3 150 4 280 4 180 5 310 5 190 6 330 6 195 7 340 7 197

a) ¿Cuál es su elección óptima? b) Si en vez de dos horas, cada película le insumiera 4 horas, ¿cual sería su elección optima? c) Separe el efecto anterior en efecto sustitución y renta (Slustky).

16) Sammy tiene una renta semanal de $60, el precio de las croquetas es de $5 por kilo, y el precio de las papas de $1 por kilo. Ambos son bienes normales para Sammy. Para cada una de las siguientes situaciones, construya un gráfico que muestre el efecto sustitución puro, así como el efecto sustitución y renta conjuntamente. Coloque la cantidad de croquetas (en kg) en el eje horizontal y la cantidad de papas (en kg) en el eje vertical. (Krugman, 2006)

a) El precio del kilo de croquetas cae de $5 a $2.50, y el precio del kilo de papas se mantiene en $1. b) El precio del kilo de croquetas aumenta de $5 a $10 y el precio del kilo de papas se mantiene en $1.

17) En el cuadro siguiente aparecen tres situaciones en las que puede encontrarse un individuo consumidor de dos bienes, X1 y X2. Para cada situación se tienen los precios de los dos bienes P1 y P2, las cantidades consumidas de cada uno, el ingreso nominal del consumidor (M) y su nivel de utilidad total.

Situación P1 P2 X1 X2 M U 1 1 1 70 20 90 10 2 1 0,50 48 84 90 15 3 1 0,50 40 70 75 10

a) Represente en un mismo gráfico las tres situaciones. b) ¿Cuál es el cambio en la cantidad demandada de X2 cuando el precio de X2 baja de 1 a 0,50,

manteniéndose constante el ingreso nominal en 90? c) ¿Qué parte del cambio mencionado en el punto anterior se debe al efecto sustitución y que

parte al efecto ingreso? d) ¿Es X2 un bien normal o inferior?

Guía de ejercicios Microeconomía- UADE

35

Estudio de Casos 1) Lea el siguiente artículo de la sección Economía de CLARIN, y luego responda las preguntas sugeridas al final del mismo Clarín.com » Edición Domingo 03.03.2002 » Economía »

El 86% de la gente cambió ya sus hábitos de consumo Natalia Muscatelli. DE LA REDACCION DE CLARÍN En los últimos dos meses, junto con el adiós a la paridad un peso-un dólar, el 86% de los consumidores cambió, en alguna medida, sus hábitos de compra. Según un estudio titulado "El consumidor posdevaluación" de la consultora ACNielsen, más de la mitad de una muestra tomada en Capital Federal y suburbios, confesó que redujo la cantidad de artículos que solía comprar para su hogar. Otros, optaron por consumir marcas más económicas, por abandonar algunos productos o por comprar artículos en envases más chicos o en menor cantidad. La tendencia a reducir el volumen de compra se encontró, en mayor medida, en los hogares más numerosos. El 70% de quienes viven en casas habitadas por cinco o más personas dijo comprar menos cantidad que los hogares de 1 y 2 personas (60%). Lo mismo reconoció un 63% de las familias, de 3 y 4 personas. Como era esperable, el consumo cayó más en los suburbios (67%) que en la Capital (59%). Las razones del achique tienen una explicación tan dramática como obvia. La pérdida de la capacidad de consumo de una familia tipo (4 personas) de Capital Federal fue del 14,3% desde diciembre hasta hoy, según datos de la consultora Equis. Ese impacto negativo en la capacidad de consumo es producto de la combinación de dos variables: "mientras a fines del año pasado, el ingreso promedio de esa familia era de 1.003 pesos, hoy pasó a ser de 932,8 pesos", explicó Artemio López, titular de esa consultora. Por otra parte, la canasta básica, estimada en diciembre, en 1.017 pesos hoy está valuada en 1.038 ", dijo. Por eso, "en medio de una situación económica cambiante y no favorable, el consumidor encuentra cada vez más limitaciones para optar y eso se refleja en sus hábitos de compra", sintetizó Martín Rosenzvit, responsable del estudio de ACNielsen. Esta actitud "ahorrativa" de los consumidores se observó más, según el sondeo, en los súper e hipermercados. En estas grandes superficies, un 79% de los consultados dijo comprar menos cantidad de productos, mientras que en los comercios de barrio, ese porcentaje fue del 51%. Al ser consultados, en qué productos los consumidores disminuyeron sus compras, el 63% dijo que el recorte fue más abrupto en los artículos de limpieza para el hogar. Un 59% dijo que en cosmética y tocador; un 56% en alimentos no básicos y un 55% en bebidas. El segmento que tuvo menos cambios fue el de alimentos básicos (52%), donde el restante 48% decidió seguir comprando normalmente sin hacer ningún cambio. En el rubro donde la gente experimentó más variaciones (limpieza y hogar) el 43% cambió por marcas más baratas; el 22% dejó de comprar algún producto y el 18% compró envases más chicos o "menos cantidad". En cuanto a las bebidas, es llamativo que el 33% (superior al 30% que cambió por marcas de menor precio) directamente dejó de comprarlas. Tal vez por considerar que, por ejemplo, una gaseosa de marca no puede ser reemplazada por otra de segunda marca o jugo. El precio, la variable clave de estos últimos meses, no escapó a la observación de los consumidores: 8 de cada 10 entrevistados estimó que los precios van a aumentar, como producto de la apreciación del dólar, en tanto una tercera parte de los encuestados opinó que el aumento será superior al 30%. Sin embargo, hubo una porción importante que demostró mucha incertidumbre en cuanto al valor del aumento. Respecto de las actuales medidas económicas, la opinión general de la gente fue de neutral a negativa, especialmente en Capital Federal. En este sentido, el 38% de los consultados se manifestó pesimista o muy pesimista, el 25% confesó mantener el optimismo hacia el futuro en tanto un 31%, seguramente "desconcertado" frente a la fluctuación de la economía, se confesó "neutral". Tal vez por la necesidad de la gente de creer en una mejora hacia el futuro, en función de los hijos, los niveles de optimismo crecen, según el trabajo de la consultora, a medida que las familias son más numerosas.

Unidad 4

36

a) Comente con sus compañeros de clase o con su familia la temática de éste artículo. Recuerde, que su capacidad de expresarse oralmente es altamente valorada en el mercado laboral y esto puede ser un training importante.

b) ¿Cómo cree Ud. que es la utilidad del “consumidor postdevaluación” comparada con su situación inicial? Muestre mediante un mapa de indiferencia dónde se ubica, “antes y después” de la devaluación.

c) De acuerdo al artículo, “el ingreso promedio (…) era de 1.003 pesos, hoy pasó a ser de 932,8 pesos”. Ilustre ésta situación con el herramental gráfico que conoce hasta el momento. ¿Cómo lo relaciona con el punto anterior?

d) Identifique la frase que deja en evidencia el efecto ingreso y el efecto sustitución. Realice un gráfico hipotético eligiendo dos bienes representativos. (Por ej. Bienes normales y Bienes de 2das marcas).

e) Considere la siguiente frase: “el 63% dijo que el recorte fue más abrupto en los artículos de limpieza para el hogar”. ¿Quiere esto decir que la compra de artículos de artículos de limpieza cayeron en un 63%? Explique la posible confusión.

f) ¿Cuáles son los bienes que más se dejaron de comprar? ¿Cómo los podría definir conceptualmente? g) ¿Recuerda Ud. haber abandonado o modificado algunos hábitos de consumo por reducción real o

nominal de su renta? Dé al menos dos ejemplos para cada caso. 2) Caso del subsidio a la vivienda Suponga que los consumidores están escogiendo entre los servicios de vivienda medidos en metros cuadrados y el consumo de otros artículos agregados y medidos en pesos.

a) Muestre el equilibrio en un gráfico. b) Suponga ahora que el gobierno acepta subsidiar a los consumidores pagándoles el 50% de su

costo de vivienda. ¿Cómo cambiaría su recta presupuestaria?. Muestre el nuevo equilibrio.

3) Caso de la caja de alimentos El gobierno quiere apoyar a los grupos menos favorecidos de la sociedad mediante la entrega de una caja conteniendo alimentos. Esa caja habitualmente contiene distintos tipos de alimentos entre los cuales cabe mencionar leche, aceite, arroz, polenta, lentejas, azúcar, yerba, té, etc. Por razones de simplicidad exclusivamente vamos a suponer que la caja solo contiene arroz. Se quiere analizar el comportamiento de una persona carenciada, digamos un jubilado cuyo ingreso mensual es de $ 300 que prácticamente no le alcanza para adquirir todos los elementos que necesita para su vida habitual. Este jubilado tiene la posibilidad de comprar arroz o cualquier otro bien que desee. Cuando acude al mercado encuentra que el arroz tiene un precio de $ 3. En los últimos meses, él ha acudido al mercado a comprar aproximadamente 1,5 kilos mensuales. Se le solicita que realice los siguientes pasos:

a) Marque la situación de equilibrio original del consumidor entre el bien arroz (que se mide en el eje horizontal) y los otros bienes (que se miden en el eje vertical). El precio de los otros bienes supóngalo en $ 1.

b) Si al jubilado se le entrega una caja con dos kilos de arroz, indique si cambian las curvas de indiferencia o la línea de precios. Dibuje el cambio.

c) Ahora defina Ud. cuanto sería la cantidad de arroz que el jubilado va a consumir y determine como se modifica el punto de equilibrio.

d) Cual sería la nueva situación de equilibrio del jubilado si se le concede un subsidio de $ 6 adicional a su jubilación, pero que se cobra en la municipalidad.

e) Si Ud. tuviera que decidir, y pudiera incorporar en su análisis otras consideraciones, que aconsejaría: entregar la caja de arroz o dinero en efectivo?

Guía de ejercicios Microeconomía- UADE

37

4) Caso del impuesto al cigarrillo. El gráfico adjunto muestra el mercado de cigarrillos. El precio de equilibrio actual es de $4 el paquete y cada día se venden 40 millones de paquetes. Con el fin de recuperar algunos de los costos en sanidad asociados al consumo de cigarrillos, el gobierno grava la compra de cigarrillos con un impuesto de $2 por paquete. Esto incrementa el precio de cada paquete a $5 y reduce la cantidad de equilibrio a 30 millones de paquetes. (Krugman, 2006). El economista que trabaja para la industria tabacalera afirma que este impuesto reducirá el excedente del consumidor de los fumadores en 40 millones de pesos diarios, ya que 40 millones de paquetes costarán $1 más. El economista que trabaja para la asociación de fumadores pasivos contra el tabaco argumenta que ese cálculo está sobrestimado y que la reducción en el excedente del consumidor es sólo de 30 millones de pesos diarios ya que después de aplicar el impuesto sólo venderán 30 millones de paquetes diarios, costando cada uno de ellos un peso más. Ambos están equivocados ¿por qué?.

Unidad 4

38

AUTOEVALUACIÓN. Puntaje obtenido:

1) Coloque Verdadero o Falso. Para éste último caso subraye la parte falsa para justificar: (3 puntos) a) El consumidor siempre preferirá disponer de más unidades de un bien (que menos) en un tramo o

sendero hasta un máximo. b) El principio de utilidad marginal decreciente enuncia que cada unidad adicional que se consume

de un bien o servicio añade más utilidad total que la unidad previa. c) Según el enfoque de la utilidad ordinal, sólo examinamos la ordenación de canastas basadas en

preferencias. Este es el enfoque que se utiliza para construir las curvas de indiferencia. d) El consumidor maximiza su utilidad asignando su gasto de manera tal que la utilidad marginal

por cada peso gastado sea la misma para todos los bienes. e) La Tasa Marginal de Sustitución (TMS) entre dos bienes muestra cuántas unidades de un bien

debe sacrificar el consumidor después de recibir una unidad adicional de otro bien para mantener constante su nivel de ingreso.

f) El efecto renta nos dice cuánto varía la cantidad consumida de un bien por la baja en el poder adquisitivo debido a la suba en el precio de ese bien. El efecto sustitución en cambio nos dice cuanto varía la cantidad consumida de un bien como resultado de que el consumidor reemplaza el bien que se volvió relativamente más caro, por uno relativamente más barato para mantener constante su nivel de utilidad.

2) En el diagrama de abajo se muestra una de las curvas de indiferencia de Joan Robinson así como su recta presupuestaria (2 puntos). a) Si el precio del bien A es de $60, ¿cuál es el ingreso de la señorita Robinson? b) ¿Cuál es la ecuación de su recta presupuestaria? c) ¿Cuál es el precio del bien B? d) ¿Cuál es su tasa marginal de sustitución? 3) Dadas las siguientes curvas de demanda P = 16 – 2Q y de oferta P = 1+ Q para un mercado hipotético. (2 puntos) a) Calcule el precio y la cantidad de equilibrio y grafique. b) Si el Estado impone un impuesto de $3 por cada unidad de producto vendido, calcule el nuevo equilibrio y represente en el gráfico anterior. c) ¿Cuánto aumentará lo que pagan los consumidores? d) ¿Cuál es la pérdida irrecuperable de eficiencia para la sociedad del impuesto de $3 por unidad? 4) Considere un individuo al que se le ofrece una cesta de consumo, con cantidades positivas de

ambos bienes, tal que 2

1

2

1

PP

UMUM

> . Entonces, dada su renta, para maximizar su satisfacción: (2

puntos) a) debería aumentar la cantidad del bien 1 y disminuir la cantidad del bien 2.

Guía de ejercicios Microeconomía- UADE

39

b) no habría que cambiar dicha cesta. c) debería aumentar la cantidad del bien 2 y disminuir la cantidad del bien 1. d) ninguna de las anteriores.

5) Suponga que María gasta toda su mensualidad en golosinas y gaseosas. El siguiente gráfico muestra sus curvas de indiferencia entre golosinas y botellas de gaseosa, y las rectas de presupuesto para distintos precios. El precio de los golosinas es de $2 y ella tiene $120 para gastar por semana. El precio de la gaseosa ha aumentado cada semana. La primera semana cada botella costaba $1, la segunda semana el precio subió a $2 y la tercera a $3 ¿Cuántas botellas de gaseosa ha comprado María cada semana? (1 punto)

a) Compró 80, 35 y 20 cada semana. b) Compró 20, 35 y 80 cada semana. c) Compró 80, 50 y 20 cada semana. d) Compró 20, 35 y 30 cada semana.

Golosinas

Botellas de Gaseosa

60

1206040

30

80 20 35

2520

50

10

Unidad 4

40

D

Respuestas Sugeridas

2) a) La nueva recta presupuestaria será I = (Px + 4).X + (Py – 2).Y b) La pendiente de la recta presupuestaria es el cociente de precios Px/Py. Inicialmente la pendiente es 120/80=1,5. Con los nuevos precios la pendiente es (120+18)/(80+12)=138/92=1,5, igual que antes. Por lo tanto el desplazamiento es paralelo. Ahora bien, como el ingreso del consumidor sigue siendo el mismo de antes, puede comprar menos de ambos bienes, por lo tanto la recta presupuestaria se encuentra por debajo de la anterior. Así es como la proposición del ejercicio resulta verdadera. 5) Dada la condición de equilibrio del consumidor: UMgx/Px = UMgy/Py. Reemplazando por los datos del ejercicio vemos que la condición de equilibrio no se cumple ya que: 80/0,3 > 15/0,6. La utilidad marginal por el último peso gastado en automóvil es mayor a la utilidad marginal por el último peso gastado en autobús. Por lo tanto, debe aumentar su uso del transporte de automóvil, y reducir el uso del transporte en autobús hasta llegar a la igualdad planteada. 8) La restricción presupuestaria es: 10X1 + 20X2 = 180 y la elección del consumidor se obtiene resolviendo:

TMSx1,x2 = - X2/2X1 = p1/p2 = - 1/2 X2 = X1 10X1 + 20X2 = 180

11) Las curvas de indiferencia son en este caso líneas rectas decrecientes de pendiente TMS = − UMgx/UMgy = -1/3 y los bienes son sustitutivos perfectos. La recta de balance es: 10x + 20y = 200, cuya pendiente es −px/py = −1/2, que en valor absoluto es mayor que la TMS. Por lo tanto, el consumidor se especializará en el consumo del bien relativamente más barato, en este caso el bien Y, consumiendo la combinación (0, M/py) = (0, 10) y obtendrá una utilidad de U(0, 10) = 30. Autoevaluación 1) a) V. b) F más →menos. c) V d) V e) F ingreso.→utilidad. f) V. 2) a) $1800 b) y = -1/2 x + 30 c) $30 c) ½ 3) a) (p*, q*) = (6, 5) b) (p*, q*) = (8, 4) c) $2 por unidad d) $1.5 por unidad 4) a 5) a

Volver al Índice.

X1* = X2* = 6

Guía de ejercicios Microeconomía- UADE

41

Unidad 5: Los mercados. Casos extremos: competencia perfecta y monopolio.

Preguntas de Repaso 1) Coloque Verdadero o Falso. Justifique en todos los casos.

a) En el corto plazo la condición de maximización de beneficios en competencia perfecta exige que el P = CMg. Si el P < CVMe la empresa obtendrá pérdidas y cerrará. Pero también existe la posibilidad que tenga ganancias extraordinarias, esto es cuando el P > CMeT. En el largo plazo en cambio, los beneficios económicos son siempre nulos.

b) La empresa competitiva ofrece una pequeña proporción de la producción total de todas las empresas de la industria. Por lo tanto considera dado el precio de mercado de producto y elige su nivel de producción suponiendo que su elección no influye en el precio. Esto se refleja mediante una curva de demanda horizontal para la empresa, mientras que la demanda de mercado tiene pendiente negativa.

c) La curva de oferta a corto plazo de la empresa competitiva es su curva de costo marginal situada por encima del punto de CVMe mínimo. En tanto, la curva de oferta de mercado a corto plazo es la suma de las cantidades ofrecidas por todas las empresas.

d) Una empresa competitiva produce hasta el punto en el cual el precio es igual al costo variable medio.

e) El punto de cierre de una empresa es aquel en el cual el precio es menor que el costo medio mínimo.

f) La curva de oferta de una empresa sólo depende de su costo marginal. Cualquier otro concepto de costo es irrelevante para las decisiones de oferta.

g) d)La regla P = CM de las industrias competitivas se cumple en el caso de las curvad de CM horizontales, de pendiente positiva y de pendiente negativa.

h) La empresa competitiva fija un precio igual al costo marginal. i) El excedente del productor y del consumidor, miden conjuntamente las ventajas que

tiene un mercado competitivo desde el punto de vista del bienestar. j) Un monopolista maximiza los beneficios cuando CM = P k) Cuanto mayor es la elasticidad-precio, más alto es el precio del monopolista con

respecto a su CM. l) Los monopolistas maximizan ventas. Por lo tanto, producen más que los competidores

perfectos, y su precio es más bajo. m) En el monopolio la elasticidad precio de la demanda es infinita. n) En el monopolio, la elasticidad precio de la demanda cruzada de los sustitutos más

cercanos es alta, esto implica que no tiene sustitutos cercanos. o) La curva de ingreso total de un competidor perfecto es proporcional a la cantidad

vendida. Para el monopolista en cambio, al principio aumenta, alcanza un máximo en el punto medio de la curva de demanda, donde la elasticidad es unitaria, y luego disminuye.

p) Cuanto más elástica sea la demanda con respecto al precio en un monopolio, más elevado será el precio en relación con el ingreso marginal.

q) En el monopolio, al no haber una correspondencia única entre el precio que cobra el monopolista y la cantidad que decide producir, se puede decir que no tiene ninguna curva de oferta, sino más bien una regla de oferta que consiste en igualar el Img al Cmg.

r) Los beneficios del monopolio subsisten, aún en el largo plazo. s) El equilibrio competitivo agota las ganancias que pueden derivarse del intercambio,

siempre que no estemos en presencia de externalidades o bienes públicos. En cambio, en el equilibrio de monopolio el valor que tiene para la sociedad una unidad más de producción es superior al costo que tienen para el monopolista los recursos necesarios para producirla.

t) Estamos en presencia de un monopolio natural cuando la curva de CmeL tiene pendiente negativa.

u) Un monopolista que discrimina precios no está interesado en vender a algunos clientes a los que sí vendo el monopolista no discriminador.

Unidad 7

42

v) Un monopolista que discrimina precios introduce una ineficiencia mayor que la que provoca el monopolista no discriminador, puesto que captura una parte mayor del excedente del consumidor.

w) Bajo discriminación de precios, un consumidor que tenga una elasticidad precio de la demanda relativamente alta pagará un precio más bajo que un consumidor que tenga una demanda inelástica.

2) Complete la siguiente oración: “En competencia perfecta el punto de cierre en el corto plazo es aquel donde _____________es menor que _______________. En tales circunstancias, el empresario decide cerrar porque si produce pierde más que sus _________________ y si cierra sólo pierde éstos. En el caso del monopolista, siempre que el ____________________, es decir el valor del precio a lo largo de la curva de demanda, sea menor que el ________________________ en todos los niveles de producción, lo mejor que puede hacer es dejar de producir a corto plazo.” 3) Considere los siguientes casos y explique a qué fuentes de monopolio pueden atribuirse:

a) El servicio de subtes en la ciudad de Buenos Aires. b) El servicio de telefonía celular c) El uso del sistema operativo Windows d) La venta de gaseosas de la línea Coca Cola solamente, dentro del edificio universitario e) La venta de anillos de diamantes de DeBeers Diamond Mines f) La comercialización de un analgésico con el nombre de “Bayaspirina”.

4) ¿En cuales de las siguientes situaciones hay discriminación de precios y en cuales no? En los casos que exista discriminación de precios, identifique los consumidores que tienen una elasticidad precio de la demanda alta y los que tienen una elasticidad precio de la demanda baja (Krugman, 2006)

a) Se venden productos con algún defecto a un precio menor. b) Los restaurantes ofrecen descuentos a los jubilados c) Los productores de alimentos utilizan periódicos y las revistas para regalar cupones de

descuento para sus productos. d) El precio del billete de avión es mayor en la temporada alta del verano.

Ejercicios 1) Dada la siguiente tabla, complete con los datos faltantes y determine el nivel de producción que maximiza el beneficio de la empresa competitiva en el corto plazo.

Q por periodo Precio Ingreso

Total Ingreso Marginal

Costo Fijo

Total

Costo Variable

Total

Costo Total

Costo Marginal

Beneficio Total

0 10 12 0 1 10 2 2 10 3 3 10 5 4 10 8 5 10 13 6 10 23 7 10 38 8 10 69

a) Muestre el nivel óptimo en un gráfico que relacione el Ingreso Total con el Costo Total. b) Muestre el nivel óptimo en un gráfico que relacione el Ingreso Marginal con Costo Marginal y el

Costo Variable Medio. c) ¿Qué ocurre con la elección del nivel de producción de la empresa y los beneficios si el precio

del producto baja de $10 a $8?

Guía de ejercicios Microeconomía- UADE

43

d) ¿Qué ocurre con la elección del nivel de producción de la empresa y los beneficios si el Costo Fijo de producción aumenta de $12 a $15 y a continuación a $25? ¿Qué conclusiones generales puede extraer sobre la influencia de los costos fijos en la elección del nivel de producción de la empresa?

2) Si el costo total de una empresa a diversos niveles de producto lo dan los valores de la tabla siguiente, y el ingreso total es IT = 4.Q :

Q 0 100 200 300 400 500 600 700 750 800 900 CT 400 1000 1300 1500 1600 1700 1850 2100 2265 2500 3600 IT 0 400 800 B

a) Determinar el nivel de producto al cual:

i) las pérdidas totales son máximas, ii) la empresa ni pierde ni gana, iii) la empresa maximiza las ganancias totales

b) Graficar las proyecciones de ingreso total y costo total en un mismo sistema de ejes. Sobre la curva de costo total marcar el punto de pérdidas totales máximas y el punto de maximización de beneficios totales.

c) ¿En qué punto está la empresa en equilibrio de corto plazo y por qué? 3) Suponga que usted debe asesorar a Just Fontaine, un empresario que opera en un mercado de competencia perfecta, acerca del volumen de ventas en el cual se maximizan beneficios en el corto plazo. El empresario le acerca la siguiente información obtenida en base a su nivel de ventas:

Ingreso total.......................$ 1000 Cantidad vendida...............$ 100 unid. Costo total..........................$ 1200 Costo medio fijo.................$ 1 Costo marginal...................$ 10,5

¿Qué le aconsejaría al productor? 4) Suponga que la función de costos de una empresa viene dada por C(Q) = 100 + Q2. Si la empresa vende su producción en un mercado de competencia perfecta, y las demás empresas de la industria venden su producto a un precio de 10 dólares, ¿qué nivel de producción debería fabricar la empresa para maximizar sus beneficios o minimizar sus pérdidas? ¿Cuál será el nivel de beneficios o pérdidas si la empresa toma la decisión óptima? (Baye, 2006) 5) Si las curvas de costo marginal y de costo variable medio a corto plazo de una empresa competitiva son Cmg = 2Q y CVMe = Q, ¿cuántas unidades producirá la empresa al precio de mercado P = 12? ¿Con qué nivel de coste fijo obtendrá unos beneficios económicos nulos? (Frank, 2004) 6) El siguiente gráfico corresponde a una firma que opera en condiciones perfectamente competitivas:

Unidad 7

44

R

R

D

a) Identifique el punto de cierre en el corto plazo (cantidad y precio). b) Señale por cuáles puntos pasaría la curva de oferta de la empresa. c) Si el precio es de ¢80 por unidad (donde ¢ es igual a $), determine:

i) Nivel de producción que maximiza las ganancias ii) Ingresos totales iii) Costos totales iv) Costos variables totales v) Costos fijos totales vi) Ganancias o pérdidas económicas totales

d) ¿A cuál precio la empresa sólo obtiene ganancias normales? e) Si en ese mercado todas las empresas tienen las mismas curvas de costos y no se presenta

ningún cambio en la demanda ni en la tecnología ni en los costos de producción, ¿a cuál precio se dará el equilibrio de largo plazo?

f) Si el precio en un momento dado fuera ¢60 por unidad, ¿qué tendería a ocurrir en el largo plazo con el precio, el número de empresas en la industria y las ganancias económicas?

g) Si el precio en un momento dado fuera ¢45 por unidad y se da un alza sostenida de la demanda, ¿qué tendería a ocurrir en el largo plazo con el precio, el número de empresas en la industria y las ganancias económicas?

h) Si el precio en un momento dado fuera ¢45 por unidad y se da un alza sostenida en los costos de producción, ¿qué tendería a ocurrir en el largo plazo con el precio, el número de empresas en la industria y las ganancias económicas?

7) La empresa “Weird. S.A.”, vende 250 unidades de un producto. El costo de producción de la unidad número 250 es de $10. El precio de venta de los productos en el mercado es de $10 por unidad. Teniendo en cuenta que es un mercado competitivo, averiguar en qué situación se encuentra la empresa en los siguientes casos: a) CMe es de $8 por unidad b) CMe es de $10 por unidad c) CVMe es de $10 por unidad y el CMe es de $12 d) CMe es de $13 y el CVMe es de $11. 8) En un mercado de competencia perfecta intervienen 100 empresas que presentan la misma estructura de costos CT = 0,125 q2 + 2q + 25. La demanda total del mercado responde a la fórmula Q = 3700 – 100 P. Se le solicita:

a) Obtenga la función de oferta del empresario individual, la oferta del mercado y halle la cantidad y el precio de equilibrio en el mercado.

b) Dado que resulta necesario aumentar la recaudación el gobierno decide imponer un nuevo tributo de $ 3,75 por unidad vendida. Calcule la nueva función de oferta del mercado y halle la cantidad y el precio de equilibrio del mercado.

c) Grafique el equilibrio en el mercado de competencia perfecta marcando especialmente el precio inicial, el precio al productor y al consumidor y el monto del impuesto.

9) Considerando la siguiente función de costos a corto plazo de una empresa que interviene en un mercado de competencia perfecta: CT = 0,1 Q2 + 2 Q +160

a) Determine la función de oferta individual de cada empresa y grafique. b) Si hay 100 empresas en el mercado y la demanda total es: Qd = 6800 – 150 P, determine:

El precio y cantidad de equilibrio de mercado. Producción y beneficio de cada empresa.

c) Si el gobierno fija un impuesto de $26 por unidad producida, determine: La proporción del impuesto en que se harán cargo los consumidores y los

productores. ¿Cómo cambia la situación de las empresas en cuanto a cantidad producida y

beneficios individuales? ¿Cuál es la recaudación del gobierno?

10) En un mercado de competencia perfecta intervienen 100 empresas que poseen la misma estructura de costos CT = 0,125 q2 + 2q + 25. La demanda total del mercado responde a la fórmula Q = 3700 – 100 P. En base a estos datos se le solicita:

a) Obtener precios y cantidades de equilibrio del mercado.

Guía de ejercicios Microeconomía- UADE

45

D

b) El gobierno por motivos sociales decide imponer al bien un precio máximo al nivel P = 7. Calcule monto del desabastecimiento.

c) Explique que otro tipo de política podría aplicar el gobierno si desea llevar el precio del bien a P = 7 y calcule cual sería el valor óptimo por unidad que debe tomar ese instrumento de política.

d) Grafique los resultados de los puntos a, b y c en un mismo gráfico detallando equilibrios antes y después de la intervención del gobierno. Determine Precios al Productor, Precios al Consumidor y monto de la política tributaria cuando corresponda.

11) Considerando la siguiente función de costos a largo plazo de una empresa CT = 3Q3 – 24Q2 + 80Q que interviene en un mercado de competencia perfecta donde la demanda de mercado es Q = 1000 – 5P:

a) Determine el precio y la cantidad que producirá la empresa en el equilibrio de largo plazo. b) Suponiendo que todas las empresas tienen la misma función de costos de largo plazo, determine la cantidad de empresas que habrá en este mercado.

12) Una empresa perfectamente competitiva tiene la siguiente función de costos de largo plazo: CTLP = 2q3 – 100 q2 + 1260 q Siendo la función de demanda del mercado: q = 2000 – 100p:

a) Encuentre precio y cantidad de equilibrio para la empresa individual a largo plazo b) Encuentre precio y cantidad de equilibrio del mercado c) Número de empresas que lo forman

13) Un mercado de competencia perfecta con libertad de entrada y salida, tiene como función de demanda: X= 500-P. La función de costos totales de cada empresa que opera en este mercado es: C(Xi) = X3

i − 20X2i + 120Xi. Encuentre la cantidad intercambiada, el precio y el número de empresas

del equilibrio de mercado a largo plazo. 14) HJK es una empresa que produce trigo para el mercado mundial, por lo tanto es precio aceptante (se maneja como si fuera un competidor perfecto).

a) Si la función de costos del empresario es: CTcp = ½ Q3 – 3Q2 + 9Q + 5, encuentre la función de oferta de esta empresa. No olvide indicar a partir de que precio y cantidad es válida.

b) Si el precio fuera $20, cual es la cantidad que debería vender? Para la cantidad encontrada, ¿cuál será la situación de la empresa? (gana, pierde, etc).

15) Suponga que una industria tiene 30 empresas, cada una de las cuales tiene la curva de oferta P = 20 + 90 Qi ¿Cuál es la curva de oferta de la industria? (Frank, 2004) 16) Una fibra vegetal se comercia en un mercado mundial competitivo y el precio mundial es de $9 /Kg. A ese precio nuestro país puede importar una cantidad ilimitada. El cuadro adjunto muestra la oferta y la demanda internas de Argentina en millones de kgs correspondientes a distintos niveles de precios: (Pindyck, 2000)

Precio Oferta Demanda

3 2 34

6 4 28

9 6 22

12 8 16

15 10 10

18 12 4

a) ¿Cuál es la ecuación de la demanda y cuál de la oferta? b) ¿Cuál es la elasticidad precio de la demanda a un precio de $9? ¿Y de $12? c) ¿Cuál es la elasticidad precio de la oferta a un precio de $9? ¿Y de $12? d) Si no hay aranceles, contingentes u otras restricciones del comercio de Argentina ¿Qué

ocurriría con el precio y el nivel de importaciones de fibra por parte de nuestro país?

Unidad 7

46

e) Si Argentina impone un arancel de $9/Kg. ¿cuáles serán el precio y el nivel de importaciones? ¿Cuántos ingresos obtendrá el Estado con el arancel? ¿Cuál es la pérdida irrecuperable de eficiencia?

f) Si Argentina no tiene ningún arancel, pero establece un contingente sobre las importaciones de 8 millones de kgs. ¿cuál será el precio interno? ¿Cuál es el costo de ese contingente para los consumidores americanos de fibra? ¿Cuál es la ganancia de los productores americanos?

17) Halle la curva de ingreso marginal que corresponde a la curva de demanda P=12 -3Q y grafique. (Frank, 2004) 18) La curva de demanda para un monopolio hipotético es Q = -0.2 P + 80.

a) Calcule el Ingreso total y el ingreso marginal (puede ayudarse de una tabla de valores). b) Grafique la curva de demanda y de ingreso marginal. En un gráfico inferior ilustre el Ingreso total. c) ¿En qué valor de la producción el monopolista maximiza su ingreso total? ¿Cómo se relacionan las curvas en ese punto?

19) Halle el precio y la cantidad óptima del monopolista descrito por la información del cuadro adjunto. ¿Le conviene al empresario seguir en el mercado? (Frank, 2004)

Q P IM CMC CVMe 0 100 100 150 150 15 86 71 71 107 25 75 50 41 84 34 66 33 33 72 50 50 0 63 63

20) Un monopolista tiene la siguiente función de costos totales: CT = 5Q2 + 20Q + 200 y se enfrenta a la curva de demanda del mercado: Q = 150 – 0.5 P

a) Determine la cantidad, el precio de equilibrio y el beneficio económico, sabiendo que el monopolista maximiza beneficios.

b) Grafique las curvas de CMg, CMe, IMe e IMg, indicando el beneficio económico que recibe la empresa.

c) Demuestre que el monopolista opera en este mercado en el tramo elástico de la curva de demanda (es decir, donde la elasticidad-precio de la demanda es mayor que 1 y el ingreso marginal es positivo).

d) Calcule el grado de poder de monopolio de Lerner. e) El Estado decide cobrar un impuesto de $1.000 al monopolista. Determine analíticamente la

cantidad, precio, beneficio económico y recaudación tributaria en este caso. f) Suponga que en lugar del caso planteado en e), El Estado decide cobrar un impuesto de t=20

por unidad producida al monopolista. Determine analíticamente la cantidad, precio, beneficio económico y recaudación tributaria en este caso. Grafique. ¿Qué diferencia existen con respecto al caso anterior?

21) Suponga que la función de demanda inversa del producto de un monopolista viene dada por P = 10 - 2Q. ¿Cuál es el precio máximo por unidad que puede cobrar un monopolista para poder vender 3 unidades? ¿Cuál es el ingreso marginal de 3 unidades de producto? (Baye, 2006) 22) Un monopolista tiene la curva de demanda que viene dada por P = 100 – Q y la curva de costo total que viene dada por CT = 16 + Q2. La curva de Costo Marginal correspondiente es CMg = 2Q. (Frank, 2004. Adaptado)

a) Halle la cantidad y el precio que maximizan el beneficio del monopolista. ¿Qué beneficio económico obtendrá este? b) Suponga que los costos se modifican a CT = 32 + Q2. Halle la cantidad y el precio que maximizan el beneficio del monopolista. ¿Qué beneficio económico obtendrá este? c) Suponga ahora que los costos se modifican a CT = 16 + 4Q2. La curva de costo marginal ahora es CM = 8Q. Halle la cantidad y el precio que maximizan el beneficio del monopolista. ¿Qué beneficio económico obtendrá este?

Guía de ejercicios Microeconomía- UADE

47

R

D

e) Suponga que el monopolista tiene una curva de Costo Marginal a largo plazo de CM = 20. Halle la cantidad y el precio que maximizan el beneficio del monopolista. Halle la pérdida de eficiencia provocada por este monopolio. f) Grafique cada caso.

23) La demanda del producto q está dada por q = 250- p/2. El bien es producido por una empresa cuya función de costo total es CT = 200 + 20q + 5q². Determine el precio y la cantidad de equilibrio: a) En situación de competencia perfecta. a) Actuando como monopolio maximizador de beneficios. b) Demuestre que el monopolista opera en este mercado en el tramo elástico de la curva de

demanda (es decir, donde la elasticidad-precio de la demanda, en valor módulo, es mayor que 1, y el ingreso marginal es positivo).

24) Dados dos mercados aislados abastecidos por un solo monopolista y siendo las funciones de demanda de ambos mercados las siguientes:

P1 = 12 - Q1 P2= 20 - 3Q2 conociendo que la función de costos totales del monopolista es CT = 3 + 2 (Q1 + Q2), se le pide:

a) halle los precios, las cantidades y los beneficios totales que percibe el monopolista en cada uno de los mercados si efectúa discriminación de mercados. Halle las respectivas elasticidades precio de demanda en cada uno de los dos puntos de equilibrio.

b) Halle el precio, la cantidad y el beneficio total si no efectúa discriminación de mercados. Compare ambas situaciones.

c) Obtenga la situación de equilibrio si se pudiera hacer discriminación perfecta de precios. Determine precio, cantidad y beneficio total. Compare con las dos situaciones anteriores.

25) Un monopolista tiene como función de costes totales a largo plazo C(x) = x2/2 + x. Existen dos mercados distintos en los que puede vender su producción, cuyas funciones de demanda son x1 = 20 − P y x2 = 30 − P. ¿Cuáles serán las cantidades y los precios a los que el monopolista venderá si separa ambos mercados? ¿Y si no puede discriminar precios? 26) Un monopolista enfrenta dos mercados perfectamente separables: el externo y el nacional. Las demandas en estos mercados son: DE = 240 – 2PE y DN = 100 – PN Su función de Costo Marginal es CMg = 40 + ½Q. Grafique y compruebe si le conviene o no realizar una discriminación de precios. 27) Un monopolista tiene como función de costes totales a corto plazo C(x) = 10x+200. Existen dos mercados diferentes a los cuales puede vender su producción, con funciones de demanda respectivas de x1=40 − 2P y x2 = 25 − P. Encuentre los precios y cantidades que maximizan su beneficio, con y sin discriminación de precios. Estudio de Casos 1) El mercado de viviendas en alquiler (Mansfield, 1998): Según estudios recientes, la elasticidad precio de la demanda de viviendas en alquiler en ciudades estadounidenses es 1.0 y la elasticidad precio de la oferta de viviendas en alquiler es de 0.5 a largo plazo.

a) Suponga que el gobierno de una ciudad particular decide que el nivel de alquiler debe ser aumentado para fomentar una mayor oferta de viviendas en alquiler. Si el gobierno desea incrementar la cantidad de viviendas en alquiler ofrecidas en un 2%, ¿qué incremento se requiere en el alquiler?

b) ¿Tal incremento en el alquiler conducirá al incremento deseado en la cantidad de viviendas ofrecidas en el corto plazo?

c) ¿Cuál será el efecto sobre la cantidad total de viviendas demandadas?

Unidad 7

48

d) Suponga que cuando el gobierno decide incrementar los alquileres, la oposición política es generalizada y que por eso se decide disminuirlos. Si hace que los alquileres disminuyan hasta el uno por ciento debajo del valor de equilibrio, la cantidad de viviendas demandadas excederá la cantidad ofrecida. ¿A cuánto ascenderá la diferencia entre la cantidad demandada y la cantidad ofrecida como porcentaje de la cantidad de equilibrio de viviendas en alquiler?

2) Lea el siguiente artículo de la agencia de noticias Reuters, y luego responda las preguntas sugeridas al final del mismo: Miércoles, 6 de Agosto de 2003, 9h36 Fuente: Reuters

Unión Europea define medidas contra abusos de Microsoft

BRUSELAS - La Comisión Europea definió el miércoles por primera vez las medidas que pretende imponer al gigante estadounidense del software estadounidense Microsoft por lo que calificó como abusos persistentes de su posición dominante en el mercado.

El órgano ejecutivo de la UE dijo que estaba dando a Microsoft una última oportunidad para presentar argumentos antes de ordenar al líder mundial del software que comparta información crucial de sus códigos internos con sus competidores en el mercado de los servidores de red y requerir una competencia más justa entre los programas reproductores de medios.

"La conclusión preliminar de la Comisión es que los abusos de Microsoft siguen produciéndose", dijo la Comisión Europea en un comunicado.

Señaló que Microsoft estaba utilizando su posición dominante en los populares servidores de bajo rendimiento, que proporcionan servicios claves a las computadoras de las redes corporativas.

Sobre los llamados "media players", programas utilizados para reproducir sonidos e imágenes en las computadoras, la Comisión dijo que Microsoft tenía dos alternativas para facilitar la competencia.

Esas medidas son: desligar el programa Windows Media Player de su omnipresente sistema operativo Windows, ofreciendo una versión de Windows sin el Media Player, o incluir reproductores de la competencia en el sistema Windows.

"Ambas soluciones tratan de garantizar que los consumidores tengan una alternativa justa en lo que respecta a los reproductores de medios", dijo la Comisión

El comisionado europeo de competencia, Mario Monti, dijo que la Comisión había enviado a Microsoft una última "declaración de objeciones, que incluye la identificación de remedios apropiados" dándole una oportunidad final para que se manifieste antes de que concluya la investigación antimonopolio de tres años.

La declaración da a la compañía otra oportunidad para solicitar una audiencia oral o replicar por escrito.

"Estados decididos a asegurar que la salida final de este caso sea para beneficio de la innovación y en beneficio de los consumidores", agregó Monti en un comunicado.

a) ¿Qué plantea el artículo con respecto a Microsoft? ¿Qué tipo de mercado cree Ud. que es el de los

sistemas operativos? ¿Cómo afecta a los consumidores? Discuta con sus compañeros. b) Enumere las condiciones que definen un mercado competitivo y diga si están presentes en el

caso tratado. c) ¿Cuáles de las siguientes fuentes de monopolio están presentes en el caso tratado? De al menos

un ejemplo: • Control exclusivo de factores importantes. • Economías de escala • Patentes • Economías de red • Licencia o concesión del Estado.

Guía de ejercicios Microeconomía- UADE

49

3) Caso: Estados Unidos contra IBM (Mansfield, 1998): Uno de los casos antitrust más largos y caros den la historia comenzó en 1969, cuando el gobierno hizo la acusación que IBM “ha procurado monopolizar y ha monopolizado (…) el comercio interestatal de computadoras de fines generales violando la sección 2 de la ley Sherman”. Según los economistas de IBM, su participación en el ingreso por las ventas de procesamiento electrónico de datos en los Estados Unidos fue la siguiente:

1952 90.1%

1961 56.4%

1968 54.0%

1972 40.7%

a) Basándose en estas cifras ¿Era IBM un monopolio? b) Aunque IBM no tuviera el 100% del mercado, ¿podría haber tropezado con las leyes antitrust? c) Si una empresa tiene una participación muy grande en el mercado ¿significa esto que debe

ser juzgada bajo las leyes antitrust? d) ¿Cómo debería uno definir en ese caso un mercado? e) Mientras IBM argumentó a favor de una definición amplia de la industria de la computadora

(incluyendo control de procesos de fines especiales, distribución de mensajes, computadoras militares, alquiler de computadoras y actividades de servicios), el gobierno argumentó a favor de una definición más estrecha (sistemas de computación digital electrónicos para fines generales) ¿por qué?

4) La economía del correo (Mansfield, 1998) Los servicios postales, que desde 1845 han constituido en general un monopolio gubernamental en los Estados Unidos, han sido objeto de controversias continuas. Suponga que las curvas de demanda y costo de corto plazo de la oficina de correos de Filadelfia son las siguientes:

a) ¿Los servicios postales son un monopolio natural, como algunos aducen? b) Si los servicios postales son un monopolio natural, ¿deben funcionar como propiedad

gubernamental? c) Si la oficina de correos de Filadelfia desea repartir todas las unidades de correspondencia

que pueda sin incurrir en un déficit de corto plazo, ¿cuántas debe repartir por día? d) Los datos disponibles indican que el ingreso medio (por unidad de correspondencia) ha

excedido el costo total medio y el costo marginal para la correspondencia de primera clase, pero no para la correspondencia de tercera clase. ¿Qué tipo de correspondencia atraiga a los competidores privados?

e) ¿Qué ventajas podría acarrear que los servicios postales enfrentaran una mayor competencia privada?

Ingreso marginal Costo marginal

6 4 2

Centavos por unidad de correspondencia

0

Número de unidades de correspondencia repartidas por día (millones)

Demanda

Costo total medio

Unidad 7

50

5) Suponga que Usted es el gerente general de una importante empresa que opera en un mercado bajo competencia perfecta. En la última reunión de directorio, los dueños le manifestaron su preocupación por los bajos ingresos que obtuvo la empresa. Además, concluyeron que el problema se debe al bajo precio que tiene su producto y le pidieron desarrollar un plan estratégico que apunte a subir el precio. Piense Usted cómo podría alcanzar el objetivo planteado por los dueños y escriba su propuesta para presentarla en la próxima reunión de directorio.

Guía de ejercicios Microeconomía- UADE

51

AUTOEVALUACIÓN. Puntaje obtenido:

1) Suponga que la función de demanda inversa del producto de un monopolista viene dada por P = 100 - 2Q y la función de costos por C(Q) = 10 + 2Q (2 puntos).

a) Calcule la curva de ingreso marginal. b) Calcule el precio y la cantidad que maximizan los beneficios. c) Calcule los beneficios máximos. d) Grafique a mano alzada

2) Suponga que la función de costos de una empresa viene dada por C(Q) = 80 + Q2. Si la empresa vende su producción en un mercado de competencia perfecta, y las demás empresas de la industria venden su producto a un precio de 10 dólares. (2 puntos).

a) ¿Estamos hablando de corto o largo plazo? Justifique. b) ¿Qué nivel de producción debería fabricar la empresa para maximizar sus beneficios o minimizar sus pérdidas? c) ¿Cuál será el nivel de beneficios o pérdidas si la empresa toma la decisión óptima? d) ¿Debe permanecer en el mercado?

3) Evalúe las siguientes situaciones y coloque Verdadero, Falso o ambos. Justifique su respuesta de manera clara y concisa (5 puntos).

a) En un mercado de competencia perfecta como el del trigo, o los alquileres, no hay beneficios. b) Los servicios de gas natural en red, electricidad y agua son monopolios naturales cuando pertenecen al Estado y éste los regula. c) Si el cine es más barato de lunes a miércoles, y mucho más caro los sábados y domingos que el jueves o el viernes, podemos decir que hay diferenciación de precios. d) En la competencia perfecta se realizan todos los intercambios mutuamente beneficiosos por lo tanto se maximiza tanto el excedente del consumidor como el del productor. e) Una banda de rock de Nueva Zelanda que ofrece un show en vivo por única vez en la Argentina a un precio demasiado elevado para el bolsillo de cualquier consumidor de ingresos medios es acusada de monopolio. La banda argumenta no ser un monopolio porque hay muchos sustitutos cercanos y tiene razón (Utilice su capacidad de imaginación para solucionar este dilema)

4) Señale, mediante la redacción de no más de 4 párrafos, las principales diferencias entre la competencia perfecta y el monopolio (1 punto)

Unidad 7

52

R

D

0

5

10

15

20

25

0 20 40 60 80 100Cantidad

Q

CVM

e, C

Mg

CMg = Curva de oferta

CVMe

Respuestas Sugeridas

8) a) qi = 4 Px – 8; Sm = 400 Px – 800; Dm = Sm = 2800; Px = 9 b) qi = 4Px – 23; Sm = 400Px – 2300; Dm = Sm = 2500; Px = 12 c) Pprod = 8,25; Pcons = 12; Monto del impuesto = 2500x3,75 = 9375

10) a) qi = 4 Px – 8 Sm = 400 Px – 800 Dm = Sm = 2800 Px = 9

b) Cantidad ofertada=2000; Cantidad demandada=3000; Desabastecimiento=1000 c) Debería aplicar un impuesto específico de 2,5 pesos por unidad vendida. d) Precio al consumidor = 7 ; Precio al Productor = 9,5 Monto de subsidio = 7500

25) Con discriminación de precios (separación de mercados) x1=3,5 ; p1 = 16,5

x2=8,5 ; p2 = 21,5 Sin discriminación de precios x=12 ; p=19

9) a) (1) CMg = 0,2 Q + 2 ; (2) CVMe = 0,1 Q + 2

En este caso, son dos funciones lineales con igual ordenada al origen, pero donde la pendiente de la curva de CMg es del doble que la de la CVMe, lo cual indica que para Q>0 se cumpla CMg>CVMe. Por lo tanto la curva de oferta de la empresa es igual a la de CMg para 0≥Q . Recordando que en competencia perfecta P = CMg y despejando P de (1):

Curva de oferta de la empresa qs = -10 + 5P (3) b) Sabiendo que hay 100 empresas en el mercado, por lo que la oferta en el mercado estará dada por la producción de dichas empresas, y a partir de (3):

Qs = 100 . qs Qs = 500P – 1000 (4) El equilibrio de mercado se determina haciendo Qs = Qd (la demanda de mercado es dato del ejercicio): Qs = Qd 500P – 1000 = 6800 – 150P P = 12 Y reemplazando en ambas funciones: Q = 5.000 Entonces P = 12 y Q = 5.000 constituye el equilibrio en el mercado. La producción de cada empresa es (recordando que hay 100 empresas idénticas en el mercado): qs = Qs/100 = 5000/100 qs = 50 Y el beneficio individual de cada empresa es: )(*. qCTqPEB −= , o sea:

⇒−= )50(50*12. CTEB 90. =EB (Hacer las cuentas y verificar) c)La proporción del impuesto que pagan los consumidores está dada por:

ds

s

eee−

donde la es es la elasticidad de la oferta y ed de la demanda (ambas elasticidades

precio). Entonces:

2,1000.512*500 ==se ; 36,0

000.512*150 −=−=de ; 77,0

)36,0(2,12,1

=−−

=− ds

s

eee

Guía de ejercicios Microeconomía- UADE

53

Esto significa que el 77% (aproximadamente) del impuesto es trasladado a los consumidores, y dado que el impuesto es de t = 26 por unidad producida, $20 será pagado por los consumidores (77% de 26) y 6 por los productores (23% de 26). Esto también se puede observar si planteamos el problema algebraicamente, recordando las ecuaciones de oferta y demanda en el mercado, y llamando pv al precio percibido por los vendedores y pc al que pagan efectivamente los compradores:

-1000 + 500Pv = 6800 – 150 Pc (5) Pc – Pv= 26 (6)

Despejando (6) y reemplazando en (5):

-1000 + 500Pv = 6800 – 150 (26 + Pv) Si operamos algebraicamente llegamos a que: Pv = 6 y por lo tanto Pc = 32 Notemos que Pv = P – 6 y Pc = P + 20, siendo P = 12 (el precio que equilibra el mercado sin impuestos) y a su vez 6 y 20 son la parte del impuesto que soportan el vendedor y el comprador respectivamente (y que coinciden con lo que encontramos cuando calculamos la proporción del impuesto que enfrentan cada uno, al inicio de este ejercicio). La nueva cantidad de equilibrio se obtiene reemplazando Pv en la función de oferta de mercado o Pc en la ecuación de la demanda: Q = 2000 En lo que respecta a cómo cambia la situación para cada empresa de forma individual, y dado que hay 100 empresas en el mercado, tenemos que:

qs = Qs/100 = 2000/100 qs = 20 y ahora ⇒−= )20(20*6. CTEB 120. −=EB (Hacer las cuentas y verificar) Vemos que, al utilizar para calcular el ingreso el precio efectivamente percibido por el vendedor, descontando la proporción del impuesto que debe hacer frente, no es necesario adicionar 26*Q en la función de costo total (o sino estaríamos pagando dos veces el impuesto!!!). Por último, calculamos la Recaudación del Gobierno, o sea el impuesto por unidad de producto multiplicado por la producción de equilibrio del mercado:

Recaudación = 26*2.000 = 52.000 13) El equilibrio a largo plazo con libre entrada de empresas se alcanza cuando el precio es igual al mínimo coste medio de largo plazo (CMeL) de las empresas que operan en el mercado. Cálculo del mínimo CMeL para la empresa representativa:

CMeL(Xi) = C(Xi)/Xi = X2 i − 20Xi + 120

CMeL´(Xi) = ∂CMeL(Xi)/∂Xi = 2Xi − 20 CMeL´(Xi) = 0 2Xi − 20 = 0 Xi = 10

Min CMeL(Xi) = CMeL(Xi = 10) = 20 Luego el mínimo CMeL es 20, que se alcanza para el nivel de producción individual Xi=10, y el precio de equilibrio del mercado es P = 20. La demanda agregada será: Xmdo= 500 − P = 480 y el número de empresas que actúan en el mercado es:

N = Xmdo/Xi = 480/10 = 48 27) El equilibrio del monopolista discriminador de precios de tercer grado se produce donde: IMg1 = IMg2 = CMg En este caso:

CMg = 10; IMg1 = 20 − x1; IMg2 = 25 − 2x2 Entonces,

IMg1 = CMg x1 = 10; p1 = 15 IMg2 = CMg x2 = 7,5; p2 = 17,5

Si el monopolista no puede discriminar precios, la curva de demanda a la que se enfrenta vendría dada por la agregación en cantidades de la demanda de cada uno de los grupos de consumidores. Como los consumidores de cada mercado tienen distintas disponibilidades a pagar, la demanda agregada va a ser quebrada:

xd = 25 – p si 20 ≤ p ≤ 25 xd = 65 – 3 si 0 ≤ p < 20

La curva de ingresos marginales asociada a la curva de demanda agregada también tendrá dos tramos:

IMg = 25 – 2x si 0 ≤ x ≤ 5 IMg = 65/3 – 2x/3 si x > 5

Unidad 7

54

La condición de primer orden de la maximización de beneficios de un monopolista no discriminador implica IMg = CMg, igualdad que se verifica en el segundo tramo de la curva de IMg. Así: IMg = CMg 65/3 – 2x/3 = 10 x = 17,5 ; p = 15,8 Autoevaluación 1) a) Img = 100 – (2). 2Q = 100 – 4Q

b) Img = Cmg → 100-4Q = 2 → Q= 24,5 unidades. Reemplazando en la función de demanda inversa calculamos el precio: P = 100 – 2 (24.5) → P = 51 c) Beneficio = Ingresos – Costos = P*Q – C(Q) = 51*24.5 – (10 + 2* 24.5) = $1.190.50

2) a) Estamos hablando de corto plazo porque tenemos costos fijos de $80 y costos variables de Q2.

b) P = Cmg por ser competencia perfecta, entonces: 10 = 2Q → Q = 5. c) B = P*Q – C(Q) → B = 10 * 5 – [80 + (5) 2] → B = $ -30. El empresario tiene una pérdida de $30. d) Sí, debe seguir en el mercado a corto plazo porque P > CVM → 10 > (52) /5 y si permanece abierto sólo perderá $30, en cambio si cierra perderá $80 que es la magnitud del costo fijo.

3) a) F. No hay beneficios extraordinarios, pero sí beneficios normales.

b) F. Tales servicios son monopolios naturales porque los Costos Medios de Largo plazo son menores en un nivel de producción suficientemente alta para que un sólo productor pueda abastecer a todo el mercado (Imagine si no lo que serían las tuberías de 3 empresas de agua diferentes que abastezcan al mismo mercado!). Los monopolios naturales pueden ser administrados por el Estado mediante la provisión del servicio o puede ser que el Estado los regule, una vez cedidos en concesión o privatizados, para aproximarse a la situación de competencia . Pero de ello no se debe inferir que un monopolio es naturales si es propiedad del Estado o éste los regula. c) V d) V e) F. El producto ofrecido es único, como podría ser un cuadro por ejemplo. No hay otra banda exactamente con las mismas características. Además sólo ofrece un show, por lo tanto tiene el carácter de monopolio. f) V Los músicos pueden argumentar que existen muchas bandas sustitutas, argentinas o zelandesas y además que existe otra banda que hace “tributos”, que los imita muy bien y que ofrece un “producto” similar a un precio módico en la misma semana que ellos.

4) En la competencia perfecta como existe libre entrada y salida del mercado, existen muchos productores que venden un producto homogéneo y ninguno tiene capacidad de influir en el precio con lo cual los beneficios extraordinarios son nulos a largo plazo. Mientras tanto en el monopolio existen barreras a la entrada con lo cual existe un único productor que tiene capacidad de fijar el precio de mercado y de mantener sus beneficios a largo plazo mediante la venta de un producto único.

Volver al Índice.

Guía de ejercicios Microeconomía- UADE

55

Unidad 6: Los mercados. Casos intermedios: el oligopolio y la competencia monopolística.

Preguntas de Repaso 1) Tiene Ud. que decidir cuál de dos estructuras de mercado, es la que mejor describe a una industria, pero sólo se le permite hacer una pregunta sobre la industria. ¿Qué pregunta debería hacer para determinar si la industria es: (Krugman, 2006)

a) ¿perfectamente competitiva o monopolísticamente competitiva? b) ¿un monopolio o monopolísticamente competitiva?

2) Cada uno de los siguientes bienes y servicios son productos diferenciados. ¿Cuáles están diferenciados debido a la existencia de competencia monopolística y cuáles no? Razone sus respuestas (Krugman, 2006)

a) Escaleras b) Refrescos c) Grandes almacenes d) Acero

3) Una industria que opera bajo competencia monopolística, compuesta por empresas con curvas de costos totales medios en forma de U, está actualmente en equilibrio de largo plazo. Describa cómo se ajusta la industria, tanto en el corto como en el largo plazo, en cada una de las siguientes situaciones (Krugman, 2006)

a) Un cambio tecnológico que aumenta los costos fijos de cada una de las empresas de la industria. b) Un cambio tecnológico que disminuye el costo marginal de cada una de las empresas de la industria.

4) Explique la siguiente frase e ilustre con un gráfico. “Una empresa que opera en competencia monopolística es como un monopolio, pero sin los beneficios del monopolio. En el largo plazo, sus beneficios son nulos, como en competencia perfecta”. 5) ¿Porqué es imposible que en el largo plazo, las empresas de una industria que operan bajo competencia monopolística se fusionen para crear un monopolio? 6) Defina competencia monopolística y oligopolio. Compare con la situación extrema de competencia perfecta y monopolio puro. 7) El oligopolio maximiza beneficios en función de los supuestos alternativos sobre las reacciones de las empresas rivales a variaciones del precio o de la producción. De acuerdo a la forma de reaccionar de los rivales, identificamos 3 modelos (hay más pero son los que están en el programa de la materia): Cournot, Stackelberg y Bertrand. Complete el siguiente cuadro comparativo de los modelos:

Modelo

Cournot Stackelberg Bertrand

Relación entre nº de empresas y

consumidores Pocas empresas y

muchos consumidores.

Productos que fabrican las empresas

Productos diferenciados u

homogéneos

Unidad 7

56

¿Qué es lo que cree la empresa que harán

sus rivales?

Barreras a la entrada

Si

8) Ofrezca un ejemplo en el mundo real de un mercado que se aproxima a cada uno de los siguientes modelos de oligopolio y explique su razonamiento (Baye, 2006).

a) Oligopolio de Cournot. b) Oligopolio de Stackelberg. c) Oligopolio de Bertrand.

9) Seleccione la alternativa correcta:

I) En la solución de Stackelberg al problema del duopolio: a) El líder siempre obtiene, al menos, tantos beneficios como en la solución de Cournot b) En el equilibrio la empresa seguidora no está sobre su curva de reacción cournotiana c) Los beneficios de ambas empresas son necesariamente iguales d) El líder siempre obtiene, al menos, tantos beneficios como el seguidor

II) Suponga un mercado abastecido por dos duopolistas de Cournot con la misma estructura de costos. Si el primero produce la cantidad correspondiente al equilibrio de Cournot, el segundo maximiza el beneficio cuando:

a) Produce la misma cantidad que el primero b) Produce una cantidad menor que el primero c) produce una cantidad mayor que el primero d) No podemos asegurar nada sin conocer la demanda del mercado

III) En la solución del Cártel al problema de un duopolio simétrico, donde las empresas tienen la misma estructura de costos:

a) El beneficio conjunto de ambas empresas es menor que en el caso de la solución de Cournot b) La cantidad total producida es mayor que en el caso de Stackelberg c) El precio del producto es mayor que en la solución de Cournot. d) Una produce y la otra no.

10) Busque en su libro de microeconomía los siguientes conceptos y explíquelos brevemente:

Juego Jugadores Pagos Estrategias Estrategias dominantes Estrategia maximin Juegos con movimientos secuenciales Juegos con movimientos simultáneos Juego de única jugada Juegos repetidos Equilibrio de Nash

Guía de ejercicios Microeconomía- UADE

57

Ejercicios 1) Suponga que la función de demanda inversa del producto de una empresa cuya estructura de mercado es la de competencia monopolística viene dada por P = 100 – 2Q y la C(Q) = 5 + 2Q Calcule el precio y la cantidad que maximizan los beneficios así como los beneficios máximos (Baye, 2006). 2) Es Ud. el directivo de una empresa de competencia monopolística y sus funciones de demanda y de costes vienen dadas por Q = 20 -2P C(Q) = 104 – 14Q + Q2. (Baye, 2006)

a) Calcule la función de demanda inversa del producto de su empresa. b) Determine el precio y el nivel de producción que maximizan los beneficios. c) Calcule los beneficios máximos de su empresa

d) ¿Qué ajustes a largo plazo debería esperar? Explique su respuesta 3) Dado el siguiente gráfico ¿Cuál es el nivel de producción que maximiza los beneficios de la empresa 2 cuando la empresa 1 fabrica cero unidades? ¿Y cuando fabrica Q1* unidades? (Modelo de Cournot) 4) Suponga que la función de demanda inversa de dos duopolistas de Cournot viene dada por: P = 10 – (Q1 + Q2) y que sus costos son nulos (Baye, 2006)

a) ¿Cuál es el ingreso marginal de cada empresa? b) ¿Cuales son las funciones de reacción de las dos empresas? c) ¿Cuáles son las producciones de equilibrio de Cournot? d) ¿Cuál es el precio de equilibrio?

5) La demanda inversa de un duopolio de Cournot con productos homogéneos es P = 100 – 2 (Q1 + Q2) y los costes son C1 (Q1) = 12 Q1 y C2 (Q2) = 20 Q2 (Baye, 2006).

a) Calcule la función de reacción de cada empresa b) Calcule la producción de equilibrio de cada empresa c) Calcule el precio de mercado de equilibrio d) Calcule el beneficio que obtiene cada empresa en equilibrio

6) La demanda inversa de un duopolio de Stackelberg es P = 20.000 – 5Q. Las estructuras de costos del líder (L) y del seguidor (F) respectivamente son CL (QL) = 10.000 QL y CF (QF) = 4.000 QF (Baye, 2006):

a) ¿Cuál es la función de reacción del seguidor? b) Calcule el nivel de producción de equilibrio tanto del seguidor como de la empresa líder. c) Calcule el precio de mercado de equilibrio d) Calcule los beneficios del líder y el seguidor.

Unidad 7

58

D

D

R

D

7) Suponga que la función de demanda inversa de un oligopolio de Stackelberg con productos homogéneos viene dada por P = 50 – (Q1 + Q2) y las funciones de costo de las dos empresas son: C(Q1) = c1 Q1 y C(Q2) = c2 Q2. La empresa 1 es la líder y la empresa 2 la seguidora. Baye, 2006)

a) ¿Cuál es la función de reacción de la empresa 2? b) ¿Cuál es la producción de la empresa 1? c) ¿Cuál es la producción de la empresa 2? d) ¿Cuál es el precio de mercado?

8) Un mercado, cuya función de demanda es Q = 12 – P, está abastecido por dos empresas cuyas funciones de costos son respectivamente, C1 = q2 y C2 = 2q. Si la primera se comporta como un líder de Stackelberg y la segunda como un seguidor, encontrar las cantidades producidas por cada una, y el precio de mercado resultante. 9) Analice un oligopolio de Bertrand compuesto por cuatro empresas que fabrican un producto idéntico con un costo marginal de 100 dólares. La demanda inversa de éste producto es P = 500 -2Q (Baye, 2006):

a) Calcule el nivel de producción de equilibrio en el mercado. c) Calcule el precio de mercado de equilibrio d) Calcule los beneficios de cada empresa.

10) Dada la función de demanda inversa de un mercado P = 400 – 4Q, y dadas dos únicas empresas cuyas funciones de costos son: C1 = 2q2 + 80 y C2 = 8q, encontrar el resultado si las empresas cooperan entre sí, formando un cártel. 11) Dados la siguiente función de demanda inversa: P = 1000 – (Q1 + Q2) y la función de costos de cada empresa Ci (Qi) = 4Qi , compare los resultados que obtendría en cada modelo ¿Qué conclusiones saca?

Cournot Bertrand Stackelberg

Nivel de Producción Q1 =........ Q2 = .......

Precio

Beneficio

12) La demanda de un mercado formado por dos empresas iguales, con costos C1 = C2 = 4q, es P = 200 – Q. Encuentre las cantidades ofrecidas por cada empresa, la cantidad total producida, y el precio de mercado si las empresas:

a) compiten según los supuestos del modelo de Cournot b) compiten según los supuestos del modelo de Bertrand c) deciden cooperar entre sí, formando un cártel

13) Considere el siguiente juego y responda:

Jugador B Estrategia Izquierda Derecha

Arriba 10 , 20 15 , 8 Jugador A Abajo -10 , 7 10 , 10

a) ¿Tiene el jugador B una estrategia dominante? ¿Y qué tal A? b) ¿Cuál es la estrategia segura (maximin) para el jugador B? c) ¿Cuáles son las estrategias de equilibrio de Nash para los jugadores A y B?

Guía de ejercicios Microeconomía- UADE

59

R

14) Suponga que una empresa compite con otras empresas para conseguir clientes. Usted y su rival saben que sus productos estarán obsoletos al final de cada año y deben determinar simultáneamente si van a contratar publicidad o no. Pero, en su industria, la publicidad no eleva la demanda total de la industria, sino que induce a los consumidores a cambiar entre los productos de las distintas empresas.

Empresa B

Estrategia Publicidad No publicidad

Publicidad $4 , $4 $20 , $1 Empresa A

No publicidad $1 , $20 $10 , $10

Así pues, si tanto Ud. como su rival contratan publicidad, las dos campañas publicitarias se compensarán entre sí, y cada una de las empresas obtendrá 4 millones de beneficios. Si ninguna empresa contrata publicidad, cada una obtendrá 10 millones de beneficios. Sin embargo, si una contrata publicidad y otra no, la que contrata obtendrá 20 millones de beneficios y la que no, obtendrá 1 millón de beneficios. ¿Su elección para maximizar los beneficios consiste en contratar publicidad o no? ¿Cuánto espera ganar? (Baye, 2006) 15) Utilice la siguiente forma normal de un juego único para responder a las siguientes preguntas (Baye, 2006),

Jugador 2 Estrategia D E F

A 100 , 125 300 , 250 200 , 100 B 250 , 0 500 , 500 750 , 400 Jugador 1 C 0 , 100 400 , 300 -100 , 350

a) Determine la estrategia dominante de cada jugador si existe. b) Determine la estrategia segura de cada jugador c) Encuentre el equilibrio de Nash.

16) En un juego único con movimientos simultáneos y dos jugadores, cada jugador puede elegir la estrategia A o la estrategia B. Si ambos jugadores eligen la estrategia A, cada uno obtiene un pago de $500. Si ambos eligen la estrategia B, cada uno obtiene un pago de $100. Si el jugador 1 elije la estrategia A y el jugador 2 elije la estrategia B, el jugador 1 gana $0 y el jugador 2 gana $650. Si el jugador 1 elije la estrategia B y el jugador 2 elije la estrategia A, el jugador 1 gana $650 y el jugador 2 gana $0. (Baye, 2006)

a) Muestre el juego en forma normal. b) Determine la estrategia dominante de cada jugador, si existe. c) Determine el equilibrio (o los equilibrios) de Nash para este juego. d) Clasifique los pares de estrategias por pagos agregados (de mayor a menor) e) Puede mantenerse en el equilibrio el resultado con mayores pagos agregados? ¿Porqué si o porqué no?

Unidad 7

60

Estudio de Casos 1) Lea el siguiente artículo de la sección Economía de CLARIN, y luego responda las preguntas sugeridas al final del mismo: CLARÍN, 30/01/2006. MANAGEMENT Una rubia, cuatro amigas y los beneficios de la cooperación Por Manuel Sbdar*.

[email protected] Los cuatro muchachos tomaban aburridos una gaseosa en el bar de la universidad de Princeton. Súbitamente, se despertaron de su sopor. Una rubia despampanante llegó con cuatro amigas. A los muchachos se les iluminaron los ojos. Tal vez no sería una noche aburrida después de todo. Cada cual peinó su jopo y preparó sus mejores armas de seducción. “No se apuren, amigos”, los detuvo John Nash, quien sería Nobel de Economía en 1994, “si vamos todos sobre la rubia, ninguno se irá acompañado esta noche”. Los muchachos lo miraron desconcertados. Nash prosiguió: “Si competimos por la rubia, nos estorbaremos. En el mejor de los casos, sólo uno de nosotros se irá con ella. Pero si nos rechaza, también perderemos a sus amigas. A las mujeres no les gusta ser la segunda opción. Si vamos directo sobre las amigas es más probable que todos logremos nuestro objetivo”. Dicho y hecho. La rubia se quedó sola en la barra mientras los muchachos bailaban con sus amigas. Así es como Nash expone su teoría de los juegos en el film Una mente brillante. La teoría de los juegos es una rama de las matemáticas que estudia situaciones estratégicas donde los agentes eligen distintos cursos de acción para maximizar sus beneficios. La escuela de economía clásica fundada por Adam Smith sostenía que el mecanismo de la competencia hacía que siempre el mejor lograra su objetivo. El ejemplo de la rubia nos enseña que esto no siempre es verdad. En muchos casos, cuando todos buscan su propio interés, todos acaban perdiendo. Entonces, tal vez no sea mala idea cooperar. Si bien fue John Nash quien pasó a la posteridad como el padre de la teoría de los juegos en la década del 50, los antecedentes se remontan hasta el economista francés del siglo XVIII, Antoine Augustin Cournot y los profesores de Nash en Princeton, John Von Neumann y Oskar Morgenstern. La computación, la ética y la biología son algunas de las disciplinas donde se aplica el marco conceptual de la teoría de los juegos. En el campo del management es con frecuencia utilizada para analizar fenómenos tan disímiles como las negociaciones y las estructuras de mercado oligopólicas. En general, se inserta dentro del campo más amplio de la teoría de la decisión, cómo deciden los individuos y cómo pueden llegar a resultados óptimos en situaciones de incertidumbre, información imperfecta y alto riesgo. Es en este marco donde cobra relevancia la teoría de los juegos como un instrumento para analizar la forma en que se decide así como una herramienta para optimizar el rendimiento. Raymond W. Smith, CEO de la telefónica Bell Atlantic, dijo allá por 1996: “En Bell Atlantic, notamos que las enseñanzas de la teoría de los juegos nos brindan una visión más amplia de nuestra situación y nos provee un mejor enfoque para el planeamiento corporativo”. Veamos un ejemplo. Usted es el CEO de una compañía que opera en un mercado oligopólico. Su objetivo es, desde luego, complacer a los accionistas maximizando los beneficios. Usted tiene el incentivo de reducir el precio de sus productos para “robar” market share a sus competidores. El mayor volumen de ventas podría más que compensar la caída en el precio por unidad. Sus beneficios se dispararían. Pero piénselo bien. ¿Cómo reaccionarían sus competidores ante su agresión? Tal vez lo imiten y se embarquen en una guerra de precios que destruya los beneficios de todos. Es posible entonces que piense en cooperar con sus “íntimos” competidores. Al controlar buena parte del mercado, podrían fijar un volumen de producción que maximice los beneficios a expensas del bolsillo de los consumidores. “Producimos menos y lo vendemos más caro”. En este punto, la ley antitrust se convierten en un jugador imprescindible para equilibrar el marcador y desalentar esta poco feliz tentación. En virtud de la oligopolización de los mercados mundiales durante el siglo pasado, la teoría de los juegos ha tenido grandes desarrollos. En 2005, el Nobel de Economía fue para los teóricos Thomas Schelling y Robert Aumann, quienes profundizaron en la senda de Nash. La teoría de los juegos es el producto de un pensamiento de investigación académica que aporta un modelo de decisión eficiente aplicable al management. A diferencia de la teoría sobre colas, la de juegos esta muy cerca de cuestiones tan intangibles como la motivación, la creatividad y la pasión. Profundizar en esta teoría y sus múltiples aplicaciones es una jugada con altísimas probabilidades de éxito. *Manuel Sbdar es Director de Educación Ejecutiva de la Escuela de Negocios de la Universidad Torcuato Di Tella [email protected]

Guía de ejercicios Microeconomía- UADE

61

a) De acuerdo al artículo, ¿Qué es la teoría de los juegos? ¿Quién la desarrolló y cuáles fueron sus antecedentes?

b) ¿Le parece a Ud. que la película describe correctamente el planteo hecho por Nash? (Ayuda: considere la siguiente frase “Si vamos directo sobre las amigas es más probable que todos logremos nuestro objetivo”. También puede leer en “Economía de Empresa, por Baye. Mc Graw Hill, 5ta edición, pag 358 sobre el “equilibrio de Opie”).

c) ¿En qué campos es aplicable la teoría de los juegos y para qué? d) ¿Qué es la teoría de la decisión? e) Para el caso del mercado oligopólico, ¿qué significa “El mayor volumen de ventas podría más

que compensar la caída en el precio por unidad.”? f) ¿Qué se busca con la ley antitrust?

2) Dilema del prisionero: Dos prisioneros se encuentran encarcelados en celdas separadas por un grave delito que han cometido. Sin embargo, el fiscal sólo tiene pruebas suficientemente irrefutables para acusarlos de una falta leve, por lo que la condena es, de un año de cárcel. A cada uno de ellos se les comunica que si uno confiesa y el otro no, el primero quedará impune y el otro pasará 20 años en la cárcel. Si confiesan los dos, serán condenados a una pena intermedia de 5 años de cárcel. Los prisioneros no pueden comunicarse entre sí. El siguiente cuadro resume los resultados:

Prisionero Y Estrategia Confesar No confesar

Confesar 5 años para cada uno

X queda libre y 20

años para Y Prisionero X

No confesar 20 años

para X e Y queda libre

1 año para cada uno

a) ¿Quiénes son los jugadores y cuáles son sus estrategias? b) ¿Cuál es la estrategia dominante de cada uno de los jugadores? ¿Qué implica que una

estrategia sea la dominante? c) ¿Cuál es el equilibrio del juego? ¿Qué tipo de equilibrio es? ¿Existe una solución mejor? ¿Cuál?

3) Caso: El dilema del prisionero y la tendencia a hacer trampa en los acuerdos de cartel (Mansfield, 1998) La teoría de los juegos es útil para explicar una amplia variedad de fenómenos, incluyendo la tendencia a hacer trampa por parte de los miembros de un cartel (es decir, la tendencia a rebajar el precio de manera oculta). Suponga que dos empresas –Atlas y Baker- forman un cartel. Cada empresa tiene dos estrategias posibles: la de atenerse al acuerdo de cartel y la de hacer trampa. Hay cuatro resultados posibles, según qué estrategia siga cada empresa.

Estrategias posibles para Baker Estrategias posibles para Atlas Atenerse al acuerdo Hacer trampa

Atenerse al acuerdo Atlas y Baker comparten los beneficios monopólicos

Baker se desempeña mejor que bajo el acuerdo; Atlas se desempeña mucho peor

Hacer trampa Atlas se desempeña mejor que bajo el acuerdo; Baker se desempeña mucho peor

Ambas empresas se desempeñan algo peor que bajo el acuerdo

a) Si Baker se atiene al acuerdo ¿cuál es la mejor estrategia para Atlas? Si Baker hace trampa, ¿cuál es la mejor estrategia para Atlas? ¿Qué estrategia elegirá Atlas?

b) Si Atlas se atiene al acuerdo ¿cuál es la mejor estrategia para Baker? Si Atlas hace trampa, ¿cuál es la mejor estrategia para Baker? ¿Qué estrategia elegirá Baker?

c) En una situación de este tipo (a menudo llamada el “dilema del prisionero”), ¿ambas partes se desempeñan peor que si cooperaran?

d) ¿Qué tipo de artificio puede adoptar el cartel para desincentivar a hacer trampa?

Unidad 7

62

4) Caso: Lanzamiento de un nuevo producto. Su empresa tiene que decidir si va a introducir o no un nuevo producto. Si introduce el nuevo producto, su rival tendrá que decidir si va a clonar o no el nuevo producto. Si no introduce el nuevo producto, Ud. y su rival ganarán 1 millón de dólares cada una. Si introduce un nuevo producto y su rival lo clona, perderá 5 millones de dólares y su rival ganará 20 millones (ha invertido mucho en inversión y desarrollo, y su rival no tiene que hacer esta inversión para competir con su clon). Si introduce el nuevo producto y su rival no lo clona, obtendrá 100 millones de dólares y su rival obtendrá cero.

a) Dibuje la forma extendida de éste juego. b) ¿Debe introducir el nuevo producto? c) ¿Cómo cambiaría su respuesta si su rival ha “prometido” no clonar el producto? d) ¿Qué haría si las leyes sobre patentes impidieran que su rival clonara el producto?

5) Caso: La Publicidad, las gafas y la CFC (Mansfield, 1986) La teoría de la competencia monopolística realza el significado de los gastos de venta, incluyendo la publicidad. El mercado de anteojos en las grandes ciudades tiene muchas de las características de la competencia monopolística, habiendo un gran número de vendedores de anteojos, siendo el producto de cada uno un poquito diferente del de los otros. Algunos estados han prohibido la publicidad de precios por parte de vendedores de anteojos. El siguiente cuadro muestra el precio medio de los anteojos en estos estados, así como en los estados que no tienen restricciones a la publicidad.

Precio promedio

Naturaleza de la ley del Estado Anteojos

Anteojos y revisación de la vista

Prohibición de publicidad $33.04 $40.96

Ausencia de prohibición $26.34 $37.10

a) Como la publicidad es un gasto de venta que requiere el uso de recursos, ¿es razonable

esperar que los costos de una empresa serían menores sin no tuviera que efectuar publicidad?

b) Si los costos de publicidad aumentan los costos, ¿porqué los precios de los anteojos tendieron a ser menores en los estados que no prohíben la publicidad?

c) En 1978, la Comisión Federal de Comercio (CFC) declaró que las restricciones sobre la publicidad de anteojos es ilegal ¿Porqué tomó esa acción la CFC?

6) Caso: Examen Final Al calificar un examen final un profesor descubre que dos estudiantes tienen respuestas casi idénticas. Habla con cada uno de ellos por separado y les dice que está seguro de que compartieron las respuestas, pero no puede saber con seguridad quién copió a quién. Les ofrece un trato: si ambos firman una declaración admitiendo que han hecho trampa, los dos serán suspendidos en el curso. Si sólo uno firma la declaración, podrá anular la matrícula del curso y el otro será expulsado de la universidad. Por último, si no firma ninguno de los dos la declaración, ambos obtendrán un aprobado por el curso porque el profesor no tiene suficientes pruebas de que han hecho trampa. Elabore la matriz de rendimientos resultante suponiendo que los estudiantes no pueden comunicarse entre sí. ¿Tiene cada estudiante una estrategia dominante? (Frank, 2005) 7) Caso: Fallo de la Comisión Nacional de defensa de la Competencia en contra de empresas de Cemento Pórtland. Lea el siguiente artículo publicado el 29 de Agosto de 2008 en el suplemento I-Eco del diario Clarín y responda las preguntas que siguen. LA JUSTICIA AVALO UNA DECISION DEL EX MINISTRO LAVAGNA

Multa millonaria contra cinco cementeras por cartelización Las principales firmas y la cámara que las agrupa deberán pagar $ 308 millones.

Guía de ejercicios Microeconomía- UADE

63

Por Lucio Fernández Moores | [email protected] La Sala B de la Cámara Nacional de Apelaciones en lo Penal Económico confirmó una multimillonaria multa que el gobierno de Néstor Kirchner había aplicado contra cinco empresas cementeras acusadas de haberse repartido negocios para cobrarle sobreprecios al Estado. La multa asciende a 308,4 millones de pesos y ahora las empresas podrán apelar a la Corte Suprema de Justicia a través de un recurso extraordinario. Se trata de la mayor multa aplicada en defensa de la competencia. La sanción había sido aplicada el 26 de julio de 2005, cuando el ministro de Economía era Roberto Lavagna. En aquel entonces, la Comisión Nacional de Defensa de la Competencia, que investigó el tema durante cinco años tras una denuncia periodística, llegó a la conclusión de que las empresas actuaron en forma "cartelizada" y establecieron un sistema de información compartida a través de la Asociación de Cemento Portland. "Estas compañías pactaron acuerdos para mantener precios elevados a través de la concertación de cuotas", dijo al anunciar la aplicación de la multa el por entonces ministro Lavagna. La investigación alcanzó al período 1981-1999 y fue encabezada por los vocales Diego Pávolo y Humberto Guardia Mendonça. Ahora, la Cámara en lo Penal Económico, tribunal de apelación para este tipo de procedimientos, confirmó las sanciones impuestas por el Poder Ejecutivo. El fallo fue rubricado por los jueces Carlos Pizzatelli, Marcos Grabivker y Roberto Hornos, aunque este último magistrado se pronunció por reducir las multas en un 20%. De acuerdo con el fallo, al que Clarín tuvo acceso, las empresas, a través del "intercambio de información competitivamente sensible" que obtuvieron a través de la Asociación de Fabricantes de Cemento Portland, coordinaron "alcanzar una finalidad en común". Esto fue "llevar a la práctica la asignación de cuotas y el reparto del mercado acordado previamente, y controlar el acatamiento --propio y ajeno-- a lo pactado", según confirmaron los camaristas. El tribunal también confirmó la exigencia formulada en su momento por la Comisión de Defensa de la Competencia a la Asociación de Fabricantes de Cemento Portland para que "adopte los recaudos para que la información que concentra sea accesible sólo para su personal, quien deberá guardar estricta confidencialidad". Además, se le exigió a la asociación que "adecue el diseño y el funcionamiento del sistema estadístico a las exigencias legales correspondientes". Las empresas sancionadas son Loma Negra, Juan Minetti, Cemento Avellaneda, Petroquímica Rivadavia y Cementos San Martín, además de la Asociación que las agrupa (ver infografía). Los camaristas rechazaron todas las objeciones de las empresas, principalmente en lo que refiere a la supuesta prescripción de los hechos investigados y a la aplicación de la Ley 22.262 en lugar de la para ellas "más benigna" Ley 25.156. La Ley 22.262 aplicada en este caso y confirmada por la Justicia castiga "los actos o conductas (...) que limiten, restrinjan o distorsionen la competencia o que constituyan abuso de una posición dominante en el mercado, de modo que pueda resultar perjuicio para el interés económico general". Para Defensa de la Competencia --y ahora también para la Justicia-- hubo "un acuerdo global a nivel país de asignación de cuotas y de participaciones en el mercado del cemento portland". En función de éste, según las autoridades, "se realizaban concertadamente ajustes de los despachos y las participaciones de cada empresa por provincias o zonas, y se realizaban acuerdos de precios y demás condiciones comerciales en diferentes localidades o zonas del país".

a) En este caso las empresas decidieron actuar cooperativamente. ¿Qué lograron con la cooperación en vez de competir entre ellas? b) ¿Cómo se controlaban las empresas entre sí para evitar que alguna de ellas hiciera “trampa”? c) ¿Por qué es necesaria una ley que proteja la competencia entre empresas?

Unidad 7

64

AUTOEVALUACIÓN. Puntaje obtenido:

1) Dada la siguiente matriz de resultados (ganancias) de un par de empresas que deben decidir el monto a destinar a I&D. Responda (2 puntos):

Empresa 1

Estrategia Elevado presupuesto de investigación

Bajo presupuesto de investigación

Elevado presupuesto de investigación 40 , 200 100 , 60

Empresa 2 Bajo presupuesto de

investigación 30 , 0 80 , 40

a) ¿Tiene alguna de las empresas una estrategia dominante en el juego representado en el

cuadro superior? Si es así diga cuál empresa (o cuales empresas) y cuál es la estrategia dominante. En cualquier caso diga qué estrategia sigue cada empresa.

b) ¿Existe un equilibrio de Nash en éste juego? Explique el concepto.

2) Coloque Verdadero o Falso. Si es falso, subraye la parte falsa para justificar (2,5 puntos):

a) La competencia monopolística comparte con la competencia perfecta la existencia de numerosas empresas y la libre entrada y salida del mercado. Sólo que, en la competencia monopolística, cada empresa produce un artículo que no es homogéneo sino que es un sustitutivo cercano, pero imperfecto, de los productos de las otras.

b) El rasgo característico de los mercados oligopolísticos es la independencia de las empresas. La misma es analizada mediante la teoría de los juegos.

c) Una función de reacción define el nivel de precios que maximiza el nivel de beneficios de la empresa dados los niveles de producción de las otras.

d) El equilibrio de Cournot es la situación en la que ninguna empresa tiene el incentivo para cambiar su nivel de producción una vez que la lider fijó su nivel de producción.

e) El oligopolio de Bertrand es deseable desde el punto de vista de los productores porque dá lugar al resultado que se obtiene en competencia perfecta.

3) La curva de demanda del mercado de un líder y un seguidor de Stackelberg viene dada por P = 10 – Q. Si cada uno puede producir con un costo marginal de 2, halle el precio y la cantidad de equilibrio de cada uno (3,5 puntos). (Frank, 2005) 4) Los duopolistas de Bertrand se enfrentan con una curva de demanda del mercado que viene dada por P = 56 – 2Q. Cada uno puede producir con un costo marginal constante de 20 por unidad. Halle el precio y la cantidad de equilibrio (2 puntos) (Frank, 2005).

Guía de ejercicios Microeconomía- UADE

65

D

Respuestas Sugeridas

8) La empresa seguidora, la 2, determina la cantidad que maximiza sus beneficios, tomando como dada la producción de la empresa líder. Para ello debe satisfacer la condición IMg2 = CMg2. El CMg2 se obtiene derivando la función de costo total, por lo tanto es igual a 2. El IMg2 lo podemos obtener derivando la función de ingreso total para la empresa 2, que resultará de:

IT2 = p.q2 = (12 – Q).q2 = [12 – (q1 + q2)].q2 = 12q2 – q1q2 – q22

Si derivamos la expresión anterior para q2 (teniendo en cuenta que q1 es constante para la empresa 2), llegamos al ingreso marginal:

IMg2 = 12 – q1 – 2q2 Lo igualamos con el costo marginal, y despejamos q1. La función que obtengamos será la curva de reacción de la empresa 2. 12 – q1 – 2q2 = 2 10 – q1 = 2q2 5 – q1/2 = q2 Función de reacción de la empresa 2 Ahora planteamos el problema de maximización de la empresa líder. La empresa 1 maximiza sus beneficios cuando IMg1 = CMg1 pero a diferencia de la empresa seguidora, conoce la función de reacción de su competidora y la incluye en su proceso de decisión sobre cuando producir. Dado que CMg1 = 2q1, planteamos la función de ingreso total:

IT1 = p.q1 = (12 – Q).q1 = [12 – (q1 + q2)].q1 = 12q1 – q1q2 – q12

Introducimos la función de reacción de la empresa 2:

IT1 = 12q1 – q1q2 – q12 = 12q1 – q1(5 – q1/2) – q1

2 = 12q1 – 5q1 + q12/2 – q1

2 = 7q1 – q12/2

Y derivando:

IMg1 = 7 – q1

Al igualar con el CMg encontramos la cantidad que ofrecerá la empresa líder:

IMg1 = CMg1 7 – q1 = 2q1 7 = 3q1 q1 = 7/3

Y reemplazando en la función de reacción de la empresa 2, encontramos q2 = 23/6. Por lo tanto la cantidad ofrecida en el mercado es Q = q1 + q2 = 37/6 y el precio de mercado P = 35/6. 12) a) Solución de Cournot: En el modelo de Cournot cada empresa, individual y simultáneamente a su rival, determina la cantidad que maximiza sus beneficios, tomando como dada la producción de su competidora. Así, cada empresa iguala su ingreso marginal con sus costos marginales, y de esa condición de maximización encontramos la función de reacción de cada empresa. Empresa 1:

IT1 = p.q1 = (200 – Q).q1 = [200 – (q1 + q2)].q1 = 200q1 – q1q2 – q12

IMg1 = 200 – q2 – 2q1

IMg1 = CMg1 200 – q2 – 2q1 = 4 196 – q2 = 2q1 q1 = 98 – q2/2 Curva de reacción Emp.1

Empresa 2:

IT2 = p.q2= (200 – Q).q2 = [200 – (q1 + q2)].q2 = 200q2 – q1q2 – q22

IMg2 = 200 – q1 – 2q2

IMg2 = CMg2 200 – q1 – 2q2 = 4 196 – q1 = 2q2 q2 = 98 – q1/2 Curva de reacción Emp.2

Con ambas curvas de reacción armamos un sistema de dos ecuaciones con dos incógnitas. Resolviendo encontramos q1 = q2 = 69; Q = 138; P = 61 b) Solución de Bertrand: Al competir por precios y tener costos similares, la guerra de precios desatada llevará a la solución de competencia perfecta, donde P = CMgi, por lo tanto: P = 4; Q = 196; q1 = q2 = 98

Unidad 7

66

R

c) Solución de Cartel: las empresas cooperan para conseguir la maximización del beneficio conjunto actuando, por tanto, como un monopolio multiplanta, donde el ingreso marginal en conjunto es igual a los costos marginales respectivos, o sea:

IMg = CMg1 = CMg2 200 – 2Q = 4 196/2 = Q Q = 98; q1=q2= 49; P = 102 13) a) El jugador B no tiene estrategia dominante (si A elige Arriba, le conviene elegir Izquierda, mientras que si A elige Abajo, le conviene elegir Derecha), pero el jugador A si la tiene, ya que independientemente de lo que elija B, siempre le conviene elegir Arriba. b) La estrategia segura para B es elegir Derecha, ya que de esa forma logra el mejor de sus peores resultados posibles. c) El equilibrio de Nash es Izquierda-Arriba.

10) q1 = 2; q2 = 47; Q = 49; P = 204 15) a) El jugador 1 tiene como estrategia dominante a la alternativa B, pero el jugador 2 no tiene estrategia dominante. b) La estrategia segura para B es elegir E. c) El equilibrio de Nash es B-E. Autoevaluación 1) a) Sí, la empresa 2 tiene una estrategia dominante: cualquiera sea la estrategia de la empresa 1, la mejor estrategia para 2 es tener un elevado presupuesto para investigación. La empresa 1, en cambio no tiene estrategia dominante. Si 2 elije bajo presupuesto para investigación, lo mejor para 1 es elegir bajo presupuesto para investigación. Si 2 elije alto presupuesto para investigación, lo mejor para 1 es elegir alto presupuesto para investigación. Como 1 puede predecir que 2 elegirá su estrategia dominante, esto es elevado presupuesto para investigación, luego, 1 elegirá elevado presupuesto para investigación. b) El equilibrio de Nash es: “elevado presupuesto para investigación- elevado presupuesto para investigación”. Esto significa que la estrategia de cada uno de los jugadores es óptima, dada la elección del otro. 2) a) V b) F independencia → interdependencia c) F nivel de precios → nivel de producción d) F una vez que la lider fijó su nivel de producción → dada la producción de otra empresa e) F desde el punto de vista de los productores → desde el punto de vista de los consumidores 3) La curva de ingreso marginal de la empresa 2 viene dada por 10 – Q1 –Q2. Partiendo de la igualdad IM = CM = 2, obtenemos la función de reacción de la empresa 2: R2 (Q1) = Q2

* = 4 – (Q1/ 2). Introduciendo el resultado en la función de demanda de la empresa 1, tenemos que P1 = 10 – Q1 -4 + (Q1/ 2) = 6 - (Q1/ 2) y la curva de ingreso marginal correspondiente, IM1 = 6 – Q1. IM1 = CM = 2, de donde se obtiene Q1

* = 4. Esto significa que Q2 será 2 unidades, en el caso en el que

la producción total del mercado es de 6 unidades. El precio del mercado será 10 – 6 = 4. 4) La solución consiste en que ambas empresas fijen un precio igual a su costo marginal, P = CM = 20. La producción de la industria es determinada por la demanda del mercado: 20 = 56 – 2Q, implica que Q = 18. Las empresas se reparten el mercado por igual, por lo que cada una produce la mitad de la producción de la industria Q1 = Q2 = Q / 2 = 9.

Volver al Índice.

Guía de ejercicios Microeconomía- UADE

67

Unidad 7: Introducción a los mercados de factores. Preguntas de Repaso 1) Explique la siguiente frase: “las demandas de factores, a diferencia de la demanda de bienes y servicios de los consumidores, son demandas derivadas”. 2) La demanda de factores de producción citados a continuación han aumentado. ¿Qué conclusiones puede extraer sobre las variaciones de las demandas de los bienes de consumo relacionados con ellos? Si las demandas de bienes de consumo no varían, ¿qué otra explicación tiene un aumento de las demandas derivadas de éstos artículos? (Pindyck, 2000).

a) Los chips de memoria para computadoras. b) El combustible diesel para aviones de pasajeros. c) El papel utilizado para imprimir periódicos. d) El aluminio utilizado para las latas de bebidas.

3) Dado el siguiente gráfico explique qué es la Renta Económica, qué área representaría la Renta Económica del factor trabajo y cuál podría representar al Renta Económica del factor tierra.

4) En un mercado de factores competitivo, explique porqué la curva de Ingreso del Producto Marginal tiene pendiente negativa cuando el mercado de productos es competitivo y cuando no lo es. Grafique. 5) Explique que ocurriría con las diferencias salariales como consecuencia de (Samuelson, 2003):

a) Un aumento del costo de estudiar en la universidad b) La libertad de migración entre los países europeos c) La introducción de la educación pública y gratuita en un país en el que ésta es privada y cara. d) Un gran aumento (gracias a los cambios tecnológicos) en el número de personas a las que llegan los programas populares de deportes y entretenimiento.

6) Explique porqué la curva de oferta de trabajo “vuelve hacia atrás” a niveles de salario suficientemente altos. Grafique y muestre a partir de qué punto “el efecto ingreso” es mayor que “el efecto sustitución”.

Unidad 7

68

R

Ejercicios 1) Dados los siguientes datos:

a) ¿Cuántas unidades del insumo x debe utilizar la empresa maximizadora de beneficios si el mismo cuesta $12 por unidad? b) ¿Cómo se modificaría su respuesta si el precio del insumo sube a $15? ¿Y si baja a $6? c) Recalcule el Valor del Producto Marginal si el precio del producto es de $2 ¿Cuántas unidades del insumo x utilizará la empresa si el precio de éste sigue siendo $12?

2) Dado el siguiente gráfico, averigüe cuántas unidades de trabajo contrataría la empresa si no vendiera su producto a $2 la unidad sino a $3. (Frank, 2005. Adaptado) 3) Modelo Renta- Ocio:

a) Suponga que el salario es de $10 la hora. Halle la ecuación de la restricción presupuestaria de la renta y el ocio y represéntela gráficamente. b) Suponga que ante tal salario el individuo elige h = 10 horas de ocio. Halle la renta diaria M del trabajador correspondiente a esa cantidad de ocio.

4) Curvas de oferta de trabajo:

a) García quiere ganar $200 al día porque con esa cantidad puede vivir cómodamente y cumplir todos sus compromisos económicos. Represente su curva de oferta de trabajo. ¿Qué implicancia tiene esta curva? ¿Qué comportamiento tendrá García frente al ocio cuando le aumenten el sueldo? b) Perez piensa que la renta y el ocio son complementarios perfectos en una proporción 10 a 1 (que exige una hora de ocio por cada $10 de renta). Halle la demanda de ocio correspondiente a un salario de w = $20.

5) Una fábrica textil utiliza trabajo y capital. El precio de una unidad de trabajo es $5 y el precio de una unidad de capital es $6. El producto marginal de una unidad de trabajo es el mismo que el producto marginal de una unidad de capital. ¿Maximiza beneficios la empresa (la cual es competidora perfecta)? Explique (Mansfield, 1998)

Cantidad de x

(insumo)

Producto marginal

Precio del Producto

($)

Valor del Producto

Marginal ($) 3 8 3 24 4 7 3 21 5 6 3 18 6 5 3 15 7 4 3 12 8 3 3 9 9 2 3 6 10 1 3 3

Producto marginal del trabajo (unidad de producción por unidad de trabajo)

120 80 40 L (hora- persona al día)

PML

Valor del producto marginal ($ por unidad de trabajo)

120 80 40 L (hora- persona al día)

8

W =12

16

VPML = P x PML

Cantidad óptima de trabajadores cuando w=12

6

4

8

Guía de ejercicios Microeconomía- UADE

69

R

R

6) La demanda de trabajo de una industria viene dada por la curva Ld = 1800 – 10 w, donde L es el trabajo demandado cada día y w es el salario. La curva de oferta viene dada por Ls = 30w.

a) ¿Cuáles son el salario de equilibrio y la cantidad contratada de trabajo? ¿Cuál es la renta económica que ganan los trabajadores? b) Suponga ahora que el único trabajo existente es controlado por un sindicato monopolístico que desea maximizar la renta económica que ganan los afiliados. ¿Cuál será la cantidad empleada de trabajo y el salario? ¿Qué diferencia hay entre su respuesta y la del punto anterior? c) ¿Cuál será el nivel de empleo y el salario fijado por una empresa monopsonista, es decir, la única demandante en el mercado de trabajo? ¿Qué ocurre con la renta económica del trabajo?

7) Halle la demanda óptima de ocio correspondiente a un salario de w = $20 la hora de una persona que considera que la renta y el ocio son sustitutivos perfectos en una proporción 10 a 1 (está dispuesta a sacrificar 1 hora de ocio por $10 de renta). Pista: Las curvas de indiferencia de esta persona son líneas rectas, M = a – 10 h correspondientes a distintos valores de a (Frank, 2005). Estudio de Casos 1) La curva de oferta de Servicios Médicos (Mansfield, 1998) Martín Feldstein estimó que los médicos tienen una curva de oferta de trabajo que se dobla hacia atrás. Determinó que la elasticidad precio de la oferta de los servicios médicos es alrededor de -0.91.

a) Dibuje un gráfico con las horas por semana de ocio en el eje horizontal y el ingreso proveniente del trabajo en el eje vertical. Siendo el ocio un bien y el ingreso que proviene del trabajo otro bien, construya la recta presupuestaria y las curvas de indiferencia de un médico individual.

b) Usando el gráfico construido en (a), muestre cómo la cantidad deseada de ocio por parte del médico sufre la influencia de la disminución en la tasa salarial si su curva de oferta de trabajo se dobla hacia atrás.

c) La Asociación Médica Norteamericana ha argumentado que toda legislación que reduzca los aranceles médicos reducirá la oferta de los servicios médicos ¿Es cierto?

2) Caso: los diferentes efectos de las leyes de salario mínimo en competencia perfecta (Mansfield, 1998) El Congreso ha promulgado muchas leyes de salario mínimo. Siendo el salario mínimo sólo 0.25 ctvos de dólar por hora en 1938, llegó a US$ 30.35 a principios de 1984. Suponga que un salario mínimo, OPm, se establece en un mercado laboral competitivo, siendo las siguientes las curvas de demanda y oferta de trabajo.

Pm

P

0 C B A

Precio del trabajo

Cantidad de trabajo

Oferta

Demanda

Unidad 7

70

a) Afectará el salario mínimo el nivel de empleo? Si es así, ¿qué magnitud tendrá ese efecto? b) ¿Cuántos empleadores (y trabajadores) eluden el salario mínimo? c) ¿El salario mínimo afecta más a un tipo de trabajadores que a otros? Si es así, ¿qué tipo de

trabajadores resulta más afectado? 3) Caso: Los efectos de las leyes de salario mínimo bajo el monopsonio (Mansfield, 1998) En un ejemplo anterior examinamos los efectos de un salario mínimo cuando el mercado de trabajo es perfectamente competitivo. Veamos ahora sus efectos cuando el mercado de trabajo es monopsónico. No habiendo salario mínimo, suponga que la curva de oferta de trabajo, la curva de producto ingreso marginal del trabajo y la curva de gasto marginal en el trabajo son las que se muestran a continuación:

a) Si se establece un salario mínimo de OWo, ¿cuánto trabajo contratará el monopsonista? ¿Habrá desempleo? Si la respuesta es afirmativa ¿qué proporción de desempleo?

b) Si el salario mínimo es OW1, cuánto trabajo contratará el monopsonista? ¿Habrá desempleo? Si la respuesta es afirmativa ¿qué proporción de desempleo?

c) Bajo monopsonio ¿reduce el empleo un salario mínimo? d) ¿Piensa Ud. que la mayoría de los mercados de trabajo de bajos salarios son monopsónicos?

4) Caso: Efectos económicos de los sindicatos (Mansfield, 1998) Suponga que es posible dividir la economía en un sector sindicalizado y en un sector no sindicalizado. Sea Du la curva de demanda de trabajo del sector sindicalizado, Dn la curva de demanda de trabajo del sector no sindicalizado y Dc la curva de demanda de trabajo de toda la economía.

L3 L2

L0 0 L1

G

H

K

Salario (pesos)

Cantidad de trabajo

Gasto marginal

Oferta

Producto Ingreso marginal

E

S W1

W0

35 40 60 66 100

12

0

Du Dn

Dc

Tasa salarial (pesos)

Millones de trabajadores

Oferta

4

6

8

10

Guía de ejercicios Microeconomía- UADE

71

a) Antes de la aparición del sindicato ¿cuál es la tasa salarial? b) Si el sindicato eleva la tasa salarial a $10 en el sector sindicalizado ¿cuántos trabajadores

despedirá el sector sindicalizado? c) Si todos estos trabajadores obtienen empleo en el sector no sindicalizado ¿cuál será el

efecto en la tasa salarial del sector no sindicalizado? d) A veces los sindicatos hacen que los empleadores empleen más trabajadores de los que ellos

quisieran. Por ejemplo, a menudo se ha logrado que los ferrocarriles contrataran más empleados operativos por tres de lo que a los administradores les parecía necesario (o rentable). En el corto plazo, tales prácticas incrementan a menudo el empleo de los miembros del sindicato ¿persiste ese efecto en el largo plazo?

5) Lea el siguiente artículo de Clarín, y luego responda las preguntas sugeridas al final del mismo: 20 marzo 2007-03-29

LA MARCHA DE LA ECONOMIA : INTERVENCION DEL MINISTERIO DE TRABAJO

El Gobierno busca destrabar la pulseada salarial de la UOM

Tras otro fracaso de la negociación convocó al gremio y las empresas a reunirse el lunes. Este convenio es clave, ya que podría marcar los pasos de todas las paritarias. Sin acuerdo, la paritaria que reúne a los representantes de la Unión Obrera Metalúrgica (UOM) y cinco cámaras empresarias industriales metalúrgicas pasó a un cuarto intermedio hasta el próximo lunes. Fue luego de que interviniera de manera directa el Ministerio de Trabajo, que busca de esta forma destrabar una pulseada salarial clave: por tratarse de una de las primeras paritarias de actividad, este convenio se ha transformado en un convenio testigo, cuyo resultado podría marcar los pasos a las negociaciones salariales de 2007. Si bien al término de las deliberaciones —fue el tercer intento en ocho días— el titular de la UOM, Antonio Caló, dijo que "todavía no nos pudimos poner de acuerdo, lamentablemente, pero avanzamos bastante", se sabe que la distancia que separa el reclamo gremial de la oferta que está haciendo la patronal es apreciable. La UOM quiere un 20% de aumento y una redefinición de las categorías: eliminar la más baja —peón— y crear dos nuevas en la escala superior. Del lado empresario, ofrecieron hasta el 10,5%. Sin embargo, el convenio metalúrgico arrastra un retraso salarial importante con relación a otras actividades, como sería el caso de camioneros o mecánicos (Smata). Eso hace que los dirigentes sindicales estén más apremiados por alcanzar un 20%, como mínimo, de tal manera de alinearse con otros gremios. Del lado empresario, si bien admiten que hay mayor actividad, plantean que la oferta salarial que pueden ofrecer está limitada por la fuerte competencia china y brasileña. El convenio de la UOM, que engloba a más de 200.000 trabajadores, venció el 28 de febrero pasado. Pero incluye una cláusula de paz social hasta el 30 de abril, lo que deja margen para que sigan las negociaciones durante más de un mes. Sin embargo, tanto el sector empresario como el sindical quieren cerrar la negociación antes de fin de mes. Por eso Caló dijo que la UOM continúa en "estado de alerta y movilización", pero descartó que durante la semana la organización sindical disponga la implementación de "un plan de lucha". Y del lado empresario admitieron que "empezamos a diseñar el acuerdo". Una variante que comenzó a barajarse en la reunión fue la posibilidad de que el sector empresario eleve su oferta salarial pero en forma escalonada —en dos o tres etapas— y que la vigencia del convenio se extienda más allá de un año. También se conversó sobre la posibilidad de constituir comisiones técnicas que aborden el tema de la competitividad industrial, en especial frente a las importaciones chinas y brasileñas. Y el de las categorías. Por ejemplo, los empresarios habrían argumentado que en el 40% de las fábricas metalúrgicas tiene efectividad la categoría del peón, desestimando el pedido de la UOM de eliminar esa categoría porque no tendría vigencia efectiva. Se descuenta que en la semana habrá contactos informales entre las partes para preparar la reunión del lunes.

Unidad 7

72

a) Comente sintéticamente qué tema trata el artículo. ¿A qué factor hace referencia? b) ¿A qué estructura de mercado de factores responde? (competencia perfecta, monopolio

bilateral, monopsonio) ¿Quiénes son las partes y qué ofrecen? ¿De qué gremio se trata? c) Explique la siguiente frase: “Del lado empresario, si bien admiten que hay mayor

actividad, plantean que la oferta salarial que pueden ofrecer está limitada por la fuerte competencia china y brasileña.” ¿A qué hacen referencia los empresarios? ¿Cómo afectará esta situación al nivel de empleo y el nivel de salarios?. Utilice herramientas gráficas.

6) Considere los dos programas siguientes de lucha contra la pobreza: una transferencia de $24 diarios o una transferencia del 40% de la renta salarial. Suponiendo que los pobres tienen una posibilidad de trabajar a cambio de $5 la hora, demuestre como afectaría cada programa a la restricción presupuestaria de un trabajador pobre representativo. ¿Cuál tendría más posibilidades de reducir el número de horas trabajadas? (Frank, 2005) 7) El efecto de un flujo de inmigrantes (Mansfield, 1998) En los últimos años el flujo de inmigrantes desde Asia, Mexico, Cuba y otras áreas hacia los EE.UU fue importante. Suponga que la economía es competitiva (con rendimientos constantes a escala) y que la curva de valor del producto marginal del trabajo es la que se muestra a continuación:

a) Si la inmigración conduce a un aumento desde OL1 hasta OL2 en la oferta de trabajo ¿cuál es el efecto sobre el salario de equilibrio?

b) ¿Cuál es el efecto sobre el monto total de salarios recibido por los trabajadores norteamericanos no inmigrantes?

c) ¿Cuál es el efecto sobre el ingreso total de los propietarios de capital (y de otros recursos no laborables) norteamericanos? Suponga que todo el capital (y otros recursos no laborales) son propiedad de norteamericanos no inmigrantes.

d) Teniendo en cuenta los efectos de la inmigración tanto sobre los salarios recibidos por los trabajadores norteamericanos no inmigrantes como sobre el ingreso de los propietarios del capital (y otros recursos no laborables), ¿hay un beneficio neto para los norteamericanos no inmigrantes? ¿Si así fuere, cuánto es?

8) Lea el siguiente artículo publicado por el diario Clarín el 23 de Agosto de 2008, y conteste las preguntas sugeridas al final del mismo. El desempleo bajó al 8%, pero es más difícil crear nuevos puestos de trabajo Pese al fuerte crecimiento del PBI, se crearon sólo 171.000 puestos en un año. La desocupación bajó a 8% en el segundo trimestre, cinco décimas menos que en la medición del año pasado. Pero aún con la economía en crecimiento, la cantidad de empleos prácticamente sigue siendo la misma. Así, los

L1 L2

G

C

E

Salario, o valor del producto marginal

Cantidad de trabajo

Valor del producto marginal

A

Oferta de trabajo

W1 W2

Guía de ejercicios Microeconomía- UADE

73

números que difundió ayer el INDEC permiten ver que la caída del desempleo continúa desacelerándose. Respecto del segundo trimestre de 2007 la desocupación bajó 0,5%. En ese período, la subocupación llegó a 8,6%, por debajo del 10% año anterior. Lo llamativo es que el desempleo y la subocupación bajan, pero la tasa de empleo -que refleja el porcentaje de la población ocupada respecto de la población total-- permanece prácticamente quieta. Llegó a 42,2%, dos décimas menos que el año anterior. Esto implica que en un año, y con la economía creciendo a tasas del 8%, tan solo se generaron 171.000 empleos. Para el analista Ernesto Kritz, esto "refleja una desaceleración en la creación de puestos de trabajo que ya venía registrándose desde el cuarto trimestre del año pasado. En el mejor de los casos, el empleo crece en forma vegetativa, acompañando el aumento de la población". Con estos datos puede verse que hay 2.656.000 personas con problemas laborales en el país. De éstos, 1.280.000 no tienen empleo y 1.376.000 están subempleados. Además, la tasa de actividad, porcentaje entre la población económicamente activa y la población total, permanece estancada en 45,9%. La medición que cuenta como desempleados a los beneficiarios de los planes sociales registró una desocupación del 8,4%, lo que implica una baja de 1,1% respecto del segundo trimestre de 2007. Así se va cerrando la brecha entre la medición clásica que toma a los beneficiarios como empleados y esta variante que las registra como desocupados. "Estamos en un contexto de desaceleración en la creación de puestos de trabajo e incremento de la informalidad laboral. Es decir, se crean menos y peores puestos de trabajo", señala un informe de la consultora Prefinex. Aún no se conocen los datos del empleo en negro durante el segundo trimestre, pero hay indicios que muestran que las condiciones laborales estarían empeorando. Prefinex aporta dos ejemplos: por un lado, el salario informal presenta una suba del 35%, muy por encima del 20% promedio que obtuvieron los trabajadores formales. Esta mejora obedece a la necesidad de ofrecer ingresos más atractivos para captar trabajadores. Por otro lado hubo un aumento del 7% en la cantidad de horas trabajadas, lo que implica que antes de contratar nuevos empleados, las empresas optan por aumentar la carga horaria de los que ya tienen. En este contexto, el pronóstico de Prefinex es que la tasa de desempleo quedaría en torno al 7,7% como promedio anual.

a) Explique porqué pese a que el desempleo bajó la tasa de empleo continúa siendo la misma. ¿Cómo se calcula dicha tasa? b) En un gráfico de oferta y demanda de trabajo grafique la siguiente situación: “Por otro lado hubo un aumento del 7% en la cantidad de horas trabajadas, lo que implica que antes de contratar nuevos empleados, las empresas optan por aumentar la carga horaria de los que ya tienen” c) ¿Cree usted que si las condiciones económicas mejoraran el desempleo podría llegar a ser nulo? d) En el artículo se mencionan dificultades para que se generen nuevos empleos. ¿El origen de dichas dificultades está en la oferta o en la demanda de trabajo? ¿Cuál es la principal razón por la que se estaría desacelerando la creación de empleos?

Unidad 7

74

AUTOEVALUACIÓN. Puntaje obtenido:

1) Suponga que el salario es w = 20 pesos la hora (2 puntos) (Frank, 2005) a) Halle la ecuación de la restricción presupuestaria de la renta y el ocio y represéntela

gráficamente. b) Suponga que ante este salario un individuo elige h = 14 horas de ocio. Halle la renta diaria M

del trabajador correspondiente a esta cantidad de ocio. 2) Considere los siguientes casos (2 puntos):

a) Juancito quiere ganar $120 por día porque estima que con ese dinero le alcanza de manera suficiente para la vida austera que lleva. ¿Cómo es su curva de oferta de trabajo (Ls)? Grafique para al menos 3 diferentes niveles de salario.

b) Lila exige una hora de ocio por cada $ 10 de salario. Sabiendo que el salario es de w=$20 la hora, halle la demanda óptima de ocio y la cantidad demandada por día.

3) Coloque Verdadero o Falso. Para éste último caso subraye la parte falsa para justificar: (2 puntos)

a) En la competencia perfecta, la curva de oferta de trabajo es la propia curva de oferta del mercado.

b) La renta económica es la diferencia entre lo que se paga a los factores de producción y el pago mínimo necesario para emplear esos factores.

c) La maximización del empleo, la renta económica y los salarios son tres objetivos probables de los sindicatos.

d) El tramo de la curva de oferta de trabajo que se vuelve hacia atrás surge cuando el efecto-sustitución correspondiente a la subida del salario (que fomenta el ocio) es mayor que el efecto-ingreso (que fomenta el trabajo).

4) La demanda de trabajo de una industria viene dada por la curva Ld = 1200 – 10 w, donde L es el trabajo demandado cada día y w es el salario. La curva de oferta viene dada por Ls = 20w. (4 puntos)

a) ¿Cuáles son el salario de equilibrio y la cantidad contratada de trabajo? b) ¿Cuál es la renta económica que ganan los trabajadores? Grafique. c) Suponga ahora que el único trabajo existente es controlado por un sindicato monopolístico que desea maximizar la renta económica que ganan los afiliados. ¿Cuáles serán la cantidad empleada de trabajo y el salario? d) Muestre en un gráfico cuál es la renta económica para los trabajadores sindicados empleados.

Guía de ejercicios Microeconomía- UADE

75

R

Respuestas Sugeridas

1) a) Debe demandar 7 unidades de X. b) Si el precio de X es 15, le conviene utilizar 6 unidades de X, y 9 unidades si el precio baja a $6. c) Utilizará 5 unidades de X 6) a) w = 45; L = 1350; Renta económica del trabajo = $30375 b) w = 102,86; L = 771,43 c) w = 36; L = 1080; La renta económica disminuye (es la menor en los tres casos considerados) 7) Su demanda de ocio será nula (trabajará todas las horas que pueda). Autoevaluación 1)a) Ecuación de la presupuestaria de la renta y el ocio: M = w (24 – h) = 20 (24 - h) = 480 – 20 h b) Si h = 14 horas diarias, la renta diaria es de $200. 2) a) Ls = 120/w b) M = w (24-h) = 480 – 20h. Como M = 10h por el “trade off” luego: 480 – 20h = 10 h h = 16 horas.

3) a) F competencia perfecta → monopsonio b) V c) V d) F efecto-sustitución → efecto ingreso y donde dice efecto-ingreso → efecto ingreso 4) a) Ld = Ls => w = 40 y L = 800. b) Renta económica = (40*800)/2 = 16.000 (Grafico) c) w (Ls) = w (IM) => L = 480 Reemp. en Ld , w = 72. d) Grafico.

Volver al Índice.

Unidad 7

76

Guía de ejercicios Microeconomía- UADE

77

Unidad 8: Incertidumbre, riesgo e información privada. Externalidades.

Preguntas de Repaso 1) ¿Cuándo puede decirse que existe una externalidad? ¿Por qué provocan ineficiencias? De al menos dos ejemplos de externalidades positivas y externalidades negativas. Mencione algunas maneras de resolver las ineficiencias provocadas por las externalidades a través del Estado y a través del mercado. 2) Dados los ejemplos abajo indicados, señale (Krugman 2006, adaptado):

a) ¿Qué tipo de externalidad (positiva o negativa) se describe en cada uno ellos? b) ¿El ingreso marginal social de una actividad es mayor o igual que el ingreso marginal de los

individuos? c) ¿El costo marginal social de una actividad es mayor o igual que el costo marginal de los

individuos? d) Consecuentemente, sin intervención del gobierno, ¿habrá poca o mucha cantidad de ése

bien (en relación con la cantidad socialmente óptima)? La señorita Chai planta muchas flores de colores en el patio. Una marca de ropa muy popular abre una tienda en un centro comercial. Esta tienda

atrae a muchos compradores que visitarán otras tiendas. Su compañero de habitación de al lado pone la música demasiado alta y no lo deja

estudiar. Mirella vive cerca de un campo de flores y decide tener abejas para producir miel. Francisco compra un automóvil todo terreno que consume mucha gasolina.

3) En cada uno de los siguientes casos determine si se genera un costo externo o un ingreso externo y qué políticas deberían aplicarse.

a) Plantar árboles en áreas urbanas mejora la calidad del aire y baja la temperatura en el verano. b) Los sanitarios diseñados para ahorrar agua reducen la necesidad de sacar agua de los ríos y de los acuíferos. El costo de un litro de agua para los usuarios es virtualmente cero. c) Los antiguos monitores de computadoras contienen materiales tóxicos que contaminan el medio ambiente cuando no se retiran de forma apropiada.

4) Indique si las afirmaciones son Verdaderas o Falsas. En caso de que sean falsas, subraye la parte errónea y justifique.

a) Las externalidades son las actividades que una persona o de una empresa que producen un efecto en otra persona o en otra empresa por la que ésta última debe pagar o debe ser pagada.

b) Nunca es posible lograr la eficiencia económica sin tener que recurrir a la intervención del Estado.

c) El Teorema de Coase afirma que siempre que hay externalidades, las partes afectadas pueden unirse y llegar a un acuerdo por el que se internalice la externalidad y se garantice la eficiencia.

d) En presencia de bienes públicos o altos costos de transacción las soluciones públicas tienen grandes dificultades.

5) Algunos detractores de los permisos de emisión comercializables se oponen a ellos porque consideran que las empresas contaminantes que venden sus permisos obtienen un ingreso monetario de su contribución a la contaminación del medioambiente. Discuta este argumento (Krugman, 2006) 6) Explique lo siguiente (Krugman, 2006):

a) Para un nivel de producción igual a QQPT, ¿porqué un impuesto sobre las emisiones que sea mayor o menor que el costo marginal social reduce el excedente total? b) ¿Porqué un sistema de permisos de emisión comercializables que fija la cantidad total de contaminación permitida en un nivel mayor o menor que QQPT reduce el excedente total?

Bibliografía

78

7) Complete en el siguiente esquema los espacios en blanco y de ejemplos de cada caso:

8) Busque en su libro de economía y anote los siguientes conceptos:

Incertidumbre Diversificación Información asimétrica Selección Adversa Riesgo Moral Señalización

9) En base al punto anterior, indique a qué conceptos aluden los siguientes ejemplos didácticos:

a) Un automovilista conduce con menor precaución desde que contrató un seguro automotor. Incluso, no se molesta en chequear si cerró con llaves el auto cuando lo estaciona en cualquier lugar de la calle. b) Un profesor toma un examen de elección múltiple a un curso de 60 alumnos, y 20 alumnos obtienen un 10 como nota. Decide entonces cambiar de manera de evaluar para el segundo examen, con el fin de saber si el alumno incorporó los conocimientos de la materia o la nota obtenida se debió exclusivamente al “factor suerte” (que es lo que el profesor presume). Los alumnos al mismo tiempo afirman públicamente ser merecedores de dicha nota... c) Un estudiante decide vender su computadora de escritorio de manera que con ese dinero y unos ahorros que tiene le alcance para comprar una notebook, que le resulta más práctica por la movilidad, cuando viaja al interior a visitar a sus padres. Sin embargo, a pesar de que su computadora funciona perfectamente, no encuentra ningún comprador que esté dispuesto a pagar el precio que el estudiante considera apropiado para su computadora. Los compradores se preguntan: si funciona tan bien, ¿por qué la vende? d) Una empresa que fabrica navajas multiuso, ideales para camping, a un precio relativamente elevado en relación a sus competidores de mercado ofrece una garantía de por vida contra cualquier defecto, falla o rotura. e) Cuando compramos un billete de lotería para Navidad, sabemos que podemos ganar o que podemos perder. Además tenemos la intuición que es más probable que perdamos, sin embargo, lo compramos y confiamos en nuestra suerte, porque no sabemos con certeza qué ocurrirá realmente...¿y si tal vez nos volvemos millonarios? f) Ud. es un fanático del lema “prevenido vale por dos” por lo cual siempre lleva en su portafolio el paraguas y el bronceador, para estar preparado ante cualquier eventualidad del clima; tiene la mitad de su dinero en dólares y la mitad en pesos; y además tiene acciones en River y en Boca.

10) En cada una de las siguientes situaciones (Krugman, 2006):

a) Determine si hay un problema de riesgo moral o selección adversa.

EXTERNALIDADES

Soluciones ___________________

Soluciones ________________

MULTAS Negociación entre partes: _______ ___ _________

Fusión de firmas: _____________________

SUBVENCIONES

REGULACIONES

SISTEMA JUDICIAL

Estas soluciones fallan cuando existen altos __________ ___ __________ o se presenta el problema del ________ _______ asociado a bienes públicos. Entonces se hace necesaria la intervención de...

Guía de ejercicios Microeconomía- UADE

79

b) Explique porqué hay ineficiencias en cada uno de los casos y de qué manera las soluciones propuestas reducen la ineficiencia.

i. Cuando Ud. adquiere un automóvil de segunda mano no sabe si es un cacharro (baja

calidad) o una ganga (alta calidad), pero el vendedor sí lo sabe. Una solución para el vendedor es ofrecerle una garantía del automóvil, por medio de la cual se hace cargo de los costos de reparación.

ii. Cierta gente es propensa a ir al médico innecesariamente cuando tiene pequeños malestares como dolor de cabeza. Las compañías de seguros médicos no saben la urgencia con que las personas necesitan un médico. Una solución para las aseguradoras es que los asegurados tengan que pagar una pequeña suma (llamado “copago”) cada vez que visitan al médico. Todos los asegurados son aversos al riesgo.

iii. Cuando las compañías aéreas venden billetes no saben si el comprador es un hombre de negocios (que está dispuesto a pagar mucho por un asiento) o un turista (que está dispuesto a pagar menos). Una solución para una empresa maximizadora de beneficios es ofrecer un billete caro que sea muy flexible (permite cambiar fecha y destino) y un billete barato que sea muy rígido (tiene que reservarse con antelación y no se puede cambiar).

iv. Una empresa no sabe si los trabajadores de una cadena de montaje se esfuerzan o, por el contrario, holgazanean. Una solución es contratarlos para que trabajen a destajo, en cuyo caso la remuneración que reciben depende del número de unidades producidas diariamente. Todos los trabajadores son aversos al riesgo, mientras que la empresa no es aversa al riesgo.

v. Cuando lo contratan, los que lo hacen no saben si usted es un trabajador productivo o improductivo. Una posible solución para los trabajadores productivos es proporcionar referencias de trabajos anteriores.

11) ¿En cuál de las siguientes situaciones es más probable que Ud. contrate un seguro de automóvil? (Krugman, 2006)

a) debe trabajar para sobrevivir y necesita el automóvil para ir a trabajar. b) sus padres son ricos y, si lo necesita, pueden comprarle otro automóvil sin ninguna dificultad.

12) ¿Cómo se manifiesta la selección adversa y el riesgo moral en el mercado de seguros?

Bibliografía

80

D

Ejercicios 1) Una acería cree ser dueña del derecho de utilizar el río para verter sus residuos. Los ecologistas a su vez, creen que tienen el derecho a tener un río limpio. Suponga que el costo marginal externo de fabricar acero es: CMgE = 10Q y el costo marginal interno es: CMgI = 20Q. La demanda inversa del acero viene dada por: P = 100 – 5Q.

a) ¿Cuál es el nivel de producción socialmente eficiente? b) ¿Cuánto fabricaría una industria competitiva? c) ¿Cuánto fabricaría un monopolio? d) Analice las acciones que emprendería el gobierno para inducir a las empresas en esta industria a fabricar el nivel de producción socialmente eficiente.

2) Un apicultor vive al lado de un manzanar, cuyo dueño se beneficia de las abejas porque cada colmena poliniza alrededor de un acre de manzanos. Sin embargo, el dueño del manzanar no paga nada por este servicio, porque las abejas acuden al manzanar sin que él tenga que hacer nada. No hay suficientes abejas para polinizar todo el manzanar, por lo que su dueño debe completar la polinización por medios artificiales con un costo de $10 por hectárea de árboles. La apicultura tiene un costo marginal CM = 10 + 2Q, donde Q es el número de colmenas. Cada colmena produce miel por el valor de $20. (Pindyck, 2000)

a) ¿Cuántas colmenas mantendrá el apicultor? b) ¿Es económicamente eficiente este número de colmenas? c) ¿Qué cambios harían que esta actividad fuera más eficiente? Mencione diferentes alternativas

para su solución. 3) Una empresa tiene 20 empleados, y cada uno quiere un entorno laboral más agradable. Por tanto, está analizando la posibilidad de plantar árboles cerca del estacionamiento de la empresa. (Baye, 2006) Cada empleado tiene una demanda inversa de árboles de P = 10 – Q, donde Q es el número de árboles. El coste marginal de plantar los árboles es de 20 dólares cada uno.

a) ¿Cuál es la cantidad socialmente eficiente de árboles que hay que plantar? b) ¿Cuánto tendría que pagar cada empleado por árbol para lograr la cantidad eficiente? c) ¿Cuántos árboles se plantarán? ¿Por qué?

4) La educación es un ejemplo de externalidad positiva: adquirir más educación beneficia a los estudiantes individuales y tener una fuerza de trabajo con un nivel de educación más alto beneficia a la sociedad en su conjunto. La siguiente tabla contiene datos sobre los ingresos y los costos marginales de un año de educación en Sian. Cada año de educación le proporciona a la sociedad un ingreso marginal externo igual a $800. Suponga que el costo marginal social es el mismo que el costo marginal pagado por un estudiante individual (Krugman, 2006).

Cantidad de educación

(años)

Ingreso marginal por

año de educación en

Sian ($)

Costo marginal

por año de educación en Sian ($)

920.000 15.000

10 19.000 16.000

11 18.000 17.000

12 17.000 18.000

13 16.000 19.000

14 15.000 20.000

15 14.000 21.000

16 13.000 22.000

17

Guía de ejercicios Microeconomía- UADE

81

R

D

D

a) Encuentre el número de años de educación que equilibra el mercado de la economía de Sian. b) Calcule la tabla de ingresos marginales sociales. ¿Cuál es el número de años de educación socialmente óptimo? c) Si Ud. estuviera a cargo de los fondos de educación, ¿utilizaría un impuesto pigouviano o una subvención pigouviana para inducir a que en Sian se alcance la cantidad de educación socialmente óptima? ¿Cuál será la cuantía de la subvención o del impuesto por cada año de educación?

5) Don Pepe, antiguo comerciante del barrio, sufre con una regularidad asombrosa la rotura de los cristales de su vidriera (en los últimos 10 años, aproximadamente 1 vez cada 6 meses). Cada vez que se produce este doloroso hecho, Don Pepe debe gastar $480 para reponerlos, y pierde además la posibilidad de vender durante un día entero, con lo que su lucro cesante asciende aproximadamente a $50. Indique que prima a pagar mensualmente a la compañía de seguros “La Veloz” haría equivalente el valor esperado de asegurar los cristales y el de no hacerlo (Lombardero, 1999) 6) La renta de Karina para el año próximo es incierta: con una probabilidad del 60% obtendrá $22.000, y con una probabilidad de 40% obtendrá $35.000. La tabla que se adjunta muestra algunos niveles de renta y utilidad de Karina (Krugman, 2006):

Renta ($) Utilidad total (útiles) 22.000 850 25.000 1014 26.000 1056 35.000 1260

a) ¿Cuál es la renta esperada de Karina? ¿Y su utilidad esperada? b) ¿Qué nivel de renta obtenido con certeza deja indiferente a Karina con la situación en que la

renta es incierta? ¿Qué implicación tiene esto acerca de la actitud de Karina hacia el riesgo? Justifique su respuesta.

c) ¿Estaría Karina dispuesta a pagar alguna cantidad positiva de dinero a cambio de una póliza de seguros que le garantizara una renta de $26.000? Justifique su respuesta.

7) A María le depositan su sueldo mensual de $1100 en el cajero automático. Su función de utilidad viene dada por:

( )U W W= a) Suponga que le ofrecieran un seguro contra robos al costo de $5 por extracción. Sabiendo que la probabilidad de que le roben es de 0.05 y que extrae en una sola vez la totalidad del dinero. ¿Contrataría el seguro? b) ¿Cómo calificaría su actitud frente al riesgo? c) ¿Cómo debería ser la probabilidad de robo para que María sea indiferente entre contratar el seguro y no hacerlo? Calcule analíticamente. d) Si María retirara su sueldo en dos extracciones de igual monto ¿Cambiaría su decisión respecto de contratar el seguro?

8) Suponga que posee $1.000 y que puede invertirlos en la heladería de Ted o en la chocolatería de Ethel. El precio de la acción de ambas empresas es de $100. Los resultados futuros de ambas empresas dependen del tiempo. Cuando hace calor, el valor de cada acción de la heladería de Ted aumenta hasta $150, mientras que el valor de cada acción de la chocolatería de Ethel disminuye hasta $60. Cuando hace frío, el valor de la acción del negocio de Ethel aumenta hasta $150, mientras que el valor la acción del negocio de Ted disminuye hasta $60. La probabilidad de que el tiempo sea cálido o sea frío es la misma (Krugman, 2006).

a) Si invierte todo su dinero en el negocio de Ted, ¿cuál será el valor esperado de sus acciones? ¿Y cual sería si lo invirtiera todo en el negocio de Ethel? b) Suponga que diversifica e invierte la mitad de sus $1000 en cada una de las empresas. ¿Cuál será el valor esperado de todas sus acciones si el tiempo es cálido? ¿Y si hace frío? c) Suponga que es averso al riesgo. ¿Preferiría invertir todo su dinero en el negocio de Ted, como en el apartado a? O por el contrario, ¿preferiría diversificar como en el apartado b? Justifique su respuesta.

Bibliografía

82

D

9) El director de la Empresa XYZ está introduciendo un nuevo producto que ofrecerá unos beneficios de 1.000 dólares si la economía no entra en una recesión. Sin embargo, si se produce una recesión, la demanda de este bien normal caerá tan drásticamente que la empresa perderá 4.000 dólares. Si los economistas estiman que hay un 10 por ciento de posibilidades de que la economía entre en recesión, ¿cuáles son los beneficios esperados de la empresa de introducir el nuevo producto? ¿Qué riesgo tiene la introducción del nuevo producto? (Baye, 2006) 10) Sara tiene una función de utilidad von Neumann-Morgenstern que viene dada por U = 1- 1/M, donde M es el valor actual de la renta que obtendrá durante toda su vida. Si se dedica a la enseñanza, obtendrá M = 5 con una probabilidad de 1. Si se dedica al teatro, obtendrá M = 400 si se convierte en una estrella, pero sólo M = 2 en caso contrario. La probabilidad de que se convierta en una estrella es de 0.01. Sánchez es un juez infalible del talento del actor. Tras una breve entrevista, puede decir con certeza si Sara será una estrella. ¿Cuál es la cantidad máxima que estaría dispuesta a pagar ella por esa información? (Frank, 2006) 11) Un directivo neutro ante el riesgo está intentando contratar a un trabajador. Todos los trabajadores del mercado tienen la misma calidad pero difieren respecto al salario al que están dispuestos a trabajar. Suponga que la mitad de los trabajadores del mercado laboral está dispuesta a trabajar por un salario de 40.000 dólares, y la otra mitad aceptará un salario de 38.000. El directivo pasa tres horas entrevistando a un determinado trabajador y valora su tiempo en 300 dólares. El primer trabajador que entrevista el directivo afirma que sólo trabajará si se la pagan 40.000 dólares. ¿Debería hacer una oferta, o entrevistar a otro trabajador? (Baye, 2006) 12) Suponga que sólo hay cuatro tipo de automóviles, los nuevos y buenos, los nuevos y defectuosos, los usados y buenos, y los usados y defectuosos, y que las siguientes son las cantidades existentes para cada tipo (Mansfield, 1998):

Viejos Nuevos Buenos 20 millones 4 millones

Defectuosos 2 millones 0.4 millones

a) Si uno compra un automóvil nuevo, ¿cuál es la probabilidad de adquirir un automóvil defectuoso?

b) Después de comprar un automóvil nuevo, una persona se entera de si es bueno o defectuoso. Sin embargo, si se ofrece este automóvil a la venta (como automóvil usado), un comprador potencial no podrá determinar (antes de comprarlo) si es bueno o defectuoso. Bajo tales circunstancias, ¿será el precio de equilibrio de un automóvil usado mayor, menor o igual al precio de un automóvil nuevo?

c) ¿Los automóviles defectuosos constituirán una elevada proporción de los automóviles usados en venta?

d) ¿Estaría el comprador de un automóvil usado dispuesto a pagar más que el precio de equilibrio para estar seguro de estar comprando un auto bueno? ¿Estaría el vendedor dispuesto a realizar tal transacción?

e) Si tanto el comprador como el vendedor estarían dispuestos a realizar tal transacción ¿porqué no tiene lugar?

f) ¿Cómo hacen los vendedores de automóviles usados buenos para tratar de obtener un precio mayor al de equilibrio?

13) Un individuo asiste a un salón de juegos y le ofrecen participar en alguno de los siguientes 3 juegos:

Juego 1 : Si sale cara, se ganan $100; si sale cruz, se pierden $0.50. Juego 2 : Si sale cara, se ganan $200; si sale cruz, se pierden $100. Juego 3 : Si sale cara, se ganan $20.000; si sale cruz, se pierden $10.000.

a) ¿Cree Ud. que la persona elegirá el juego con el mayor valor esperado? (Calcúlelos)

b) Suponga que el individuo tiene una función de utilidad de la siguiente forma: ( )U W W= . Si la riqueza inicial es de $10.000 ¿Cuál de los juegos anteriores le proporciona la máxima utilidad esperada. ¿Qué conclusión puede sacar, comparando con el punto anterior?

c) Sabiendo que si este individuo rechaza participar en los juegos, su utilidad, dada su riqueza, sólo será de 100 (verifique), ¿A cuál juego nunca apostaría, en caso de decidirse a jugar?

Guía de ejercicios Microeconomía- UADE

83

R 14) Suponga que tiene $1000 para invertir. Un agente de bolsa lo llama por teléfono y le da alguna información que Ud. le solicitó sobre ciertos bonos basura. Si la empresa que emite los bonos anuncia beneficios este año, le pagará un tipo de interés del 40% por el bono. Si se declara en quiebra Ud. perderá todo lo invertido. Si la empresa no obtiene beneficios ni pérdidas Ud. obtendrá un tipo de interés del 10%. Su agente le dice que hay un 50% de probabilidades de que no obtenga ni beneficios ni pérdidas y un 20% de probabilidades que se declare en quiebra. Su otra opción es invertir en un bono del Estado libre de riesgo que le garantiza un tipo de interés del 8% durante un año.

a) ¿Cuál es el tipo de interés esperado de la inversión en bonos basura? b) ¿Qué inversión elegirá si su función de utilidad es U = M2? c) ¿Qué inversión elegirá si su función de utilidad es de U = √M?

15) Un directivo averso al riesgo está analizando dos proyectos. El primer proyecto implica expandirse en el mercado de salchichas ahumadas; el segundo implica expandirse en el mercado de caviar. Hay una posibilidad del 10 por cierto de que se produzca una recesión y una posibilidad de 90 por ciento de que se produzca una expansión económica. Durante la expansión económica, el proyecto de salchichas ahumadas perderá 10.000 dólares, mientras que el proyecto del caviar ganará 20.000 dólares. Durante la recesión, el proyecto de salchichas ahumadas ganará 12.000 dólares, mientras que el proyecto del caviar perderá 8.000 dólares. Si la alternativa es ganar 3.000 dólares en un activo seguro (por ejemplo, en Bonos de Tesoros) ¿Qué tiene que hacer el directivo? ¿Por qué? (Baye, 2006)

Bibliografía

84

Estudio de Casos 1) Lea el siguiente artículo de la sección Economía de CLARIN, y luego responda las preguntas sugeridas al final del mismo: Clarín, Miércoles 21 de febrero de 2007. Opinión Papeleras: el laberinto tiene salida Un seguro de caución para indemnizar por eventual daño ambiental o la instalación de una planta de biodiesel de segunda generación en Entre Ríos son dos ideas que pueden ayudar a resolver la crisis. Claudio Molina* El conflicto por la papelera Botnia merece, a esta altura de las circunstancias, intentar un salto. El tema ha trascendido fronteras; el pueblo de Gualeguaychú y sus vecinos han mostrado una virtud enorme, dando un ejemplo de unión y perseverancia en pos de un objetivo protectivo. El punto de partida es que hay planteos encontrados sobre los posibles efectos de la puesta en marcha de la planta de Botnia. Si además de la alteración del paisaje, se concretase alguna de las calamidades que agitan a los asambleístas, todos estos comentarios estarían demás. Pero hay fundamentos técnicos respetables que pueden dejar descolocada a la posición argentina en un futuro, si la papelera comenzara a operar. La primera idea propuesta en estas líneas es que Botnia tome un fuerte seguro de caución, para indemnizar a Gualeguaychú y a Entre Ríos en su conjunto por el eventual daño ambiental que pudiera causarse. De emitirse esta póliza, al mismo tiempo, se debería constituir una Comisión Técnica Auditora, integrada por profesionales de reconocida trayectoria en Argentina y Uruguay, más veedores de la ONU y de dos o tres entidades ambientalistas de reconocido prestigio, imparciales, para monitorear el tema. En caso de no cumplirse con los compromisos adquiridos al momento de emisión de la póliza, el Gobierno de Entre Ríos tendría derecho a ejecutar la misma. Veamos ahora cómo convertir la crisis en una oportunidad, típica tarea de buen emprendedor. Hay sinergias poderosas que sustentan una posible solución para el conflicto con ganancias para todos. Entre Ríos está en plena expansión agropecuaria. La producción de soja es la abanderada y cubre el 20% del territorio provincial, con una producción superior a las 3 millones de toneladas. Pero toda la producción sale de la provincia sin industrializarse, retornando parte de ella en forma de pellet, para abastecer el poderoso polo avícola, una actividad de suma importancia que crece sin cesar. El viernes 9 de febrero, el presidente Néstor Kirchner firmó la partida de nacimiento de una nueva y promisoria industria: la de los biocombustibles, destacándose entre éstos, el biodiésel, producto que se obtiene a partir de aceite vegetal o de grasa animal. Es decir, con la soja entrerriana se podría obtener la harina de soja para producir pollos y el aceite para producir biodiésel. Una empresa finlandesa con fuerte ingerencia estatal, la petrolera Neste Oil, está inaugurando en estos días la primera planta de biodiésel del mundo basada en lo que se llama Tecnología de Segunda Generación. Convierte directamente aceite vegetal o grasa animal en biodiésel de alta calidad, a través de un proceso comparable al de las destilerías de petróleo. Neste Oil también tiene en desarrollo la Tercera Generación de Biodiésel, que se basa en el aprovechamiento de la biomasa. Surge entonces la segunda idea: ¿por qué no negociar con Finlandia la instalación de una planta de biodiésel de segunda generación en Entre Ríos, en donación a su gobierno? Con esta donación se compensaría la fatiga que viene sufriendo la población de Gualeguaychú y alrededores. Y con la póliza de seguro de caución, se cubriría el riesgo eventual. La planta a ser donada debe incluir el crushing y la elaboración de biodiésel para el mercado mundial, con escala acorde y tecnología de punta. Son inversiones insignificantes, al lado de lo que Botnia invirtió en Fray Bentos. Posteriormente, esta planta deberá ser vendida por la provincia de Entre Ríos —la donataria— en oferta pública, otorgando prioridad a inversores entrerrianos, principalmente pequeños y medianos productores agropecuarios, pero garantizando un management profesional, exigencia acorde a una nueva forma de hacer política. Además, las islas del sur tienen un gran potencial para producir biomasa, la materia prima que viene para la

Guía de ejercicios Microeconomía- UADE

85

próxima generación de biocombustibles. Y Neste Oil está desarrollando la tecnología. Y hay más. Uruguay también está expandiendo la producción de soja. Y carece de plantas procesadoras. La planta de crushing propuesta podría tomar la soja uruguaya y abastecer al país hermano con biodiésel. Así como fluiría la soja uruguaya a la planta argentina, la madera argentina iría a la planta uruguaya. El descarte de Botnia vendría a procesarse en el futuro en la planta de biodiésel. Pareciera que todos ganan. ¿Quién pierde? Quizá, sólo el que medra en el laberinto. *DIRECTOR EJECUTIVO DE LA ASOCIACION ARGENTINA DE BIOCOMBUSTIBLES E HIDROGENO

a) ¿De acuerdo al artículo quién estaría creando una externalidad negativa y porqué? (asegúrese de definir qué es una externalidad). b) ¿Porqué las externalidades provocan ineficiencias? ¿Cuáles serían las consecuencias concretas en éste caso? Ilustre mediante un gráfico e indique cuál podría ser el nivel óptimo de producción (y/o contaminación). ¿Cree Ud. que los ecologistas estarían de acuerdo?¿Y los economistas? c) Discuta con sus compañeros de clase los posibles costos y beneficios de instalar la papelera. ¿Quién cree Ud. que posee “derechos de propiedad” sobre el medio ambiente? d) ¿Qué tipo de soluciones propone el autor del artículo para llegar a un acuerdo? ¿Es una solución pública o privada? ¿Por qué podrían fallar las soluciones privadas?

2) Caso: Ingresos recaudados a partir de los aranceles sobre los efluentes (Mansfield, 1998). El gobierno está estudiando la posibilidad de imponer un arancel sobre los efluentes contaminantes que la industria química descarga en las vías fluviales locales. Pueden utilizarse los ingresos obtenidos a partir del arancel sobre los efluentes para reducir los impuestos del público en general o para compensar a los usuarios del agua por la contaminación del agua que la industria provoca. Abajo se muestra la curva de demanda de materiales químicos. SS’ es la curva de oferta de materiales químicos (para simplificar, suponemos que es horizontal). El daño por contaminación que cada tonelada de producción de materiales químicos les impone a los usuarios del agua es OH y el arancel sobre los efluentes por cada tonelada de producción es ST, donde ST = OH. ¿Cuál es el efecto sobre el público general y sobre los usuarios del agua, de cada forma de utilizar los ingresos obtenidos a partir del arancel de los efluentes? ¿Qué forma es más equitativa? 3) Lea el siguiente artículo de la sección Economía de CLARIN, y luego responda las preguntas sugeridas al final del mismo:

E F

G

A

B

0 Q1 Q2 Producto

Precio

T S H

T’ S’ H’

Demanda

Bibliografía

86

Jueves 8 de septiembre de 2005 NUEVAS NORMAS: DIFERENTES BLOQUES DE LA LEGISLATURA DE LA CIUDAD TRABAJAN JUNTOS EN UN PROYECTO

Hay consenso para prohibir fumar en los espacios públicos Ahora el consumo está restringido sólo en los sectores de atención al público de las oficinas del Estado. Los diputados buscan ampliarlo a otros lugares, como shoppings, cibercafés y salas de fiesta. Daniel Gutman. [email protected] Para los fumadores porteños se avecinan tiempos cada vez más difíciles. De la misma manera que sucede en muchas otras ciudades importantes del mundo, Buenos Aires va a camino a convertirse en un sitio cada vez menos amigable para el cigarrillo. Después de que se presentaran varios proyectos distintos con el mismo espíritu, legisladores locales de todo el arco político están cerrando un consenso alrededor de una ley que restringirá severamente las posibilidades de fumar en espacios públicos de la Ciudad. Si bien los diputados todavía negocian la letra chica, lo más grueso del proyecto que sería aprobado en las próximas semanas por la Legislatura ya está acordado. Algunos de los puntos son estos: � Se prohibirá fumar en todos los lugares con acceso al público. En las oficinas del Estado porteño no se puede fumar desde 1994, debido a una ordenanza del ex Concejo Deliberante. Ahora se agregarán una cantidad de lugares privados: teatros, cines, shoppings, estadios cerrados, salas de fiestas, ascensores, cabinas telefónicas, recintos de cajeros automáticos, terminales ómnibus de mediana y larga distancia, cibercafés. � Se limitará la publicidad de cigarrillos en la vía pública. Podría acordarse la prohibición total, que es lo que impulsa el interbloque de izquierda. � En los restorantes y bares se cerrará fuertemente el cerco sobre los fumadores, aunque todavía hay varias alternativas en danza. El socialista Norberto La Porta es el más duro: propone que sólo se pueda fumar en espacios al aire libre. La macrista Paula Bertol y el kirchnerista Helio Rebot quieren que se les imponga a los locales la instalación de una separación física (una pared o un vidrio) entre el sector de no fumadores y el de no fumadores. La peronista Alicia Bello plantea que haya tres tipos de habilitaciones distintas para los bares y restorantes: para fumadores, para no fumadores y mixtos. Estos últimos deberían tener un mínimo de 40 metros cuadrados. � Las obras sociales deberán incluir en sus coberturas la prevención y el tratamiento del tabaquismo. � Se creará un comité interdisciplinario con funcionarios y miembros de organismos no gubernamentales que deberá diseñar un programa de reducción, prevención y control del uso del tabaco. � Se aplicarán multas de hasta el valor de 500 paquetes de cigarrillos (un promedio de $ 1.500) a los responsables del lugar donde se detecten incumplimientos a la prohibición de fumar. "La idea, básicamente, es proteger a quien no fuma, pero es un fumador pasivo", explicó la legisladora Susana Etchegoyen, del interbloque de izquierda. De acuerdo a una estadística que difundió el lunes el ministro de Salud, Ginés González García, unos 6.000 fumadores pasivos mueren anualmente en la Argentina a causa del tabaco. "Los que no fuman pero conviven con fumadores sufren desde irritación ocular y lagrimeo hasta cáncer de pulmón. Es una locura", dijo a Clarín la diputada Bertol, que ayer encabezó junto a Rebot y a Julio De Giovanni, del Partido de la Ciudad, una jornada legislativa "Por el derecho de los no fumadores", en la que participaron especialistas de distintas áreas. El ministerio de Salud de la Nación, por su lado, ya tiene preparado un proyecto de Ley Nacional Antitabáquica que prohibirá fumar en espacios cerrados públicos y privados, y restringirá la publicidad de cigarrillos. Esa ley actuará como paraguas para la legislación de las provincias y de la Ciudad de Buenos Aires, que podrán establecer restricciones más severas para la actividad de los fumadores. Esta tendencia ya se advierte en una cantidad de provincias y es bien actual el caso de Tucumán, que esta misma semana aprobó una ley que prohíbe fumar en ámbitos cerrados, que entrará en vigencia dentro de un año. En la Legislatura porteña, la cuestión todavía pasar el filtro de tres comisiones —Salud, Desarrollo Económico y Justicia— para llegar al recinto, donde los antifumadores ya dan por seguros un mínimo de 35 votos, suficientes para aprobar la ley.

a) Las tendencias sociales recientes apuntan a una creciente intolerancia hacia el consumo de tabaco en zonas públicas. Las investigaciones médicas han mostrado los efectos negativos que produce en la salud de los fumadores “pasivos”. Si Ud. fuma y desea continuar fumando a pesar del endurecimiento de la legislación contra el tabaco, describa el efecto que produciría en su conducta cada una de las propuestas legislativas mencionadas en el artículo.

Guía de ejercicios Microeconomía- UADE

87

b) Como consecuencia de estos programas, ¿se beneficia Ud. como fumador individual? ¿Se beneficia la sociedad en su conjunto?

Bibliografía

88

AUTOEVALUACIÓN. Puntaje obtenido: 1) Es Ud. un analista de la industria, especializado en una industria donde la demanda inversa del mercado es P = 100 – Q. El Costo marginal externo de fabricar el producto es CMg Externo = 3Q, y el coste interno es CMg Interno = 6Q.( 3 puntos)

a) ¿Cuál es el nivel de producción socialmente eficiente? b) Dados los costes y demanda del mercado ¿qué producción fabricaría una industria competitiva? c) Dados los costes y demanda del mercado ¿qué producción fabricaría un monopolista?

2) Coloque V o F y subraye la parte falsa para justificar (2 puntos) a) El teorema de Coase, indica que los individuos pueden encontrar una forma de internalizar la externalidad, haciendo intervenir al gobierno. b) Dos métodos eficientes para reducir la contaminación son los impuestos sobre las emisiones y los permisos comercializables. Para los economistas la regulación medioambiental es ineficiente. c) Cuando el desarrollo de un bien genera externalidades positivas, si el estado no interviene, el nivel de producción en ese tipo de industria será mayor que el socialmente óptimo. Una subvención pigouviana óptima a los productores, igual al ingreso marginal externo, llevará a producir una cantidad del bien socialmente óptima. d) Los elevados costos de transacción son aquellos que impiden resolver el problema de externalidades al sector privado por sí mismo.

3) Suponga que el 25% de las computadoras personales nuevas son defectuosas. Sin embargo, las defectuosas no pueden ser identificadas salvo por los dueños. Los consumidores son neutrales ante el riesgo y conceden a las computadoras que no son defectuosas un valor de $2000 cada una. Suponemos que las computadoras no se deprecian físicamente con el uso. Si las usadas se venden por $600, ¿por cuánto dinero se venderán las nuevas? (Frank, 2006) (2 puntos). 4) Coloque V o F y subraye la parte falsa para justificar (3 puntos)

a) Los individuos racionales eligen siempre los juegos con mayor valor esperado. b) El problema del principal y el agente es básicamente un problema de selección adversa. c) El riesgo moral no sólo altera la conducta sino que también crea ineficiencia económica ya

que una vez asegurado la percepción del costo o beneficio de la actividad económica que tiene el individuo es diferente del costo o beneficio social.

d) La señalización y la reputación son dos medios que tienen los vendedores de convencer a los compradores que sus productos son de buena calidad en mercados donde la información es asimétrica.

e) Si la probabilidad de robos en cajeros automáticos es del 0.001% la persona preferiría extraer todo de una vez. En cambio, si la probabilidad aumentara a 0.01%, podría reducir el riesgo extrayendo en un mayor número de veces montos más pequeños.

f) Un individuo averso al riego preferiría un cartera con mayor riesgo y mayor rentabilidad que la que elegiría un individuo neutral al riesgo.

Guía de ejercicios Microeconomía- UADE

89

D

R

Respuestas Sugeridas

1) a) Cantidad socialmente óptima P = CMgsocial = CMginterno + CMgexterno

100 – 5Q = 10Q + 20Q = 30Q 100 = 35 Q Q = 2,86 ; P = 85,71

b) Competencia perfecta P = CMginterno 100 – 5Q = 20Q Q = 4 ; P = 80 c) Monopolio IMg = CMginterno 100 – 10Q = 20Q Q = 3,33 ; P = 83,33 6) a) Renta esperada = 22.000 * 0,6 + 35.000 * 0,4 = 27.200 Utilidad esperada = 850 * 0,6 + 1260 * 0,4 = 1014 b) Karina es aversa al riesgo, ya que un nivel de renta cierta de $25.000 le da la misma utilidad que una renta incierta de $27.200. c) Si, ya que al aumentar su renta cierta, mejora su nivel de utilidad, y estará mejor que con los $27.200 inciertos. 7) a) Con seguro Renta (cierta) = $1.095 Utilidad (cierta) = 33,09 Sin seguro Renta (esperada) = $1.100 * 0,95 + 0 * 0,05 = $1045 Utilidad (esperada) = 31,51 Por lo tanto, como su utilidad es mayor contratando el seguro, María contratará la póliza. b) Es aversa al riesgo (su utilidad es mayor con seguro que sin él) c) La probabilidad del robo debería ser tal que su utilidad esperada sin seguro sea igual a la utilidad con seguro. O sea, (1- x) * √1100 + x * √0 = 33,09 x = 0,0023 (riesgo de robo) d) Si hace dos extracciones, extrae $550 por vez. Suponiendo que el riesgo de robo del 5% es por cada vez que va al cajero (y no por mes) tenemos: Con seguro Renta (cierta) = $545 Utilidad (cierta) = 23,345 Sin seguro Renta (esperada) = $522,50 Utilidad (esperada) = 22,28 Por lo tanto le sigue conviniendo contratar el seguro. 13) a) Valores esperados: Juego 1: 100 * 0,5 + (-0,50)*0,5 = 49,75 Juego 2: 200 * 0,5 + (-100) * 0,5 = 50 Juego 3: 20.000 * 0,5 + (-10.000) * 0,5 = 5.000 No necesariamente elegirá el juego de mayor valor esperado. Su elección dependerá de su utilidad frente al riesgo. b) Utilidad inicial (sin jugar) = √10.000 = 100 Utilidad esperada del juego 1: √10.100 * 0,5 + √9.999,50 * 0,5 = 100,25 Utilidad esperada del juego 2: √10.200 * 0,5 + √9.900 * 0,5 = 100,25 Utilidad esperada del juego 3: √30.000 * 0,5 + √0 * 0,5 = 86,60 Prefiere jugar los juegos 1 y 2 al 3 (1 y 2 le son indiferentes entre sí). Nunca elegirá el 3 si tiene una renta inicial de $10.000 ya que la renta esperada del juego 3 es menor que la que tiene sin jugar. Es averso al riesgo, por eso no elige el juego de mayor valor esperado.

5) Prima = $88,33 14) a) Interés esperado = -3 % (negativo) b) Preferirá invertir en las acciones, ya que la utilidad es mayor (1.193.000 de las acciones contra 1.166.400 del bono del Estado). c) Preferirá comprar el bono, ya que le reporta una utilidad mayor (32,86 del bono contra 19,21 de las acciones).

Bibliografía

90

Autoevaluación 1) a) CMg Social = CMg Externo + Cmg Interno = 3Q + 6Q = 9Q P = 9Q => 100 – Q = 9Q => Q = 10 unidades.

b) P = Cmg Interno 100 – Q = 6Q => Q = 14.3 unidades c) Img = Cmg Interno 100 – 2Q = 6Q => Q = 12.5 unidades

2) a) F haciendo intervenir al gobierno → sin que intervenga el gobierno.

b) V c) F mayor → menor d) V

3) Como los dueños de computadoras nuevas no podrían venderlas en el mercado de segunda mano por lo que valen para ellos, sólo las computadoras defectuosas usadas estarían en venta. Si las usadas se venden a $600, el valor de una computadora nueva para un comprador neutral al riesgo debe ser: 0.25 * 600 + 0.75*2000 = $1.650 4) a) F valor esperado → utilidad esperada.

b) F selección adversa → información asimétrica. c) V d) V e) V f) F averso al riego → propenso al riesgo.

Volver al Índice.

Guía de ejercicios Microeconomía- UADE

91

Bibliografía.

Baye, Michael. Economía de Empresa. 5ta ed. Madrid: McGraw Hill, 2006.

Frank, Robert H. Microeconomía y conducta. 4a ed. Madrid : McGraw Hill, 2001.

Lombardero de Almeida, Ana. Problemas de microeconomía. Editorial Machi, 1999.

Krugman, Paul y Wells Robin. Introducción a la economía. Microeconomía. 1ra edición. Editorial Reverté, 2006.

Mansfield, Edwin. Microeconomía. Teoría y Aplicaciones. 5° edición. Grupo editorial Norma, 1998.

Nicholson, Walter. Microeconomía intermedia y sus aplicaciones. Novena Edición,2005. Thomson Ediciones

Pindyck, Robert S. y Rubinfeld, Daniel L. y Beker, Víctor Alberto. Microeconomía. Buenos Aires : Prentice Hall, Pearson Educación, 2000

Samuelson, P. A, Nordhaus, W. y Pérez Enrri, D. Economía, 1° edición. McGraw-Hill, Buenos Aires, 2003.

Varian, H. R. Microeconomía Intermedia, 5° edición. Antoni Bosch, España, 1999

Sitios Web:

www.mhhe.com/economics/baye5e

www.worthpublishers.com/krugmanwells

www.clarin.com

www.lanacion.com

www.mecon.gov.ar

www.indec.mecon.gov.ar

www.worldbank.org

www.iadb.org;

www.cepal.org.ar

www.ucm.edu.es/info/microint

Volver al Índice.

Bibliografía

92

Guía de ejercicios prácticos – Microeconomía, UADE.

Segunda Edición.

Buenos Aires, Diciembre de 2008.